Merge branch 'master' of ssh://git_opytex:/lafrite/2020-2021
continuous-integration/drone/push Build is passing
Details
continuous-integration/drone/push Build is passing
Details
This commit is contained in:
commit
de988adecf
|
@ -0,0 +1,136 @@
|
||||||
|
\documentclass[a4paper,10pt]{article}
|
||||||
|
\usepackage{myXsim}
|
||||||
|
|
||||||
|
% Title Page
|
||||||
|
\title{DS8 \hfill BAHBAH Zakaria}
|
||||||
|
\tribe{TST sti2d}
|
||||||
|
\date{\hfillÀ render pour le vendredi 9 avril à 10h au plus tard}
|
||||||
|
|
||||||
|
\xsimsetup{
|
||||||
|
solution/print = false
|
||||||
|
}
|
||||||
|
|
||||||
|
\begin{document}
|
||||||
|
\maketitle
|
||||||
|
|
||||||
|
\begin{exercise}[subtitle={Étude de fonction}]
|
||||||
|
On considère la fonction $f$ définie sur $\intOF{0}{+\infty}$ par $ f(x) = 4x^2 + 72x + 160\ln(x)$
|
||||||
|
\begin{enumerate}
|
||||||
|
\item Démontrer que la dérivée de $f$ est $f'(x) = \frac{8x^2 + 72x + 160}{x}$.
|
||||||
|
\item Étude du numérateur de $f'(x)$: $N(x) = 8x^2 + 72x + 160$
|
||||||
|
\begin{enumerate}
|
||||||
|
\item Démontrer que $x=- 4$ et $x=- 5$ sont deux racines de $N(x)$..
|
||||||
|
\item Proposer une forme factorisée de $N(x)$.
|
||||||
|
\item Proposer une forme factorisée de $f'(x)$.
|
||||||
|
\end{enumerate}
|
||||||
|
\item Étudier le signe de $f'$ et en déduire les variations de $f$.
|
||||||
|
\end{enumerate}
|
||||||
|
\end{exercise}
|
||||||
|
|
||||||
|
\begin{solution}
|
||||||
|
\begin{enumerate}
|
||||||
|
\item pas de correction disponible
|
||||||
|
\item
|
||||||
|
\begin{enumerate}
|
||||||
|
\item \[N(- 4) = 0\]
|
||||||
|
\[N(- 5) = 0\]
|
||||||
|
\item \[
|
||||||
|
N(x) = 8(x - - 4)(x - - 5)
|
||||||
|
\]
|
||||||
|
\item
|
||||||
|
\[
|
||||||
|
f'(x) = \frac{8(x - - 4)(x - - 5)}{x}
|
||||||
|
\]
|
||||||
|
\end{enumerate}
|
||||||
|
\item Pas de correction disponible
|
||||||
|
\end{enumerate}
|
||||||
|
\end{solution}
|
||||||
|
|
||||||
|
\begin{exercise}[subtitle={Complexes}]
|
||||||
|
\begin{enumerate}
|
||||||
|
\item Mettre le nombre complexe suivant sous forme algébrique $z_1 = \dfrac{2 + 8 i}{-9 + 3 i} $
|
||||||
|
\item Mettre le complexe suivante sous forme exponentielle $z_2 = 10 \sqrt{3} - 10 i$
|
||||||
|
\item Mettre le complexe suivante sous forme exponentielle $z_3 = 7 + 7 \sqrt{3} i$
|
||||||
|
\item Calculer le produit $z_4=z_2\times z_3$ donner le résultat sous forme exponentielle puis algébrique.
|
||||||
|
\item Calculer le quotient $z_5=\frac{z_2}{z_3}$ donner le résultat sous forme exponentielle puis algébrique.
|
||||||
|
\end{enumerate}
|
||||||
|
\end{exercise}
|
||||||
|
|
||||||
|
\begin{solution}
|
||||||
|
\begin{enumerate}
|
||||||
|
\item $z_1 = \frac{1}{15} - \frac{13 i}{15}$
|
||||||
|
\item $z_2 = 20 e^{- \frac{i \pi}{6}}$
|
||||||
|
\item $z_3 = 14 e^{\frac{i \pi}{3}}$
|
||||||
|
\item $z_4 = 280 e^{\frac{i \pi}{6}} = 140 \sqrt{3} + 140 i = 243.0 + 140.0 i$
|
||||||
|
\item $z_5 = \frac{10}{7} e^{- \frac{i \pi}{2}} = - \frac{10 i}{7} = - 1.43 i$
|
||||||
|
\end{enumerate}
|
||||||
|
\end{solution}
|
||||||
|
|
||||||
|
\begin{exercise}[subtitle={Sortie du congélateur}]
|
||||||
|
Marie a invité quelques amis pour le thé. Elle souhaite leur proposer ses macarons maison.
|
||||||
|
|
||||||
|
Elle les sort de son congélateur à $-17$~\degres C et les place dans une pièce à $16$~\degres C.
|
||||||
|
|
||||||
|
Au bout de 15 minutes, la température des macarons est de $-3$~\degres C.
|
||||||
|
|
||||||
|
\bigskip
|
||||||
|
|
||||||
|
\textbf{Premier modèle}
|
||||||
|
|
||||||
|
\medskip
|
||||||
|
|
||||||
|
On suppose que la vitesse de décongélation est constante : chaque minute la hausse de
|
||||||
|
température des macarons est la même.
|
||||||
|
|
||||||
|
Estimer dans ce cadre la température au bout de $30$~minutes, puis au bout de $45$~minutes.
|
||||||
|
|
||||||
|
Cette modélisation est-elle pertinente?
|
||||||
|
|
||||||
|
\bigskip
|
||||||
|
|
||||||
|
\textbf{Deuxième modèle}
|
||||||
|
|
||||||
|
\medskip
|
||||||
|
|
||||||
|
On suppose maintenant que la vitesse de décongélation est proportionnelle à la différence
|
||||||
|
de température entre les macarons et l'air ambiant (il s'agit de la loi de Newton).
|
||||||
|
|
||||||
|
On désigne par $\theta$ la température des macarons à l'instant $t$, et par $\theta'$ la vitesse de décongélation.
|
||||||
|
|
||||||
|
L'unité de temps est la minute et l'unité de température le degré Celsius.
|
||||||
|
|
||||||
|
\smallskip
|
||||||
|
|
||||||
|
On négligera la diminution de température de la pièce et on admettra donc qu'il existe un
|
||||||
|
nombre réel $a$ tel que, pour $t$ positif :
|
||||||
|
|
||||||
|
\[\theta'(t) = a [\theta(t) - 16]\quad (E)\]
|
||||||
|
|
||||||
|
\medskip
|
||||||
|
|
||||||
|
\begin{enumerate}
|
||||||
|
\item Vérifier que l'équation $(E)$ a pour solutions $\theta(t) = K e^{at} + 16$ où $K$ est un nombre réel.
|
||||||
|
|
||||||
|
Donner alors, en fonction de $a$, l'ensemble des solutions de $(E)$.
|
||||||
|
\end{enumerate}
|
||||||
|
On rappelle que la température des macarons à l'instant $t = 0$ est égale à $-17$~\degres C et que, au bout de $15$~min, elle est de $-3$~\degres C.
|
||||||
|
\begin{enumerate}
|
||||||
|
\setcounter{enumi}{1}
|
||||||
|
\item En utilisant la condition à $t=0$ démontrer que $K = -33$.
|
||||||
|
\item En utilisant la condition à $t=15$ démontrer que $a \approx -0.04$.
|
||||||
|
\item En déduire l'expression de la solution de l'équation différentielle puis étudier ses variations.
|
||||||
|
\item La température idéale de dégustation des macarons étant de $13$~\degres C, Marie estime que
|
||||||
|
celle-ci sera atteinte au bout de $30$~min. A-t-elle raison ? Justifier la réponse.
|
||||||
|
|
||||||
|
Sinon, combien de temps faudra-t-il attendre ?
|
||||||
|
\end{enumerate}
|
||||||
|
\end{exercise}
|
||||||
|
|
||||||
|
|
||||||
|
|
||||||
|
\end{document}
|
||||||
|
|
||||||
|
%%% Local Variables:
|
||||||
|
%%% mode: latex
|
||||||
|
%%% TeX-master: "master"
|
||||||
|
%%% End:
|
|
@ -0,0 +1,136 @@
|
||||||
|
\documentclass[a4paper,10pt]{article}
|
||||||
|
\usepackage{myXsim}
|
||||||
|
|
||||||
|
% Title Page
|
||||||
|
\title{DS8 \hfill BENALI Ilyas}
|
||||||
|
\tribe{TST sti2d}
|
||||||
|
\date{\hfillÀ render pour le vendredi 9 avril à 10h au plus tard}
|
||||||
|
|
||||||
|
\xsimsetup{
|
||||||
|
solution/print = false
|
||||||
|
}
|
||||||
|
|
||||||
|
\begin{document}
|
||||||
|
\maketitle
|
||||||
|
|
||||||
|
\begin{exercise}[subtitle={Étude de fonction}]
|
||||||
|
On considère la fonction $f$ définie sur $\intOF{0}{+\infty}$ par $ f(x) = 2.5x^2 + - 95x + 450\ln(x)$
|
||||||
|
\begin{enumerate}
|
||||||
|
\item Démontrer que la dérivée de $f$ est $f'(x) = \frac{5x^2 + - 95x + 450}{x}$.
|
||||||
|
\item Étude du numérateur de $f'(x)$: $N(x) = 5x^2 - 95x + 450$
|
||||||
|
\begin{enumerate}
|
||||||
|
\item Démontrer que $x=9$ et $x=10$ sont deux racines de $N(x)$..
|
||||||
|
\item Proposer une forme factorisée de $N(x)$.
|
||||||
|
\item Proposer une forme factorisée de $f'(x)$.
|
||||||
|
\end{enumerate}
|
||||||
|
\item Étudier le signe de $f'$ et en déduire les variations de $f$.
|
||||||
|
\end{enumerate}
|
||||||
|
\end{exercise}
|
||||||
|
|
||||||
|
\begin{solution}
|
||||||
|
\begin{enumerate}
|
||||||
|
\item pas de correction disponible
|
||||||
|
\item
|
||||||
|
\begin{enumerate}
|
||||||
|
\item \[N(9) = 0\]
|
||||||
|
\[N(10) = 0\]
|
||||||
|
\item \[
|
||||||
|
N(x) = 5(x - 9)(x - 10)
|
||||||
|
\]
|
||||||
|
\item
|
||||||
|
\[
|
||||||
|
f'(x) = \frac{5(x - 9)(x - 10)}{x}
|
||||||
|
\]
|
||||||
|
\end{enumerate}
|
||||||
|
\item Pas de correction disponible
|
||||||
|
\end{enumerate}
|
||||||
|
\end{solution}
|
||||||
|
|
||||||
|
\begin{exercise}[subtitle={Complexes}]
|
||||||
|
\begin{enumerate}
|
||||||
|
\item Mettre le nombre complexe suivant sous forme algébrique $z_1 = \dfrac{10 + 5 i}{-3 + 10 i} $
|
||||||
|
\item Mettre le complexe suivante sous forme exponentielle $z_2 = - 5 \sqrt{3} + 5 i$
|
||||||
|
\item Mettre le complexe suivante sous forme exponentielle $z_3 = - 8 \sqrt{2} - 8 \sqrt{2} i$
|
||||||
|
\item Calculer le produit $z_4=z_2\times z_3$ donner le résultat sous forme exponentielle puis algébrique.
|
||||||
|
\item Calculer le quotient $z_5=\frac{z_2}{z_3}$ donner le résultat sous forme exponentielle puis algébrique.
|
||||||
|
\end{enumerate}
|
||||||
|
\end{exercise}
|
||||||
|
|
||||||
|
\begin{solution}
|
||||||
|
\begin{enumerate}
|
||||||
|
\item $z_1 = \frac{20}{109} - \frac{115 i}{109}$
|
||||||
|
\item $z_2 = 10 e^{\frac{5 i \pi}{6}}$
|
||||||
|
\item $z_3 = 16 e^{- \frac{3 i \pi}{4}}$
|
||||||
|
\item $z_4 = 160 e^{\frac{i \pi}{12}} = 40 \sqrt{2} + 40 \sqrt{6} + i \left(- 40 \sqrt{2} + 40 \sqrt{6}\right) = 155.0 + 41.4 i$
|
||||||
|
\item $z_5 = \frac{5}{8} e^{\frac{19 i \pi}{12}} = - \frac{5 \sqrt{2}}{32} + \frac{5 \sqrt{6}}{32} + i \left(- \frac{5 \sqrt{6}}{32} - \frac{5 \sqrt{2}}{32}\right) = 0.162 - 0.604 i$
|
||||||
|
\end{enumerate}
|
||||||
|
\end{solution}
|
||||||
|
|
||||||
|
\begin{exercise}[subtitle={Sortie du congélateur}]
|
||||||
|
Marie a invité quelques amis pour le thé. Elle souhaite leur proposer ses macarons maison.
|
||||||
|
|
||||||
|
Elle les sort de son congélateur à $-17$~\degres C et les place dans une pièce à $17$~\degres C.
|
||||||
|
|
||||||
|
Au bout de 15 minutes, la température des macarons est de $-1$~\degres C.
|
||||||
|
|
||||||
|
\bigskip
|
||||||
|
|
||||||
|
\textbf{Premier modèle}
|
||||||
|
|
||||||
|
\medskip
|
||||||
|
|
||||||
|
On suppose que la vitesse de décongélation est constante : chaque minute la hausse de
|
||||||
|
température des macarons est la même.
|
||||||
|
|
||||||
|
Estimer dans ce cadre la température au bout de $30$~minutes, puis au bout de $45$~minutes.
|
||||||
|
|
||||||
|
Cette modélisation est-elle pertinente?
|
||||||
|
|
||||||
|
\bigskip
|
||||||
|
|
||||||
|
\textbf{Deuxième modèle}
|
||||||
|
|
||||||
|
\medskip
|
||||||
|
|
||||||
|
On suppose maintenant que la vitesse de décongélation est proportionnelle à la différence
|
||||||
|
de température entre les macarons et l'air ambiant (il s'agit de la loi de Newton).
|
||||||
|
|
||||||
|
On désigne par $\theta$ la température des macarons à l'instant $t$, et par $\theta'$ la vitesse de décongélation.
|
||||||
|
|
||||||
|
L'unité de temps est la minute et l'unité de température le degré Celsius.
|
||||||
|
|
||||||
|
\smallskip
|
||||||
|
|
||||||
|
On négligera la diminution de température de la pièce et on admettra donc qu'il existe un
|
||||||
|
nombre réel $a$ tel que, pour $t$ positif :
|
||||||
|
|
||||||
|
\[\theta'(t) = a [\theta(t) - 17]\quad (E)\]
|
||||||
|
|
||||||
|
\medskip
|
||||||
|
|
||||||
|
\begin{enumerate}
|
||||||
|
\item Vérifier que l'équation $(E)$ a pour solutions $\theta(t) = K e^{at} + 17$ où $K$ est un nombre réel.
|
||||||
|
|
||||||
|
Donner alors, en fonction de $a$, l'ensemble des solutions de $(E)$.
|
||||||
|
\end{enumerate}
|
||||||
|
On rappelle que la température des macarons à l'instant $t = 0$ est égale à $-17$~\degres C et que, au bout de $15$~min, elle est de $-1$~\degres C.
|
||||||
|
\begin{enumerate}
|
||||||
|
\setcounter{enumi}{1}
|
||||||
|
\item En utilisant la condition à $t=0$ démontrer que $K = -34$.
|
||||||
|
\item En utilisant la condition à $t=15$ démontrer que $a \approx -0.04$.
|
||||||
|
\item En déduire l'expression de la solution de l'équation différentielle puis étudier ses variations.
|
||||||
|
\item La température idéale de dégustation des macarons étant de $14$~\degres C, Marie estime que
|
||||||
|
celle-ci sera atteinte au bout de $30$~min. A-t-elle raison ? Justifier la réponse.
|
||||||
|
|
||||||
|
Sinon, combien de temps faudra-t-il attendre ?
|
||||||
|
\end{enumerate}
|
||||||
|
\end{exercise}
|
||||||
|
|
||||||
|
|
||||||
|
|
||||||
|
\end{document}
|
||||||
|
|
||||||
|
%%% Local Variables:
|
||||||
|
%%% mode: latex
|
||||||
|
%%% TeX-master: "master"
|
||||||
|
%%% End:
|
|
@ -0,0 +1,136 @@
|
||||||
|
\documentclass[a4paper,10pt]{article}
|
||||||
|
\usepackage{myXsim}
|
||||||
|
|
||||||
|
% Title Page
|
||||||
|
\title{DS8 \hfill BERNADAT Noah}
|
||||||
|
\tribe{TST sti2d}
|
||||||
|
\date{\hfillÀ render pour le vendredi 9 avril à 10h au plus tard}
|
||||||
|
|
||||||
|
\xsimsetup{
|
||||||
|
solution/print = false
|
||||||
|
}
|
||||||
|
|
||||||
|
\begin{document}
|
||||||
|
\maketitle
|
||||||
|
|
||||||
|
\begin{exercise}[subtitle={Étude de fonction}]
|
||||||
|
On considère la fonction $f$ définie sur $\intOF{0}{+\infty}$ par $ f(x) = 3.5x^2 + - 21x + - 196\ln(x)$
|
||||||
|
\begin{enumerate}
|
||||||
|
\item Démontrer que la dérivée de $f$ est $f'(x) = \frac{7x^2 + - 21x + - 196}{x}$.
|
||||||
|
\item Étude du numérateur de $f'(x)$: $N(x) = 7x^2 - 21x - 196$
|
||||||
|
\begin{enumerate}
|
||||||
|
\item Démontrer que $x=- 4$ et $x=7$ sont deux racines de $N(x)$..
|
||||||
|
\item Proposer une forme factorisée de $N(x)$.
|
||||||
|
\item Proposer une forme factorisée de $f'(x)$.
|
||||||
|
\end{enumerate}
|
||||||
|
\item Étudier le signe de $f'$ et en déduire les variations de $f$.
|
||||||
|
\end{enumerate}
|
||||||
|
\end{exercise}
|
||||||
|
|
||||||
|
\begin{solution}
|
||||||
|
\begin{enumerate}
|
||||||
|
\item pas de correction disponible
|
||||||
|
\item
|
||||||
|
\begin{enumerate}
|
||||||
|
\item \[N(- 4) = 0\]
|
||||||
|
\[N(7) = 0\]
|
||||||
|
\item \[
|
||||||
|
N(x) = 7(x - - 4)(x - 7)
|
||||||
|
\]
|
||||||
|
\item
|
||||||
|
\[
|
||||||
|
f'(x) = \frac{7(x - - 4)(x - 7)}{x}
|
||||||
|
\]
|
||||||
|
\end{enumerate}
|
||||||
|
\item Pas de correction disponible
|
||||||
|
\end{enumerate}
|
||||||
|
\end{solution}
|
||||||
|
|
||||||
|
\begin{exercise}[subtitle={Complexes}]
|
||||||
|
\begin{enumerate}
|
||||||
|
\item Mettre le nombre complexe suivant sous forme algébrique $z_1 = \dfrac{8 + 9 i}{-9 + 5 i} $
|
||||||
|
\item Mettre le complexe suivante sous forme exponentielle $z_2 = - 8 \sqrt{3} + 8 i$
|
||||||
|
\item Mettre le complexe suivante sous forme exponentielle $z_3 = -5 - 5 \sqrt{3} i$
|
||||||
|
\item Calculer le produit $z_4=z_2\times z_3$ donner le résultat sous forme exponentielle puis algébrique.
|
||||||
|
\item Calculer le quotient $z_5=\frac{z_2}{z_3}$ donner le résultat sous forme exponentielle puis algébrique.
|
||||||
|
\end{enumerate}
|
||||||
|
\end{exercise}
|
||||||
|
|
||||||
|
\begin{solution}
|
||||||
|
\begin{enumerate}
|
||||||
|
\item $z_1 = - \frac{27}{106} - \frac{121 i}{106}$
|
||||||
|
\item $z_2 = 16 e^{\frac{5 i \pi}{6}}$
|
||||||
|
\item $z_3 = 10 e^{- \frac{2 i \pi}{3}}$
|
||||||
|
\item $z_4 = 160 e^{\frac{i \pi}{6}} = 80 \sqrt{3} + 80 i = 139.0 + 80.0 i$
|
||||||
|
\item $z_5 = \frac{8}{5} e^{\frac{3 i \pi}{2}} = - \frac{8 i}{5} = - 1.6 i$
|
||||||
|
\end{enumerate}
|
||||||
|
\end{solution}
|
||||||
|
|
||||||
|
\begin{exercise}[subtitle={Sortie du congélateur}]
|
||||||
|
Marie a invité quelques amis pour le thé. Elle souhaite leur proposer ses macarons maison.
|
||||||
|
|
||||||
|
Elle les sort de son congélateur à $-15$~\degres C et les place dans une pièce à $22$~\degres C.
|
||||||
|
|
||||||
|
Au bout de 15 minutes, la température des macarons est de $-1$~\degres C.
|
||||||
|
|
||||||
|
\bigskip
|
||||||
|
|
||||||
|
\textbf{Premier modèle}
|
||||||
|
|
||||||
|
\medskip
|
||||||
|
|
||||||
|
On suppose que la vitesse de décongélation est constante : chaque minute la hausse de
|
||||||
|
température des macarons est la même.
|
||||||
|
|
||||||
|
Estimer dans ce cadre la température au bout de $30$~minutes, puis au bout de $45$~minutes.
|
||||||
|
|
||||||
|
Cette modélisation est-elle pertinente?
|
||||||
|
|
||||||
|
\bigskip
|
||||||
|
|
||||||
|
\textbf{Deuxième modèle}
|
||||||
|
|
||||||
|
\medskip
|
||||||
|
|
||||||
|
On suppose maintenant que la vitesse de décongélation est proportionnelle à la différence
|
||||||
|
de température entre les macarons et l'air ambiant (il s'agit de la loi de Newton).
|
||||||
|
|
||||||
|
On désigne par $\theta$ la température des macarons à l'instant $t$, et par $\theta'$ la vitesse de décongélation.
|
||||||
|
|
||||||
|
L'unité de temps est la minute et l'unité de température le degré Celsius.
|
||||||
|
|
||||||
|
\smallskip
|
||||||
|
|
||||||
|
On négligera la diminution de température de la pièce et on admettra donc qu'il existe un
|
||||||
|
nombre réel $a$ tel que, pour $t$ positif :
|
||||||
|
|
||||||
|
\[\theta'(t) = a [\theta(t) - 22]\quad (E)\]
|
||||||
|
|
||||||
|
\medskip
|
||||||
|
|
||||||
|
\begin{enumerate}
|
||||||
|
\item Vérifier que l'équation $(E)$ a pour solutions $\theta(t) = K e^{at} + 22$ où $K$ est un nombre réel.
|
||||||
|
|
||||||
|
Donner alors, en fonction de $a$, l'ensemble des solutions de $(E)$.
|
||||||
|
\end{enumerate}
|
||||||
|
On rappelle que la température des macarons à l'instant $t = 0$ est égale à $-15$~\degres C et que, au bout de $15$~min, elle est de $-1$~\degres C.
|
||||||
|
\begin{enumerate}
|
||||||
|
\setcounter{enumi}{1}
|
||||||
|
\item En utilisant la condition à $t=0$ démontrer que $K = -37$.
|
||||||
|
\item En utilisant la condition à $t=15$ démontrer que $a \approx -0.03$.
|
||||||
|
\item En déduire l'expression de la solution de l'équation différentielle puis étudier ses variations.
|
||||||
|
\item La température idéale de dégustation des macarons étant de $19$~\degres C, Marie estime que
|
||||||
|
celle-ci sera atteinte au bout de $30$~min. A-t-elle raison ? Justifier la réponse.
|
||||||
|
|
||||||
|
Sinon, combien de temps faudra-t-il attendre ?
|
||||||
|
\end{enumerate}
|
||||||
|
\end{exercise}
|
||||||
|
|
||||||
|
|
||||||
|
|
||||||
|
\end{document}
|
||||||
|
|
||||||
|
%%% Local Variables:
|
||||||
|
%%% mode: latex
|
||||||
|
%%% TeX-master: "master"
|
||||||
|
%%% End:
|
|
@ -0,0 +1,136 @@
|
||||||
|
\documentclass[a4paper,10pt]{article}
|
||||||
|
\usepackage{myXsim}
|
||||||
|
|
||||||
|
% Title Page
|
||||||
|
\title{DS8 \hfill BUDIN Nathan}
|
||||||
|
\tribe{TST sti2d}
|
||||||
|
\date{\hfillÀ render pour le vendredi 9 avril à 10h au plus tard}
|
||||||
|
|
||||||
|
\xsimsetup{
|
||||||
|
solution/print = false
|
||||||
|
}
|
||||||
|
|
||||||
|
\begin{document}
|
||||||
|
\maketitle
|
||||||
|
|
||||||
|
\begin{exercise}[subtitle={Étude de fonction}]
|
||||||
|
On considère la fonction $f$ définie sur $\intOF{0}{+\infty}$ par $ f(x) = 3.5x^2 + - 84x + 140\ln(x)$
|
||||||
|
\begin{enumerate}
|
||||||
|
\item Démontrer que la dérivée de $f$ est $f'(x) = \frac{7x^2 + - 84x + 140}{x}$.
|
||||||
|
\item Étude du numérateur de $f'(x)$: $N(x) = 7x^2 - 84x + 140$
|
||||||
|
\begin{enumerate}
|
||||||
|
\item Démontrer que $x=10$ et $x=2$ sont deux racines de $N(x)$..
|
||||||
|
\item Proposer une forme factorisée de $N(x)$.
|
||||||
|
\item Proposer une forme factorisée de $f'(x)$.
|
||||||
|
\end{enumerate}
|
||||||
|
\item Étudier le signe de $f'$ et en déduire les variations de $f$.
|
||||||
|
\end{enumerate}
|
||||||
|
\end{exercise}
|
||||||
|
|
||||||
|
\begin{solution}
|
||||||
|
\begin{enumerate}
|
||||||
|
\item pas de correction disponible
|
||||||
|
\item
|
||||||
|
\begin{enumerate}
|
||||||
|
\item \[N(10) = 0\]
|
||||||
|
\[N(2) = 0\]
|
||||||
|
\item \[
|
||||||
|
N(x) = 7(x - 10)(x - 2)
|
||||||
|
\]
|
||||||
|
\item
|
||||||
|
\[
|
||||||
|
f'(x) = \frac{7(x - 10)(x - 2)}{x}
|
||||||
|
\]
|
||||||
|
\end{enumerate}
|
||||||
|
\item Pas de correction disponible
|
||||||
|
\end{enumerate}
|
||||||
|
\end{solution}
|
||||||
|
|
||||||
|
\begin{exercise}[subtitle={Complexes}]
|
||||||
|
\begin{enumerate}
|
||||||
|
\item Mettre le nombre complexe suivant sous forme algébrique $z_1 = \dfrac{8 + 8 i}{-7 + 4 i} $
|
||||||
|
\item Mettre le complexe suivante sous forme exponentielle $z_2 = 7 \sqrt{3} - 7 i$
|
||||||
|
\item Mettre le complexe suivante sous forme exponentielle $z_3 = - 2 \sqrt{3} + 2 i$
|
||||||
|
\item Calculer le produit $z_4=z_2\times z_3$ donner le résultat sous forme exponentielle puis algébrique.
|
||||||
|
\item Calculer le quotient $z_5=\frac{z_2}{z_3}$ donner le résultat sous forme exponentielle puis algébrique.
|
||||||
|
\end{enumerate}
|
||||||
|
\end{exercise}
|
||||||
|
|
||||||
|
\begin{solution}
|
||||||
|
\begin{enumerate}
|
||||||
|
\item $z_1 = - \frac{24}{65} - \frac{88 i}{65}$
|
||||||
|
\item $z_2 = 14 e^{- \frac{i \pi}{6}}$
|
||||||
|
\item $z_3 = 4 e^{\frac{5 i \pi}{6}}$
|
||||||
|
\item $z_4 = 56 e^{\frac{2 i \pi}{3}} = -28 + 28 \sqrt{3} i = -28.0 + 48.5 i$
|
||||||
|
\item $z_5 = \frac{7}{2} e^{- i \pi} = - \frac{7}{2} = -3.5$
|
||||||
|
\end{enumerate}
|
||||||
|
\end{solution}
|
||||||
|
|
||||||
|
\begin{exercise}[subtitle={Sortie du congélateur}]
|
||||||
|
Marie a invité quelques amis pour le thé. Elle souhaite leur proposer ses macarons maison.
|
||||||
|
|
||||||
|
Elle les sort de son congélateur à $-19$~\degres C et les place dans une pièce à $23$~\degres C.
|
||||||
|
|
||||||
|
Au bout de 15 minutes, la température des macarons est de $3$~\degres C.
|
||||||
|
|
||||||
|
\bigskip
|
||||||
|
|
||||||
|
\textbf{Premier modèle}
|
||||||
|
|
||||||
|
\medskip
|
||||||
|
|
||||||
|
On suppose que la vitesse de décongélation est constante : chaque minute la hausse de
|
||||||
|
température des macarons est la même.
|
||||||
|
|
||||||
|
Estimer dans ce cadre la température au bout de $30$~minutes, puis au bout de $45$~minutes.
|
||||||
|
|
||||||
|
Cette modélisation est-elle pertinente?
|
||||||
|
|
||||||
|
\bigskip
|
||||||
|
|
||||||
|
\textbf{Deuxième modèle}
|
||||||
|
|
||||||
|
\medskip
|
||||||
|
|
||||||
|
On suppose maintenant que la vitesse de décongélation est proportionnelle à la différence
|
||||||
|
de température entre les macarons et l'air ambiant (il s'agit de la loi de Newton).
|
||||||
|
|
||||||
|
On désigne par $\theta$ la température des macarons à l'instant $t$, et par $\theta'$ la vitesse de décongélation.
|
||||||
|
|
||||||
|
L'unité de temps est la minute et l'unité de température le degré Celsius.
|
||||||
|
|
||||||
|
\smallskip
|
||||||
|
|
||||||
|
On négligera la diminution de température de la pièce et on admettra donc qu'il existe un
|
||||||
|
nombre réel $a$ tel que, pour $t$ positif :
|
||||||
|
|
||||||
|
\[\theta'(t) = a [\theta(t) - 23]\quad (E)\]
|
||||||
|
|
||||||
|
\medskip
|
||||||
|
|
||||||
|
\begin{enumerate}
|
||||||
|
\item Vérifier que l'équation $(E)$ a pour solutions $\theta(t) = K e^{at} + 23$ où $K$ est un nombre réel.
|
||||||
|
|
||||||
|
Donner alors, en fonction de $a$, l'ensemble des solutions de $(E)$.
|
||||||
|
\end{enumerate}
|
||||||
|
On rappelle que la température des macarons à l'instant $t = 0$ est égale à $-19$~\degres C et que, au bout de $15$~min, elle est de $3$~\degres C.
|
||||||
|
\begin{enumerate}
|
||||||
|
\setcounter{enumi}{1}
|
||||||
|
\item En utilisant la condition à $t=0$ démontrer que $K = -42$.
|
||||||
|
\item En utilisant la condition à $t=15$ démontrer que $a \approx -0.05$.
|
||||||
|
\item En déduire l'expression de la solution de l'équation différentielle puis étudier ses variations.
|
||||||
|
\item La température idéale de dégustation des macarons étant de $20$~\degres C, Marie estime que
|
||||||
|
celle-ci sera atteinte au bout de $30$~min. A-t-elle raison ? Justifier la réponse.
|
||||||
|
|
||||||
|
Sinon, combien de temps faudra-t-il attendre ?
|
||||||
|
\end{enumerate}
|
||||||
|
\end{exercise}
|
||||||
|
|
||||||
|
|
||||||
|
|
||||||
|
\end{document}
|
||||||
|
|
||||||
|
%%% Local Variables:
|
||||||
|
%%% mode: latex
|
||||||
|
%%% TeX-master: "master"
|
||||||
|
%%% End:
|
|
@ -0,0 +1,136 @@
|
||||||
|
\documentclass[a4paper,10pt]{article}
|
||||||
|
\usepackage{myXsim}
|
||||||
|
|
||||||
|
% Title Page
|
||||||
|
\title{DS8 \hfill CHION Léa}
|
||||||
|
\tribe{TST sti2d}
|
||||||
|
\date{\hfillÀ render pour le vendredi 9 avril à 10h au plus tard}
|
||||||
|
|
||||||
|
\xsimsetup{
|
||||||
|
solution/print = false
|
||||||
|
}
|
||||||
|
|
||||||
|
\begin{document}
|
||||||
|
\maketitle
|
||||||
|
|
||||||
|
\begin{exercise}[subtitle={Étude de fonction}]
|
||||||
|
On considère la fonction $f$ définie sur $\intOF{0}{+\infty}$ par $ f(x) = 5x^2 + 130x + 300\ln(x)$
|
||||||
|
\begin{enumerate}
|
||||||
|
\item Démontrer que la dérivée de $f$ est $f'(x) = \frac{10x^2 + 130x + 300}{x}$.
|
||||||
|
\item Étude du numérateur de $f'(x)$: $N(x) = 10x^2 + 130x + 300$
|
||||||
|
\begin{enumerate}
|
||||||
|
\item Démontrer que $x=- 10$ et $x=- 3$ sont deux racines de $N(x)$..
|
||||||
|
\item Proposer une forme factorisée de $N(x)$.
|
||||||
|
\item Proposer une forme factorisée de $f'(x)$.
|
||||||
|
\end{enumerate}
|
||||||
|
\item Étudier le signe de $f'$ et en déduire les variations de $f$.
|
||||||
|
\end{enumerate}
|
||||||
|
\end{exercise}
|
||||||
|
|
||||||
|
\begin{solution}
|
||||||
|
\begin{enumerate}
|
||||||
|
\item pas de correction disponible
|
||||||
|
\item
|
||||||
|
\begin{enumerate}
|
||||||
|
\item \[N(- 10) = 0\]
|
||||||
|
\[N(- 3) = 0\]
|
||||||
|
\item \[
|
||||||
|
N(x) = 10(x - - 10)(x - - 3)
|
||||||
|
\]
|
||||||
|
\item
|
||||||
|
\[
|
||||||
|
f'(x) = \frac{10(x - - 10)(x - - 3)}{x}
|
||||||
|
\]
|
||||||
|
\end{enumerate}
|
||||||
|
\item Pas de correction disponible
|
||||||
|
\end{enumerate}
|
||||||
|
\end{solution}
|
||||||
|
|
||||||
|
\begin{exercise}[subtitle={Complexes}]
|
||||||
|
\begin{enumerate}
|
||||||
|
\item Mettre le nombre complexe suivant sous forme algébrique $z_1 = \dfrac{8 + 10 i}{-3 + 2 i} $
|
||||||
|
\item Mettre le complexe suivante sous forme exponentielle $z_2 = - \sqrt{2} - \sqrt{2} i$
|
||||||
|
\item Mettre le complexe suivante sous forme exponentielle $z_3 = - 10 \sqrt{2} + 10 \sqrt{2} i$
|
||||||
|
\item Calculer le produit $z_4=z_2\times z_3$ donner le résultat sous forme exponentielle puis algébrique.
|
||||||
|
\item Calculer le quotient $z_5=\frac{z_2}{z_3}$ donner le résultat sous forme exponentielle puis algébrique.
|
||||||
|
\end{enumerate}
|
||||||
|
\end{exercise}
|
||||||
|
|
||||||
|
\begin{solution}
|
||||||
|
\begin{enumerate}
|
||||||
|
\item $z_1 = - \frac{4}{13} - \frac{46 i}{13}$
|
||||||
|
\item $z_2 = 2 e^{- \frac{3 i \pi}{4}}$
|
||||||
|
\item $z_3 = 20 e^{\frac{3 i \pi}{4}}$
|
||||||
|
\item $z_4 = 40 e^{0} = 40 = 40.0$
|
||||||
|
\item $z_5 = \frac{1}{10} e^{- \frac{3 i \pi}{2}} = \frac{i}{10} = 0.1 i$
|
||||||
|
\end{enumerate}
|
||||||
|
\end{solution}
|
||||||
|
|
||||||
|
\begin{exercise}[subtitle={Sortie du congélateur}]
|
||||||
|
Marie a invité quelques amis pour le thé. Elle souhaite leur proposer ses macarons maison.
|
||||||
|
|
||||||
|
Elle les sort de son congélateur à $-20$~\degres C et les place dans une pièce à $24$~\degres C.
|
||||||
|
|
||||||
|
Au bout de 15 minutes, la température des macarons est de $-3$~\degres C.
|
||||||
|
|
||||||
|
\bigskip
|
||||||
|
|
||||||
|
\textbf{Premier modèle}
|
||||||
|
|
||||||
|
\medskip
|
||||||
|
|
||||||
|
On suppose que la vitesse de décongélation est constante : chaque minute la hausse de
|
||||||
|
température des macarons est la même.
|
||||||
|
|
||||||
|
Estimer dans ce cadre la température au bout de $30$~minutes, puis au bout de $45$~minutes.
|
||||||
|
|
||||||
|
Cette modélisation est-elle pertinente?
|
||||||
|
|
||||||
|
\bigskip
|
||||||
|
|
||||||
|
\textbf{Deuxième modèle}
|
||||||
|
|
||||||
|
\medskip
|
||||||
|
|
||||||
|
On suppose maintenant que la vitesse de décongélation est proportionnelle à la différence
|
||||||
|
de température entre les macarons et l'air ambiant (il s'agit de la loi de Newton).
|
||||||
|
|
||||||
|
On désigne par $\theta$ la température des macarons à l'instant $t$, et par $\theta'$ la vitesse de décongélation.
|
||||||
|
|
||||||
|
L'unité de temps est la minute et l'unité de température le degré Celsius.
|
||||||
|
|
||||||
|
\smallskip
|
||||||
|
|
||||||
|
On négligera la diminution de température de la pièce et on admettra donc qu'il existe un
|
||||||
|
nombre réel $a$ tel que, pour $t$ positif :
|
||||||
|
|
||||||
|
\[\theta'(t) = a [\theta(t) - 24]\quad (E)\]
|
||||||
|
|
||||||
|
\medskip
|
||||||
|
|
||||||
|
\begin{enumerate}
|
||||||
|
\item Vérifier que l'équation $(E)$ a pour solutions $\theta(t) = K e^{at} + 24$ où $K$ est un nombre réel.
|
||||||
|
|
||||||
|
Donner alors, en fonction de $a$, l'ensemble des solutions de $(E)$.
|
||||||
|
\end{enumerate}
|
||||||
|
On rappelle que la température des macarons à l'instant $t = 0$ est égale à $-20$~\degres C et que, au bout de $15$~min, elle est de $-3$~\degres C.
|
||||||
|
\begin{enumerate}
|
||||||
|
\setcounter{enumi}{1}
|
||||||
|
\item En utilisant la condition à $t=0$ démontrer que $K = -44$.
|
||||||
|
\item En utilisant la condition à $t=15$ démontrer que $a \approx -0.03$.
|
||||||
|
\item En déduire l'expression de la solution de l'équation différentielle puis étudier ses variations.
|
||||||
|
\item La température idéale de dégustation des macarons étant de $21$~\degres C, Marie estime que
|
||||||
|
celle-ci sera atteinte au bout de $30$~min. A-t-elle raison ? Justifier la réponse.
|
||||||
|
|
||||||
|
Sinon, combien de temps faudra-t-il attendre ?
|
||||||
|
\end{enumerate}
|
||||||
|
\end{exercise}
|
||||||
|
|
||||||
|
|
||||||
|
|
||||||
|
\end{document}
|
||||||
|
|
||||||
|
%%% Local Variables:
|
||||||
|
%%% mode: latex
|
||||||
|
%%% TeX-master: "master"
|
||||||
|
%%% End:
|
|
@ -0,0 +1,136 @@
|
||||||
|
\documentclass[a4paper,10pt]{article}
|
||||||
|
\usepackage{myXsim}
|
||||||
|
|
||||||
|
% Title Page
|
||||||
|
\title{DS8 \hfill CLAIN Avinash}
|
||||||
|
\tribe{TST sti2d}
|
||||||
|
\date{\hfillÀ render pour le vendredi 9 avril à 10h au plus tard}
|
||||||
|
|
||||||
|
\xsimsetup{
|
||||||
|
solution/print = false
|
||||||
|
}
|
||||||
|
|
||||||
|
\begin{document}
|
||||||
|
\maketitle
|
||||||
|
|
||||||
|
\begin{exercise}[subtitle={Étude de fonction}]
|
||||||
|
On considère la fonction $f$ définie sur $\intOF{0}{+\infty}$ par $ f(x) = 3.5x^2 + - 42x + - 280\ln(x)$
|
||||||
|
\begin{enumerate}
|
||||||
|
\item Démontrer que la dérivée de $f$ est $f'(x) = \frac{7x^2 + - 42x + - 280}{x}$.
|
||||||
|
\item Étude du numérateur de $f'(x)$: $N(x) = 7x^2 - 42x - 280$
|
||||||
|
\begin{enumerate}
|
||||||
|
\item Démontrer que $x=10$ et $x=- 4$ sont deux racines de $N(x)$..
|
||||||
|
\item Proposer une forme factorisée de $N(x)$.
|
||||||
|
\item Proposer une forme factorisée de $f'(x)$.
|
||||||
|
\end{enumerate}
|
||||||
|
\item Étudier le signe de $f'$ et en déduire les variations de $f$.
|
||||||
|
\end{enumerate}
|
||||||
|
\end{exercise}
|
||||||
|
|
||||||
|
\begin{solution}
|
||||||
|
\begin{enumerate}
|
||||||
|
\item pas de correction disponible
|
||||||
|
\item
|
||||||
|
\begin{enumerate}
|
||||||
|
\item \[N(10) = 0\]
|
||||||
|
\[N(- 4) = 0\]
|
||||||
|
\item \[
|
||||||
|
N(x) = 7(x - 10)(x - - 4)
|
||||||
|
\]
|
||||||
|
\item
|
||||||
|
\[
|
||||||
|
f'(x) = \frac{7(x - 10)(x - - 4)}{x}
|
||||||
|
\]
|
||||||
|
\end{enumerate}
|
||||||
|
\item Pas de correction disponible
|
||||||
|
\end{enumerate}
|
||||||
|
\end{solution}
|
||||||
|
|
||||||
|
\begin{exercise}[subtitle={Complexes}]
|
||||||
|
\begin{enumerate}
|
||||||
|
\item Mettre le nombre complexe suivant sous forme algébrique $z_1 = \dfrac{3 + 8 i}{-10 + 5 i} $
|
||||||
|
\item Mettre le complexe suivante sous forme exponentielle $z_2 = 1 + \sqrt{3} i$
|
||||||
|
\item Mettre le complexe suivante sous forme exponentielle $z_3 = -3 + 3 \sqrt{3} i$
|
||||||
|
\item Calculer le produit $z_4=z_2\times z_3$ donner le résultat sous forme exponentielle puis algébrique.
|
||||||
|
\item Calculer le quotient $z_5=\frac{z_2}{z_3}$ donner le résultat sous forme exponentielle puis algébrique.
|
||||||
|
\end{enumerate}
|
||||||
|
\end{exercise}
|
||||||
|
|
||||||
|
\begin{solution}
|
||||||
|
\begin{enumerate}
|
||||||
|
\item $z_1 = \frac{2}{25} - \frac{19 i}{25}$
|
||||||
|
\item $z_2 = 2 e^{\frac{i \pi}{3}}$
|
||||||
|
\item $z_3 = 6 e^{\frac{2 i \pi}{3}}$
|
||||||
|
\item $z_4 = 12 e^{i \pi} = -12 = -12.0$
|
||||||
|
\item $z_5 = \frac{1}{3} e^{- \frac{i \pi}{3}} = \frac{1}{6} - \frac{\sqrt{3} i}{6} = 0.167 - 0.289 i$
|
||||||
|
\end{enumerate}
|
||||||
|
\end{solution}
|
||||||
|
|
||||||
|
\begin{exercise}[subtitle={Sortie du congélateur}]
|
||||||
|
Marie a invité quelques amis pour le thé. Elle souhaite leur proposer ses macarons maison.
|
||||||
|
|
||||||
|
Elle les sort de son congélateur à $-20$~\degres C et les place dans une pièce à $17$~\degres C.
|
||||||
|
|
||||||
|
Au bout de 15 minutes, la température des macarons est de $4$~\degres C.
|
||||||
|
|
||||||
|
\bigskip
|
||||||
|
|
||||||
|
\textbf{Premier modèle}
|
||||||
|
|
||||||
|
\medskip
|
||||||
|
|
||||||
|
On suppose que la vitesse de décongélation est constante : chaque minute la hausse de
|
||||||
|
température des macarons est la même.
|
||||||
|
|
||||||
|
Estimer dans ce cadre la température au bout de $30$~minutes, puis au bout de $45$~minutes.
|
||||||
|
|
||||||
|
Cette modélisation est-elle pertinente?
|
||||||
|
|
||||||
|
\bigskip
|
||||||
|
|
||||||
|
\textbf{Deuxième modèle}
|
||||||
|
|
||||||
|
\medskip
|
||||||
|
|
||||||
|
On suppose maintenant que la vitesse de décongélation est proportionnelle à la différence
|
||||||
|
de température entre les macarons et l'air ambiant (il s'agit de la loi de Newton).
|
||||||
|
|
||||||
|
On désigne par $\theta$ la température des macarons à l'instant $t$, et par $\theta'$ la vitesse de décongélation.
|
||||||
|
|
||||||
|
L'unité de temps est la minute et l'unité de température le degré Celsius.
|
||||||
|
|
||||||
|
\smallskip
|
||||||
|
|
||||||
|
On négligera la diminution de température de la pièce et on admettra donc qu'il existe un
|
||||||
|
nombre réel $a$ tel que, pour $t$ positif :
|
||||||
|
|
||||||
|
\[\theta'(t) = a [\theta(t) - 17]\quad (E)\]
|
||||||
|
|
||||||
|
\medskip
|
||||||
|
|
||||||
|
\begin{enumerate}
|
||||||
|
\item Vérifier que l'équation $(E)$ a pour solutions $\theta(t) = K e^{at} + 17$ où $K$ est un nombre réel.
|
||||||
|
|
||||||
|
Donner alors, en fonction de $a$, l'ensemble des solutions de $(E)$.
|
||||||
|
\end{enumerate}
|
||||||
|
On rappelle que la température des macarons à l'instant $t = 0$ est égale à $-20$~\degres C et que, au bout de $15$~min, elle est de $4$~\degres C.
|
||||||
|
\begin{enumerate}
|
||||||
|
\setcounter{enumi}{1}
|
||||||
|
\item En utilisant la condition à $t=0$ démontrer que $K = -37$.
|
||||||
|
\item En utilisant la condition à $t=15$ démontrer que $a \approx -0.07$.
|
||||||
|
\item En déduire l'expression de la solution de l'équation différentielle puis étudier ses variations.
|
||||||
|
\item La température idéale de dégustation des macarons étant de $14$~\degres C, Marie estime que
|
||||||
|
celle-ci sera atteinte au bout de $30$~min. A-t-elle raison ? Justifier la réponse.
|
||||||
|
|
||||||
|
Sinon, combien de temps faudra-t-il attendre ?
|
||||||
|
\end{enumerate}
|
||||||
|
\end{exercise}
|
||||||
|
|
||||||
|
|
||||||
|
|
||||||
|
\end{document}
|
||||||
|
|
||||||
|
%%% Local Variables:
|
||||||
|
%%% mode: latex
|
||||||
|
%%% TeX-master: "master"
|
||||||
|
%%% End:
|
|
@ -0,0 +1,136 @@
|
||||||
|
\documentclass[a4paper,10pt]{article}
|
||||||
|
\usepackage{myXsim}
|
||||||
|
|
||||||
|
% Title Page
|
||||||
|
\title{DS8 \hfill COUBAT Alexis}
|
||||||
|
\tribe{TST sti2d}
|
||||||
|
\date{\hfillÀ render pour le vendredi 9 avril à 10h au plus tard}
|
||||||
|
|
||||||
|
\xsimsetup{
|
||||||
|
solution/print = false
|
||||||
|
}
|
||||||
|
|
||||||
|
\begin{document}
|
||||||
|
\maketitle
|
||||||
|
|
||||||
|
\begin{exercise}[subtitle={Étude de fonction}]
|
||||||
|
On considère la fonction $f$ définie sur $\intOF{0}{+\infty}$ par $ f(x) = 5x^2 + 70x + 60\ln(x)$
|
||||||
|
\begin{enumerate}
|
||||||
|
\item Démontrer que la dérivée de $f$ est $f'(x) = \frac{10x^2 + 70x + 60}{x}$.
|
||||||
|
\item Étude du numérateur de $f'(x)$: $N(x) = 10x^2 + 70x + 60$
|
||||||
|
\begin{enumerate}
|
||||||
|
\item Démontrer que $x=- 6$ et $x=- 1$ sont deux racines de $N(x)$..
|
||||||
|
\item Proposer une forme factorisée de $N(x)$.
|
||||||
|
\item Proposer une forme factorisée de $f'(x)$.
|
||||||
|
\end{enumerate}
|
||||||
|
\item Étudier le signe de $f'$ et en déduire les variations de $f$.
|
||||||
|
\end{enumerate}
|
||||||
|
\end{exercise}
|
||||||
|
|
||||||
|
\begin{solution}
|
||||||
|
\begin{enumerate}
|
||||||
|
\item pas de correction disponible
|
||||||
|
\item
|
||||||
|
\begin{enumerate}
|
||||||
|
\item \[N(- 6) = 0\]
|
||||||
|
\[N(- 1) = 0\]
|
||||||
|
\item \[
|
||||||
|
N(x) = 10(x - - 6)(x - - 1)
|
||||||
|
\]
|
||||||
|
\item
|
||||||
|
\[
|
||||||
|
f'(x) = \frac{10(x - - 6)(x - - 1)}{x}
|
||||||
|
\]
|
||||||
|
\end{enumerate}
|
||||||
|
\item Pas de correction disponible
|
||||||
|
\end{enumerate}
|
||||||
|
\end{solution}
|
||||||
|
|
||||||
|
\begin{exercise}[subtitle={Complexes}]
|
||||||
|
\begin{enumerate}
|
||||||
|
\item Mettre le nombre complexe suivant sous forme algébrique $z_1 = \dfrac{3 + 5 i}{-4 + 5 i} $
|
||||||
|
\item Mettre le complexe suivante sous forme exponentielle $z_2 = 5 + 5 \sqrt{3} i$
|
||||||
|
\item Mettre le complexe suivante sous forme exponentielle $z_3 = - 4 \sqrt{3} - 4 i$
|
||||||
|
\item Calculer le produit $z_4=z_2\times z_3$ donner le résultat sous forme exponentielle puis algébrique.
|
||||||
|
\item Calculer le quotient $z_5=\frac{z_2}{z_3}$ donner le résultat sous forme exponentielle puis algébrique.
|
||||||
|
\end{enumerate}
|
||||||
|
\end{exercise}
|
||||||
|
|
||||||
|
\begin{solution}
|
||||||
|
\begin{enumerate}
|
||||||
|
\item $z_1 = \frac{13}{41} - \frac{35 i}{41}$
|
||||||
|
\item $z_2 = 10 e^{\frac{i \pi}{3}}$
|
||||||
|
\item $z_3 = 8 e^{- \frac{5 i \pi}{6}}$
|
||||||
|
\item $z_4 = 80 e^{- \frac{i \pi}{2}} = - 80 i = - 80.0 i$
|
||||||
|
\item $z_5 = \frac{5}{4} e^{\frac{7 i \pi}{6}} = - \frac{5 \sqrt{3}}{8} - \frac{5 i}{8} = -1.08 - 0.625 i$
|
||||||
|
\end{enumerate}
|
||||||
|
\end{solution}
|
||||||
|
|
||||||
|
\begin{exercise}[subtitle={Sortie du congélateur}]
|
||||||
|
Marie a invité quelques amis pour le thé. Elle souhaite leur proposer ses macarons maison.
|
||||||
|
|
||||||
|
Elle les sort de son congélateur à $-20$~\degres C et les place dans une pièce à $21$~\degres C.
|
||||||
|
|
||||||
|
Au bout de 15 minutes, la température des macarons est de $4$~\degres C.
|
||||||
|
|
||||||
|
\bigskip
|
||||||
|
|
||||||
|
\textbf{Premier modèle}
|
||||||
|
|
||||||
|
\medskip
|
||||||
|
|
||||||
|
On suppose que la vitesse de décongélation est constante : chaque minute la hausse de
|
||||||
|
température des macarons est la même.
|
||||||
|
|
||||||
|
Estimer dans ce cadre la température au bout de $30$~minutes, puis au bout de $45$~minutes.
|
||||||
|
|
||||||
|
Cette modélisation est-elle pertinente?
|
||||||
|
|
||||||
|
\bigskip
|
||||||
|
|
||||||
|
\textbf{Deuxième modèle}
|
||||||
|
|
||||||
|
\medskip
|
||||||
|
|
||||||
|
On suppose maintenant que la vitesse de décongélation est proportionnelle à la différence
|
||||||
|
de température entre les macarons et l'air ambiant (il s'agit de la loi de Newton).
|
||||||
|
|
||||||
|
On désigne par $\theta$ la température des macarons à l'instant $t$, et par $\theta'$ la vitesse de décongélation.
|
||||||
|
|
||||||
|
L'unité de temps est la minute et l'unité de température le degré Celsius.
|
||||||
|
|
||||||
|
\smallskip
|
||||||
|
|
||||||
|
On négligera la diminution de température de la pièce et on admettra donc qu'il existe un
|
||||||
|
nombre réel $a$ tel que, pour $t$ positif :
|
||||||
|
|
||||||
|
\[\theta'(t) = a [\theta(t) - 21]\quad (E)\]
|
||||||
|
|
||||||
|
\medskip
|
||||||
|
|
||||||
|
\begin{enumerate}
|
||||||
|
\item Vérifier que l'équation $(E)$ a pour solutions $\theta(t) = K e^{at} + 21$ où $K$ est un nombre réel.
|
||||||
|
|
||||||
|
Donner alors, en fonction de $a$, l'ensemble des solutions de $(E)$.
|
||||||
|
\end{enumerate}
|
||||||
|
On rappelle que la température des macarons à l'instant $t = 0$ est égale à $-20$~\degres C et que, au bout de $15$~min, elle est de $4$~\degres C.
|
||||||
|
\begin{enumerate}
|
||||||
|
\setcounter{enumi}{1}
|
||||||
|
\item En utilisant la condition à $t=0$ démontrer que $K = -41$.
|
||||||
|
\item En utilisant la condition à $t=15$ démontrer que $a \approx -0.06$.
|
||||||
|
\item En déduire l'expression de la solution de l'équation différentielle puis étudier ses variations.
|
||||||
|
\item La température idéale de dégustation des macarons étant de $18$~\degres C, Marie estime que
|
||||||
|
celle-ci sera atteinte au bout de $30$~min. A-t-elle raison ? Justifier la réponse.
|
||||||
|
|
||||||
|
Sinon, combien de temps faudra-t-il attendre ?
|
||||||
|
\end{enumerate}
|
||||||
|
\end{exercise}
|
||||||
|
|
||||||
|
|
||||||
|
|
||||||
|
\end{document}
|
||||||
|
|
||||||
|
%%% Local Variables:
|
||||||
|
%%% mode: latex
|
||||||
|
%%% TeX-master: "master"
|
||||||
|
%%% End:
|
|
@ -0,0 +1,136 @@
|
||||||
|
\documentclass[a4paper,10pt]{article}
|
||||||
|
\usepackage{myXsim}
|
||||||
|
|
||||||
|
% Title Page
|
||||||
|
\title{DS8 \hfill EVRARD Jules}
|
||||||
|
\tribe{TST sti2d}
|
||||||
|
\date{\hfillÀ render pour le vendredi 9 avril à 10h au plus tard}
|
||||||
|
|
||||||
|
\xsimsetup{
|
||||||
|
solution/print = false
|
||||||
|
}
|
||||||
|
|
||||||
|
\begin{document}
|
||||||
|
\maketitle
|
||||||
|
|
||||||
|
\begin{exercise}[subtitle={Étude de fonction}]
|
||||||
|
On considère la fonction $f$ définie sur $\intOF{0}{+\infty}$ par $ f(x) = 5x^2 + 0x + - 360\ln(x)$
|
||||||
|
\begin{enumerate}
|
||||||
|
\item Démontrer que la dérivée de $f$ est $f'(x) = \frac{10x^2 + 0x + - 360}{x}$.
|
||||||
|
\item Étude du numérateur de $f'(x)$: $N(x) = 10x^2 - 360$
|
||||||
|
\begin{enumerate}
|
||||||
|
\item Démontrer que $x=6$ et $x=- 6$ sont deux racines de $N(x)$..
|
||||||
|
\item Proposer une forme factorisée de $N(x)$.
|
||||||
|
\item Proposer une forme factorisée de $f'(x)$.
|
||||||
|
\end{enumerate}
|
||||||
|
\item Étudier le signe de $f'$ et en déduire les variations de $f$.
|
||||||
|
\end{enumerate}
|
||||||
|
\end{exercise}
|
||||||
|
|
||||||
|
\begin{solution}
|
||||||
|
\begin{enumerate}
|
||||||
|
\item pas de correction disponible
|
||||||
|
\item
|
||||||
|
\begin{enumerate}
|
||||||
|
\item \[N(6) = 0\]
|
||||||
|
\[N(- 6) = 0\]
|
||||||
|
\item \[
|
||||||
|
N(x) = 10(x - 6)(x - - 6)
|
||||||
|
\]
|
||||||
|
\item
|
||||||
|
\[
|
||||||
|
f'(x) = \frac{10(x - 6)(x - - 6)}{x}
|
||||||
|
\]
|
||||||
|
\end{enumerate}
|
||||||
|
\item Pas de correction disponible
|
||||||
|
\end{enumerate}
|
||||||
|
\end{solution}
|
||||||
|
|
||||||
|
\begin{exercise}[subtitle={Complexes}]
|
||||||
|
\begin{enumerate}
|
||||||
|
\item Mettre le nombre complexe suivant sous forme algébrique $z_1 = \dfrac{10 + 8 i}{-5 + 7 i} $
|
||||||
|
\item Mettre le complexe suivante sous forme exponentielle $z_2 = 9 \sqrt{2} - 9 \sqrt{2} i$
|
||||||
|
\item Mettre le complexe suivante sous forme exponentielle $z_3 = 7 \sqrt{3} + 7 i$
|
||||||
|
\item Calculer le produit $z_4=z_2\times z_3$ donner le résultat sous forme exponentielle puis algébrique.
|
||||||
|
\item Calculer le quotient $z_5=\frac{z_2}{z_3}$ donner le résultat sous forme exponentielle puis algébrique.
|
||||||
|
\end{enumerate}
|
||||||
|
\end{exercise}
|
||||||
|
|
||||||
|
\begin{solution}
|
||||||
|
\begin{enumerate}
|
||||||
|
\item $z_1 = \frac{3}{37} - \frac{55 i}{37}$
|
||||||
|
\item $z_2 = 18 e^{- \frac{i \pi}{4}}$
|
||||||
|
\item $z_3 = 14 e^{\frac{i \pi}{6}}$
|
||||||
|
\item $z_4 = 252 e^{- \frac{i \pi}{12}} = 63 \sqrt{2} + 63 \sqrt{6} + i \left(- 63 \sqrt{6} + 63 \sqrt{2}\right) = 243.0 - 65.2 i$
|
||||||
|
\item $z_5 = \frac{9}{7} e^{- \frac{5 i \pi}{12}} = - \frac{9 \sqrt{2}}{28} + \frac{9 \sqrt{6}}{28} + i \left(- \frac{9 \sqrt{6}}{28} - \frac{9 \sqrt{2}}{28}\right) = 0.333 - 1.24 i$
|
||||||
|
\end{enumerate}
|
||||||
|
\end{solution}
|
||||||
|
|
||||||
|
\begin{exercise}[subtitle={Sortie du congélateur}]
|
||||||
|
Marie a invité quelques amis pour le thé. Elle souhaite leur proposer ses macarons maison.
|
||||||
|
|
||||||
|
Elle les sort de son congélateur à $-15$~\degres C et les place dans une pièce à $17$~\degres C.
|
||||||
|
|
||||||
|
Au bout de 15 minutes, la température des macarons est de $-1$~\degres C.
|
||||||
|
|
||||||
|
\bigskip
|
||||||
|
|
||||||
|
\textbf{Premier modèle}
|
||||||
|
|
||||||
|
\medskip
|
||||||
|
|
||||||
|
On suppose que la vitesse de décongélation est constante : chaque minute la hausse de
|
||||||
|
température des macarons est la même.
|
||||||
|
|
||||||
|
Estimer dans ce cadre la température au bout de $30$~minutes, puis au bout de $45$~minutes.
|
||||||
|
|
||||||
|
Cette modélisation est-elle pertinente?
|
||||||
|
|
||||||
|
\bigskip
|
||||||
|
|
||||||
|
\textbf{Deuxième modèle}
|
||||||
|
|
||||||
|
\medskip
|
||||||
|
|
||||||
|
On suppose maintenant que la vitesse de décongélation est proportionnelle à la différence
|
||||||
|
de température entre les macarons et l'air ambiant (il s'agit de la loi de Newton).
|
||||||
|
|
||||||
|
On désigne par $\theta$ la température des macarons à l'instant $t$, et par $\theta'$ la vitesse de décongélation.
|
||||||
|
|
||||||
|
L'unité de temps est la minute et l'unité de température le degré Celsius.
|
||||||
|
|
||||||
|
\smallskip
|
||||||
|
|
||||||
|
On négligera la diminution de température de la pièce et on admettra donc qu'il existe un
|
||||||
|
nombre réel $a$ tel que, pour $t$ positif :
|
||||||
|
|
||||||
|
\[\theta'(t) = a [\theta(t) - 17]\quad (E)\]
|
||||||
|
|
||||||
|
\medskip
|
||||||
|
|
||||||
|
\begin{enumerate}
|
||||||
|
\item Vérifier que l'équation $(E)$ a pour solutions $\theta(t) = K e^{at} + 17$ où $K$ est un nombre réel.
|
||||||
|
|
||||||
|
Donner alors, en fonction de $a$, l'ensemble des solutions de $(E)$.
|
||||||
|
\end{enumerate}
|
||||||
|
On rappelle que la température des macarons à l'instant $t = 0$ est égale à $-15$~\degres C et que, au bout de $15$~min, elle est de $-1$~\degres C.
|
||||||
|
\begin{enumerate}
|
||||||
|
\setcounter{enumi}{1}
|
||||||
|
\item En utilisant la condition à $t=0$ démontrer que $K = -32$.
|
||||||
|
\item En utilisant la condition à $t=15$ démontrer que $a \approx -0.04$.
|
||||||
|
\item En déduire l'expression de la solution de l'équation différentielle puis étudier ses variations.
|
||||||
|
\item La température idéale de dégustation des macarons étant de $14$~\degres C, Marie estime que
|
||||||
|
celle-ci sera atteinte au bout de $30$~min. A-t-elle raison ? Justifier la réponse.
|
||||||
|
|
||||||
|
Sinon, combien de temps faudra-t-il attendre ?
|
||||||
|
\end{enumerate}
|
||||||
|
\end{exercise}
|
||||||
|
|
||||||
|
|
||||||
|
|
||||||
|
\end{document}
|
||||||
|
|
||||||
|
%%% Local Variables:
|
||||||
|
%%% mode: latex
|
||||||
|
%%% TeX-master: "master"
|
||||||
|
%%% End:
|
|
@ -0,0 +1,136 @@
|
||||||
|
\documentclass[a4paper,10pt]{article}
|
||||||
|
\usepackage{myXsim}
|
||||||
|
|
||||||
|
% Title Page
|
||||||
|
\title{DS8 \hfill HADJRAS Mohcine}
|
||||||
|
\tribe{TST sti2d}
|
||||||
|
\date{\hfillÀ render pour le vendredi 9 avril à 10h au plus tard}
|
||||||
|
|
||||||
|
\xsimsetup{
|
||||||
|
solution/print = false
|
||||||
|
}
|
||||||
|
|
||||||
|
\begin{document}
|
||||||
|
\maketitle
|
||||||
|
|
||||||
|
\begin{exercise}[subtitle={Étude de fonction}]
|
||||||
|
On considère la fonction $f$ définie sur $\intOF{0}{+\infty}$ par $ f(x) = 3x^2 + 78x + 240\ln(x)$
|
||||||
|
\begin{enumerate}
|
||||||
|
\item Démontrer que la dérivée de $f$ est $f'(x) = \frac{6x^2 + 78x + 240}{x}$.
|
||||||
|
\item Étude du numérateur de $f'(x)$: $N(x) = 6x^2 + 78x + 240$
|
||||||
|
\begin{enumerate}
|
||||||
|
\item Démontrer que $x=- 5$ et $x=- 8$ sont deux racines de $N(x)$..
|
||||||
|
\item Proposer une forme factorisée de $N(x)$.
|
||||||
|
\item Proposer une forme factorisée de $f'(x)$.
|
||||||
|
\end{enumerate}
|
||||||
|
\item Étudier le signe de $f'$ et en déduire les variations de $f$.
|
||||||
|
\end{enumerate}
|
||||||
|
\end{exercise}
|
||||||
|
|
||||||
|
\begin{solution}
|
||||||
|
\begin{enumerate}
|
||||||
|
\item pas de correction disponible
|
||||||
|
\item
|
||||||
|
\begin{enumerate}
|
||||||
|
\item \[N(- 5) = 0\]
|
||||||
|
\[N(- 8) = 0\]
|
||||||
|
\item \[
|
||||||
|
N(x) = 6(x - - 5)(x - - 8)
|
||||||
|
\]
|
||||||
|
\item
|
||||||
|
\[
|
||||||
|
f'(x) = \frac{6(x - - 5)(x - - 8)}{x}
|
||||||
|
\]
|
||||||
|
\end{enumerate}
|
||||||
|
\item Pas de correction disponible
|
||||||
|
\end{enumerate}
|
||||||
|
\end{solution}
|
||||||
|
|
||||||
|
\begin{exercise}[subtitle={Complexes}]
|
||||||
|
\begin{enumerate}
|
||||||
|
\item Mettre le nombre complexe suivant sous forme algébrique $z_1 = \dfrac{10 + 5 i}{-6 + 4 i} $
|
||||||
|
\item Mettre le complexe suivante sous forme exponentielle $z_2 = 1 + \sqrt{3} i$
|
||||||
|
\item Mettre le complexe suivante sous forme exponentielle $z_3 = -7 - 7 \sqrt{3} i$
|
||||||
|
\item Calculer le produit $z_4=z_2\times z_3$ donner le résultat sous forme exponentielle puis algébrique.
|
||||||
|
\item Calculer le quotient $z_5=\frac{z_2}{z_3}$ donner le résultat sous forme exponentielle puis algébrique.
|
||||||
|
\end{enumerate}
|
||||||
|
\end{exercise}
|
||||||
|
|
||||||
|
\begin{solution}
|
||||||
|
\begin{enumerate}
|
||||||
|
\item $z_1 = - \frac{10}{13} - \frac{35 i}{26}$
|
||||||
|
\item $z_2 = 2 e^{\frac{i \pi}{3}}$
|
||||||
|
\item $z_3 = 14 e^{- \frac{2 i \pi}{3}}$
|
||||||
|
\item $z_4 = 28 e^{- \frac{i \pi}{3}} = 14 - 14 \sqrt{3} i = 14.0 - 24.3 i$
|
||||||
|
\item $z_5 = \frac{1}{7} e^{i \pi} = - \frac{1}{7} = -0.143$
|
||||||
|
\end{enumerate}
|
||||||
|
\end{solution}
|
||||||
|
|
||||||
|
\begin{exercise}[subtitle={Sortie du congélateur}]
|
||||||
|
Marie a invité quelques amis pour le thé. Elle souhaite leur proposer ses macarons maison.
|
||||||
|
|
||||||
|
Elle les sort de son congélateur à $-18$~\degres C et les place dans une pièce à $25$~\degres C.
|
||||||
|
|
||||||
|
Au bout de 15 minutes, la température des macarons est de $4$~\degres C.
|
||||||
|
|
||||||
|
\bigskip
|
||||||
|
|
||||||
|
\textbf{Premier modèle}
|
||||||
|
|
||||||
|
\medskip
|
||||||
|
|
||||||
|
On suppose que la vitesse de décongélation est constante : chaque minute la hausse de
|
||||||
|
température des macarons est la même.
|
||||||
|
|
||||||
|
Estimer dans ce cadre la température au bout de $30$~minutes, puis au bout de $45$~minutes.
|
||||||
|
|
||||||
|
Cette modélisation est-elle pertinente?
|
||||||
|
|
||||||
|
\bigskip
|
||||||
|
|
||||||
|
\textbf{Deuxième modèle}
|
||||||
|
|
||||||
|
\medskip
|
||||||
|
|
||||||
|
On suppose maintenant que la vitesse de décongélation est proportionnelle à la différence
|
||||||
|
de température entre les macarons et l'air ambiant (il s'agit de la loi de Newton).
|
||||||
|
|
||||||
|
On désigne par $\theta$ la température des macarons à l'instant $t$, et par $\theta'$ la vitesse de décongélation.
|
||||||
|
|
||||||
|
L'unité de temps est la minute et l'unité de température le degré Celsius.
|
||||||
|
|
||||||
|
\smallskip
|
||||||
|
|
||||||
|
On négligera la diminution de température de la pièce et on admettra donc qu'il existe un
|
||||||
|
nombre réel $a$ tel que, pour $t$ positif :
|
||||||
|
|
||||||
|
\[\theta'(t) = a [\theta(t) - 25]\quad (E)\]
|
||||||
|
|
||||||
|
\medskip
|
||||||
|
|
||||||
|
\begin{enumerate}
|
||||||
|
\item Vérifier que l'équation $(E)$ a pour solutions $\theta(t) = K e^{at} + 25$ où $K$ est un nombre réel.
|
||||||
|
|
||||||
|
Donner alors, en fonction de $a$, l'ensemble des solutions de $(E)$.
|
||||||
|
\end{enumerate}
|
||||||
|
On rappelle que la température des macarons à l'instant $t = 0$ est égale à $-18$~\degres C et que, au bout de $15$~min, elle est de $4$~\degres C.
|
||||||
|
\begin{enumerate}
|
||||||
|
\setcounter{enumi}{1}
|
||||||
|
\item En utilisant la condition à $t=0$ démontrer que $K = -43$.
|
||||||
|
\item En utilisant la condition à $t=15$ démontrer que $a \approx -0.05$.
|
||||||
|
\item En déduire l'expression de la solution de l'équation différentielle puis étudier ses variations.
|
||||||
|
\item La température idéale de dégustation des macarons étant de $22$~\degres C, Marie estime que
|
||||||
|
celle-ci sera atteinte au bout de $30$~min. A-t-elle raison ? Justifier la réponse.
|
||||||
|
|
||||||
|
Sinon, combien de temps faudra-t-il attendre ?
|
||||||
|
\end{enumerate}
|
||||||
|
\end{exercise}
|
||||||
|
|
||||||
|
|
||||||
|
|
||||||
|
\end{document}
|
||||||
|
|
||||||
|
%%% Local Variables:
|
||||||
|
%%% mode: latex
|
||||||
|
%%% TeX-master: "master"
|
||||||
|
%%% End:
|
|
@ -0,0 +1,136 @@
|
||||||
|
\documentclass[a4paper,10pt]{article}
|
||||||
|
\usepackage{myXsim}
|
||||||
|
|
||||||
|
% Title Page
|
||||||
|
\title{DS8 \hfill HENRIST Maxime}
|
||||||
|
\tribe{TST sti2d}
|
||||||
|
\date{\hfillÀ render pour le vendredi 9 avril à 10h au plus tard}
|
||||||
|
|
||||||
|
\xsimsetup{
|
||||||
|
solution/print = false
|
||||||
|
}
|
||||||
|
|
||||||
|
\begin{document}
|
||||||
|
\maketitle
|
||||||
|
|
||||||
|
\begin{exercise}[subtitle={Étude de fonction}]
|
||||||
|
On considère la fonction $f$ définie sur $\intOF{0}{+\infty}$ par $ f(x) = 4.5x^2 + - 27x + - 36\ln(x)$
|
||||||
|
\begin{enumerate}
|
||||||
|
\item Démontrer que la dérivée de $f$ est $f'(x) = \frac{9x^2 + - 27x + - 36}{x}$.
|
||||||
|
\item Étude du numérateur de $f'(x)$: $N(x) = 9x^2 - 27x - 36$
|
||||||
|
\begin{enumerate}
|
||||||
|
\item Démontrer que $x=4$ et $x=- 1$ sont deux racines de $N(x)$..
|
||||||
|
\item Proposer une forme factorisée de $N(x)$.
|
||||||
|
\item Proposer une forme factorisée de $f'(x)$.
|
||||||
|
\end{enumerate}
|
||||||
|
\item Étudier le signe de $f'$ et en déduire les variations de $f$.
|
||||||
|
\end{enumerate}
|
||||||
|
\end{exercise}
|
||||||
|
|
||||||
|
\begin{solution}
|
||||||
|
\begin{enumerate}
|
||||||
|
\item pas de correction disponible
|
||||||
|
\item
|
||||||
|
\begin{enumerate}
|
||||||
|
\item \[N(4) = 0\]
|
||||||
|
\[N(- 1) = 0\]
|
||||||
|
\item \[
|
||||||
|
N(x) = 9(x - 4)(x - - 1)
|
||||||
|
\]
|
||||||
|
\item
|
||||||
|
\[
|
||||||
|
f'(x) = \frac{9(x - 4)(x - - 1)}{x}
|
||||||
|
\]
|
||||||
|
\end{enumerate}
|
||||||
|
\item Pas de correction disponible
|
||||||
|
\end{enumerate}
|
||||||
|
\end{solution}
|
||||||
|
|
||||||
|
\begin{exercise}[subtitle={Complexes}]
|
||||||
|
\begin{enumerate}
|
||||||
|
\item Mettre le nombre complexe suivant sous forme algébrique $z_1 = \dfrac{6 + 5 i}{-6 + 5 i} $
|
||||||
|
\item Mettre le complexe suivante sous forme exponentielle $z_2 = 8 \sqrt{3} - 8 i$
|
||||||
|
\item Mettre le complexe suivante sous forme exponentielle $z_3 = 5 - 5 \sqrt{3} i$
|
||||||
|
\item Calculer le produit $z_4=z_2\times z_3$ donner le résultat sous forme exponentielle puis algébrique.
|
||||||
|
\item Calculer le quotient $z_5=\frac{z_2}{z_3}$ donner le résultat sous forme exponentielle puis algébrique.
|
||||||
|
\end{enumerate}
|
||||||
|
\end{exercise}
|
||||||
|
|
||||||
|
\begin{solution}
|
||||||
|
\begin{enumerate}
|
||||||
|
\item $z_1 = - \frac{11}{61} - \frac{60 i}{61}$
|
||||||
|
\item $z_2 = 16 e^{- \frac{i \pi}{6}}$
|
||||||
|
\item $z_3 = 10 e^{- \frac{i \pi}{3}}$
|
||||||
|
\item $z_4 = 160 e^{- \frac{i \pi}{2}} = - 160 i = - 160.0 i$
|
||||||
|
\item $z_5 = \frac{8}{5} e^{\frac{i \pi}{6}} = \frac{4 \sqrt{3}}{5} + \frac{4 i}{5} = 1.39 + 0.8 i$
|
||||||
|
\end{enumerate}
|
||||||
|
\end{solution}
|
||||||
|
|
||||||
|
\begin{exercise}[subtitle={Sortie du congélateur}]
|
||||||
|
Marie a invité quelques amis pour le thé. Elle souhaite leur proposer ses macarons maison.
|
||||||
|
|
||||||
|
Elle les sort de son congélateur à $-16$~\degres C et les place dans une pièce à $23$~\degres C.
|
||||||
|
|
||||||
|
Au bout de 15 minutes, la température des macarons est de $0$~\degres C.
|
||||||
|
|
||||||
|
\bigskip
|
||||||
|
|
||||||
|
\textbf{Premier modèle}
|
||||||
|
|
||||||
|
\medskip
|
||||||
|
|
||||||
|
On suppose que la vitesse de décongélation est constante : chaque minute la hausse de
|
||||||
|
température des macarons est la même.
|
||||||
|
|
||||||
|
Estimer dans ce cadre la température au bout de $30$~minutes, puis au bout de $45$~minutes.
|
||||||
|
|
||||||
|
Cette modélisation est-elle pertinente?
|
||||||
|
|
||||||
|
\bigskip
|
||||||
|
|
||||||
|
\textbf{Deuxième modèle}
|
||||||
|
|
||||||
|
\medskip
|
||||||
|
|
||||||
|
On suppose maintenant que la vitesse de décongélation est proportionnelle à la différence
|
||||||
|
de température entre les macarons et l'air ambiant (il s'agit de la loi de Newton).
|
||||||
|
|
||||||
|
On désigne par $\theta$ la température des macarons à l'instant $t$, et par $\theta'$ la vitesse de décongélation.
|
||||||
|
|
||||||
|
L'unité de temps est la minute et l'unité de température le degré Celsius.
|
||||||
|
|
||||||
|
\smallskip
|
||||||
|
|
||||||
|
On négligera la diminution de température de la pièce et on admettra donc qu'il existe un
|
||||||
|
nombre réel $a$ tel que, pour $t$ positif :
|
||||||
|
|
||||||
|
\[\theta'(t) = a [\theta(t) - 23]\quad (E)\]
|
||||||
|
|
||||||
|
\medskip
|
||||||
|
|
||||||
|
\begin{enumerate}
|
||||||
|
\item Vérifier que l'équation $(E)$ a pour solutions $\theta(t) = K e^{at} + 23$ où $K$ est un nombre réel.
|
||||||
|
|
||||||
|
Donner alors, en fonction de $a$, l'ensemble des solutions de $(E)$.
|
||||||
|
\end{enumerate}
|
||||||
|
On rappelle que la température des macarons à l'instant $t = 0$ est égale à $-16$~\degres C et que, au bout de $15$~min, elle est de $0$~\degres C.
|
||||||
|
\begin{enumerate}
|
||||||
|
\setcounter{enumi}{1}
|
||||||
|
\item En utilisant la condition à $t=0$ démontrer que $K = -39$.
|
||||||
|
\item En utilisant la condition à $t=15$ démontrer que $a \approx -0.04$.
|
||||||
|
\item En déduire l'expression de la solution de l'équation différentielle puis étudier ses variations.
|
||||||
|
\item La température idéale de dégustation des macarons étant de $20$~\degres C, Marie estime que
|
||||||
|
celle-ci sera atteinte au bout de $30$~min. A-t-elle raison ? Justifier la réponse.
|
||||||
|
|
||||||
|
Sinon, combien de temps faudra-t-il attendre ?
|
||||||
|
\end{enumerate}
|
||||||
|
\end{exercise}
|
||||||
|
|
||||||
|
|
||||||
|
|
||||||
|
\end{document}
|
||||||
|
|
||||||
|
%%% Local Variables:
|
||||||
|
%%% mode: latex
|
||||||
|
%%% TeX-master: "master"
|
||||||
|
%%% End:
|
|
@ -0,0 +1,136 @@
|
||||||
|
\documentclass[a4paper,10pt]{article}
|
||||||
|
\usepackage{myXsim}
|
||||||
|
|
||||||
|
% Title Page
|
||||||
|
\title{DS8 \hfill HUMBERT Rayan}
|
||||||
|
\tribe{TST sti2d}
|
||||||
|
\date{\hfillÀ render pour le vendredi 9 avril à 10h au plus tard}
|
||||||
|
|
||||||
|
\xsimsetup{
|
||||||
|
solution/print = false
|
||||||
|
}
|
||||||
|
|
||||||
|
\begin{document}
|
||||||
|
\maketitle
|
||||||
|
|
||||||
|
\begin{exercise}[subtitle={Étude de fonction}]
|
||||||
|
On considère la fonction $f$ définie sur $\intOF{0}{+\infty}$ par $ f(x) = 2.5x^2 + - 25x + - 180\ln(x)$
|
||||||
|
\begin{enumerate}
|
||||||
|
\item Démontrer que la dérivée de $f$ est $f'(x) = \frac{5x^2 + - 25x + - 180}{x}$.
|
||||||
|
\item Étude du numérateur de $f'(x)$: $N(x) = 5x^2 - 25x - 180$
|
||||||
|
\begin{enumerate}
|
||||||
|
\item Démontrer que $x=- 4$ et $x=9$ sont deux racines de $N(x)$..
|
||||||
|
\item Proposer une forme factorisée de $N(x)$.
|
||||||
|
\item Proposer une forme factorisée de $f'(x)$.
|
||||||
|
\end{enumerate}
|
||||||
|
\item Étudier le signe de $f'$ et en déduire les variations de $f$.
|
||||||
|
\end{enumerate}
|
||||||
|
\end{exercise}
|
||||||
|
|
||||||
|
\begin{solution}
|
||||||
|
\begin{enumerate}
|
||||||
|
\item pas de correction disponible
|
||||||
|
\item
|
||||||
|
\begin{enumerate}
|
||||||
|
\item \[N(- 4) = 0\]
|
||||||
|
\[N(9) = 0\]
|
||||||
|
\item \[
|
||||||
|
N(x) = 5(x - - 4)(x - 9)
|
||||||
|
\]
|
||||||
|
\item
|
||||||
|
\[
|
||||||
|
f'(x) = \frac{5(x - - 4)(x - 9)}{x}
|
||||||
|
\]
|
||||||
|
\end{enumerate}
|
||||||
|
\item Pas de correction disponible
|
||||||
|
\end{enumerate}
|
||||||
|
\end{solution}
|
||||||
|
|
||||||
|
\begin{exercise}[subtitle={Complexes}]
|
||||||
|
\begin{enumerate}
|
||||||
|
\item Mettre le nombre complexe suivant sous forme algébrique $z_1 = \dfrac{8 + 3 i}{-6 + 6 i} $
|
||||||
|
\item Mettre le complexe suivante sous forme exponentielle $z_2 = -4 - 4 \sqrt{3} i$
|
||||||
|
\item Mettre le complexe suivante sous forme exponentielle $z_3 = 8 \sqrt{3} + 8 i$
|
||||||
|
\item Calculer le produit $z_4=z_2\times z_3$ donner le résultat sous forme exponentielle puis algébrique.
|
||||||
|
\item Calculer le quotient $z_5=\frac{z_2}{z_3}$ donner le résultat sous forme exponentielle puis algébrique.
|
||||||
|
\end{enumerate}
|
||||||
|
\end{exercise}
|
||||||
|
|
||||||
|
\begin{solution}
|
||||||
|
\begin{enumerate}
|
||||||
|
\item $z_1 = - \frac{5}{12} - \frac{11 i}{12}$
|
||||||
|
\item $z_2 = 8 e^{- \frac{2 i \pi}{3}}$
|
||||||
|
\item $z_3 = 16 e^{\frac{i \pi}{6}}$
|
||||||
|
\item $z_4 = 128 e^{- \frac{i \pi}{2}} = - 128 i = - 128.0 i$
|
||||||
|
\item $z_5 = \frac{1}{2} e^{- \frac{5 i \pi}{6}} = - \frac{\sqrt{3}}{4} - \frac{i}{4} = -0.433 - 0.25 i$
|
||||||
|
\end{enumerate}
|
||||||
|
\end{solution}
|
||||||
|
|
||||||
|
\begin{exercise}[subtitle={Sortie du congélateur}]
|
||||||
|
Marie a invité quelques amis pour le thé. Elle souhaite leur proposer ses macarons maison.
|
||||||
|
|
||||||
|
Elle les sort de son congélateur à $-20$~\degres C et les place dans une pièce à $25$~\degres C.
|
||||||
|
|
||||||
|
Au bout de 15 minutes, la température des macarons est de $-3$~\degres C.
|
||||||
|
|
||||||
|
\bigskip
|
||||||
|
|
||||||
|
\textbf{Premier modèle}
|
||||||
|
|
||||||
|
\medskip
|
||||||
|
|
||||||
|
On suppose que la vitesse de décongélation est constante : chaque minute la hausse de
|
||||||
|
température des macarons est la même.
|
||||||
|
|
||||||
|
Estimer dans ce cadre la température au bout de $30$~minutes, puis au bout de $45$~minutes.
|
||||||
|
|
||||||
|
Cette modélisation est-elle pertinente?
|
||||||
|
|
||||||
|
\bigskip
|
||||||
|
|
||||||
|
\textbf{Deuxième modèle}
|
||||||
|
|
||||||
|
\medskip
|
||||||
|
|
||||||
|
On suppose maintenant que la vitesse de décongélation est proportionnelle à la différence
|
||||||
|
de température entre les macarons et l'air ambiant (il s'agit de la loi de Newton).
|
||||||
|
|
||||||
|
On désigne par $\theta$ la température des macarons à l'instant $t$, et par $\theta'$ la vitesse de décongélation.
|
||||||
|
|
||||||
|
L'unité de temps est la minute et l'unité de température le degré Celsius.
|
||||||
|
|
||||||
|
\smallskip
|
||||||
|
|
||||||
|
On négligera la diminution de température de la pièce et on admettra donc qu'il existe un
|
||||||
|
nombre réel $a$ tel que, pour $t$ positif :
|
||||||
|
|
||||||
|
\[\theta'(t) = a [\theta(t) - 25]\quad (E)\]
|
||||||
|
|
||||||
|
\medskip
|
||||||
|
|
||||||
|
\begin{enumerate}
|
||||||
|
\item Vérifier que l'équation $(E)$ a pour solutions $\theta(t) = K e^{at} + 25$ où $K$ est un nombre réel.
|
||||||
|
|
||||||
|
Donner alors, en fonction de $a$, l'ensemble des solutions de $(E)$.
|
||||||
|
\end{enumerate}
|
||||||
|
On rappelle que la température des macarons à l'instant $t = 0$ est égale à $-20$~\degres C et que, au bout de $15$~min, elle est de $-3$~\degres C.
|
||||||
|
\begin{enumerate}
|
||||||
|
\setcounter{enumi}{1}
|
||||||
|
\item En utilisant la condition à $t=0$ démontrer que $K = -45$.
|
||||||
|
\item En utilisant la condition à $t=15$ démontrer que $a \approx -0.03$.
|
||||||
|
\item En déduire l'expression de la solution de l'équation différentielle puis étudier ses variations.
|
||||||
|
\item La température idéale de dégustation des macarons étant de $22$~\degres C, Marie estime que
|
||||||
|
celle-ci sera atteinte au bout de $30$~min. A-t-elle raison ? Justifier la réponse.
|
||||||
|
|
||||||
|
Sinon, combien de temps faudra-t-il attendre ?
|
||||||
|
\end{enumerate}
|
||||||
|
\end{exercise}
|
||||||
|
|
||||||
|
|
||||||
|
|
||||||
|
\end{document}
|
||||||
|
|
||||||
|
%%% Local Variables:
|
||||||
|
%%% mode: latex
|
||||||
|
%%% TeX-master: "master"
|
||||||
|
%%% End:
|
|
@ -0,0 +1,136 @@
|
||||||
|
\documentclass[a4paper,10pt]{article}
|
||||||
|
\usepackage{myXsim}
|
||||||
|
|
||||||
|
% Title Page
|
||||||
|
\title{DS8 \hfill KILINC Suleyman}
|
||||||
|
\tribe{TST sti2d}
|
||||||
|
\date{\hfillÀ render pour le vendredi 9 avril à 10h au plus tard}
|
||||||
|
|
||||||
|
\xsimsetup{
|
||||||
|
solution/print = false
|
||||||
|
}
|
||||||
|
|
||||||
|
\begin{document}
|
||||||
|
\maketitle
|
||||||
|
|
||||||
|
\begin{exercise}[subtitle={Étude de fonction}]
|
||||||
|
On considère la fonction $f$ définie sur $\intOF{0}{+\infty}$ par $ f(x) = 3.5x^2 + 21x + - 490\ln(x)$
|
||||||
|
\begin{enumerate}
|
||||||
|
\item Démontrer que la dérivée de $f$ est $f'(x) = \frac{7x^2 + 21x + - 490}{x}$.
|
||||||
|
\item Étude du numérateur de $f'(x)$: $N(x) = 7x^2 + 21x - 490$
|
||||||
|
\begin{enumerate}
|
||||||
|
\item Démontrer que $x=- 10$ et $x=7$ sont deux racines de $N(x)$..
|
||||||
|
\item Proposer une forme factorisée de $N(x)$.
|
||||||
|
\item Proposer une forme factorisée de $f'(x)$.
|
||||||
|
\end{enumerate}
|
||||||
|
\item Étudier le signe de $f'$ et en déduire les variations de $f$.
|
||||||
|
\end{enumerate}
|
||||||
|
\end{exercise}
|
||||||
|
|
||||||
|
\begin{solution}
|
||||||
|
\begin{enumerate}
|
||||||
|
\item pas de correction disponible
|
||||||
|
\item
|
||||||
|
\begin{enumerate}
|
||||||
|
\item \[N(- 10) = 0\]
|
||||||
|
\[N(7) = 0\]
|
||||||
|
\item \[
|
||||||
|
N(x) = 7(x - - 10)(x - 7)
|
||||||
|
\]
|
||||||
|
\item
|
||||||
|
\[
|
||||||
|
f'(x) = \frac{7(x - - 10)(x - 7)}{x}
|
||||||
|
\]
|
||||||
|
\end{enumerate}
|
||||||
|
\item Pas de correction disponible
|
||||||
|
\end{enumerate}
|
||||||
|
\end{solution}
|
||||||
|
|
||||||
|
\begin{exercise}[subtitle={Complexes}]
|
||||||
|
\begin{enumerate}
|
||||||
|
\item Mettre le nombre complexe suivant sous forme algébrique $z_1 = \dfrac{5 + 7 i}{-6 + 4 i} $
|
||||||
|
\item Mettre le complexe suivante sous forme exponentielle $z_2 = 1 - \sqrt{3} i$
|
||||||
|
\item Mettre le complexe suivante sous forme exponentielle $z_3 = - 8 \sqrt{2} + 8 \sqrt{2} i$
|
||||||
|
\item Calculer le produit $z_4=z_2\times z_3$ donner le résultat sous forme exponentielle puis algébrique.
|
||||||
|
\item Calculer le quotient $z_5=\frac{z_2}{z_3}$ donner le résultat sous forme exponentielle puis algébrique.
|
||||||
|
\end{enumerate}
|
||||||
|
\end{exercise}
|
||||||
|
|
||||||
|
\begin{solution}
|
||||||
|
\begin{enumerate}
|
||||||
|
\item $z_1 = - \frac{1}{26} - \frac{31 i}{26}$
|
||||||
|
\item $z_2 = 2 e^{- \frac{i \pi}{3}}$
|
||||||
|
\item $z_3 = 16 e^{\frac{3 i \pi}{4}}$
|
||||||
|
\item $z_4 = 32 e^{\frac{5 i \pi}{12}} = - 8 \sqrt{2} + 8 \sqrt{6} + i \left(8 \sqrt{2} + 8 \sqrt{6}\right) = 8.28 + 30.9 i$
|
||||||
|
\item $z_5 = \frac{1}{8} e^{- \frac{13 i \pi}{12}} = - \frac{\sqrt{6}}{32} - \frac{\sqrt{2}}{32} + i \left(- \frac{\sqrt{2}}{32} + \frac{\sqrt{6}}{32}\right) = -0.121 + 0.0323 i$
|
||||||
|
\end{enumerate}
|
||||||
|
\end{solution}
|
||||||
|
|
||||||
|
\begin{exercise}[subtitle={Sortie du congélateur}]
|
||||||
|
Marie a invité quelques amis pour le thé. Elle souhaite leur proposer ses macarons maison.
|
||||||
|
|
||||||
|
Elle les sort de son congélateur à $-19$~\degres C et les place dans une pièce à $15$~\degres C.
|
||||||
|
|
||||||
|
Au bout de 15 minutes, la température des macarons est de $-2$~\degres C.
|
||||||
|
|
||||||
|
\bigskip
|
||||||
|
|
||||||
|
\textbf{Premier modèle}
|
||||||
|
|
||||||
|
\medskip
|
||||||
|
|
||||||
|
On suppose que la vitesse de décongélation est constante : chaque minute la hausse de
|
||||||
|
température des macarons est la même.
|
||||||
|
|
||||||
|
Estimer dans ce cadre la température au bout de $30$~minutes, puis au bout de $45$~minutes.
|
||||||
|
|
||||||
|
Cette modélisation est-elle pertinente?
|
||||||
|
|
||||||
|
\bigskip
|
||||||
|
|
||||||
|
\textbf{Deuxième modèle}
|
||||||
|
|
||||||
|
\medskip
|
||||||
|
|
||||||
|
On suppose maintenant que la vitesse de décongélation est proportionnelle à la différence
|
||||||
|
de température entre les macarons et l'air ambiant (il s'agit de la loi de Newton).
|
||||||
|
|
||||||
|
On désigne par $\theta$ la température des macarons à l'instant $t$, et par $\theta'$ la vitesse de décongélation.
|
||||||
|
|
||||||
|
L'unité de temps est la minute et l'unité de température le degré Celsius.
|
||||||
|
|
||||||
|
\smallskip
|
||||||
|
|
||||||
|
On négligera la diminution de température de la pièce et on admettra donc qu'il existe un
|
||||||
|
nombre réel $a$ tel que, pour $t$ positif :
|
||||||
|
|
||||||
|
\[\theta'(t) = a [\theta(t) - 15]\quad (E)\]
|
||||||
|
|
||||||
|
\medskip
|
||||||
|
|
||||||
|
\begin{enumerate}
|
||||||
|
\item Vérifier que l'équation $(E)$ a pour solutions $\theta(t) = K e^{at} + 15$ où $K$ est un nombre réel.
|
||||||
|
|
||||||
|
Donner alors, en fonction de $a$, l'ensemble des solutions de $(E)$.
|
||||||
|
\end{enumerate}
|
||||||
|
On rappelle que la température des macarons à l'instant $t = 0$ est égale à $-19$~\degres C et que, au bout de $15$~min, elle est de $-2$~\degres C.
|
||||||
|
\begin{enumerate}
|
||||||
|
\setcounter{enumi}{1}
|
||||||
|
\item En utilisant la condition à $t=0$ démontrer que $K = -34$.
|
||||||
|
\item En utilisant la condition à $t=15$ démontrer que $a \approx -0.05$.
|
||||||
|
\item En déduire l'expression de la solution de l'équation différentielle puis étudier ses variations.
|
||||||
|
\item La température idéale de dégustation des macarons étant de $12$~\degres C, Marie estime que
|
||||||
|
celle-ci sera atteinte au bout de $30$~min. A-t-elle raison ? Justifier la réponse.
|
||||||
|
|
||||||
|
Sinon, combien de temps faudra-t-il attendre ?
|
||||||
|
\end{enumerate}
|
||||||
|
\end{exercise}
|
||||||
|
|
||||||
|
|
||||||
|
|
||||||
|
\end{document}
|
||||||
|
|
||||||
|
%%% Local Variables:
|
||||||
|
%%% mode: latex
|
||||||
|
%%% TeX-master: "master"
|
||||||
|
%%% End:
|
|
@ -0,0 +1,136 @@
|
||||||
|
\documentclass[a4paper,10pt]{article}
|
||||||
|
\usepackage{myXsim}
|
||||||
|
|
||||||
|
% Title Page
|
||||||
|
\title{DS8 \hfill M'BAREK HASNAOUI Bilal}
|
||||||
|
\tribe{TST sti2d}
|
||||||
|
\date{\hfillÀ render pour le vendredi 9 avril à 10h au plus tard}
|
||||||
|
|
||||||
|
\xsimsetup{
|
||||||
|
solution/print = false
|
||||||
|
}
|
||||||
|
|
||||||
|
\begin{document}
|
||||||
|
\maketitle
|
||||||
|
|
||||||
|
\begin{exercise}[subtitle={Étude de fonction}]
|
||||||
|
On considère la fonction $f$ définie sur $\intOF{0}{+\infty}$ par $ f(x) = 5x^2 + - 40x + - 450\ln(x)$
|
||||||
|
\begin{enumerate}
|
||||||
|
\item Démontrer que la dérivée de $f$ est $f'(x) = \frac{10x^2 + - 40x + - 450}{x}$.
|
||||||
|
\item Étude du numérateur de $f'(x)$: $N(x) = 10x^2 - 40x - 450$
|
||||||
|
\begin{enumerate}
|
||||||
|
\item Démontrer que $x=9$ et $x=- 5$ sont deux racines de $N(x)$..
|
||||||
|
\item Proposer une forme factorisée de $N(x)$.
|
||||||
|
\item Proposer une forme factorisée de $f'(x)$.
|
||||||
|
\end{enumerate}
|
||||||
|
\item Étudier le signe de $f'$ et en déduire les variations de $f$.
|
||||||
|
\end{enumerate}
|
||||||
|
\end{exercise}
|
||||||
|
|
||||||
|
\begin{solution}
|
||||||
|
\begin{enumerate}
|
||||||
|
\item pas de correction disponible
|
||||||
|
\item
|
||||||
|
\begin{enumerate}
|
||||||
|
\item \[N(9) = 0\]
|
||||||
|
\[N(- 5) = 0\]
|
||||||
|
\item \[
|
||||||
|
N(x) = 10(x - 9)(x - - 5)
|
||||||
|
\]
|
||||||
|
\item
|
||||||
|
\[
|
||||||
|
f'(x) = \frac{10(x - 9)(x - - 5)}{x}
|
||||||
|
\]
|
||||||
|
\end{enumerate}
|
||||||
|
\item Pas de correction disponible
|
||||||
|
\end{enumerate}
|
||||||
|
\end{solution}
|
||||||
|
|
||||||
|
\begin{exercise}[subtitle={Complexes}]
|
||||||
|
\begin{enumerate}
|
||||||
|
\item Mettre le nombre complexe suivant sous forme algébrique $z_1 = \dfrac{2 + 4 i}{-5 + 7 i} $
|
||||||
|
\item Mettre le complexe suivante sous forme exponentielle $z_2 = 7 + 7 \sqrt{3} i$
|
||||||
|
\item Mettre le complexe suivante sous forme exponentielle $z_3 = 7 \sqrt{2} + 7 \sqrt{2} i$
|
||||||
|
\item Calculer le produit $z_4=z_2\times z_3$ donner le résultat sous forme exponentielle puis algébrique.
|
||||||
|
\item Calculer le quotient $z_5=\frac{z_2}{z_3}$ donner le résultat sous forme exponentielle puis algébrique.
|
||||||
|
\end{enumerate}
|
||||||
|
\end{exercise}
|
||||||
|
|
||||||
|
\begin{solution}
|
||||||
|
\begin{enumerate}
|
||||||
|
\item $z_1 = \frac{9}{37} - \frac{17 i}{37}$
|
||||||
|
\item $z_2 = 14 e^{\frac{i \pi}{3}}$
|
||||||
|
\item $z_3 = 14 e^{\frac{i \pi}{4}}$
|
||||||
|
\item $z_4 = 196 e^{\frac{7 i \pi}{12}} = - 49 \sqrt{6} + 49 \sqrt{2} + i \left(49 \sqrt{2} + 49 \sqrt{6}\right) = -50.7 + 189.0 i$
|
||||||
|
\item $z_5 = 1 e^{\frac{i \pi}{12}} = \frac{\sqrt{2}}{4} + \frac{\sqrt{6}}{4} + i \left(- \frac{\sqrt{2}}{4} + \frac{\sqrt{6}}{4}\right) = 0.966 + 0.259 i$
|
||||||
|
\end{enumerate}
|
||||||
|
\end{solution}
|
||||||
|
|
||||||
|
\begin{exercise}[subtitle={Sortie du congélateur}]
|
||||||
|
Marie a invité quelques amis pour le thé. Elle souhaite leur proposer ses macarons maison.
|
||||||
|
|
||||||
|
Elle les sort de son congélateur à $-15$~\degres C et les place dans une pièce à $25$~\degres C.
|
||||||
|
|
||||||
|
Au bout de 15 minutes, la température des macarons est de $0$~\degres C.
|
||||||
|
|
||||||
|
\bigskip
|
||||||
|
|
||||||
|
\textbf{Premier modèle}
|
||||||
|
|
||||||
|
\medskip
|
||||||
|
|
||||||
|
On suppose que la vitesse de décongélation est constante : chaque minute la hausse de
|
||||||
|
température des macarons est la même.
|
||||||
|
|
||||||
|
Estimer dans ce cadre la température au bout de $30$~minutes, puis au bout de $45$~minutes.
|
||||||
|
|
||||||
|
Cette modélisation est-elle pertinente?
|
||||||
|
|
||||||
|
\bigskip
|
||||||
|
|
||||||
|
\textbf{Deuxième modèle}
|
||||||
|
|
||||||
|
\medskip
|
||||||
|
|
||||||
|
On suppose maintenant que la vitesse de décongélation est proportionnelle à la différence
|
||||||
|
de température entre les macarons et l'air ambiant (il s'agit de la loi de Newton).
|
||||||
|
|
||||||
|
On désigne par $\theta$ la température des macarons à l'instant $t$, et par $\theta'$ la vitesse de décongélation.
|
||||||
|
|
||||||
|
L'unité de temps est la minute et l'unité de température le degré Celsius.
|
||||||
|
|
||||||
|
\smallskip
|
||||||
|
|
||||||
|
On négligera la diminution de température de la pièce et on admettra donc qu'il existe un
|
||||||
|
nombre réel $a$ tel que, pour $t$ positif :
|
||||||
|
|
||||||
|
\[\theta'(t) = a [\theta(t) - 25]\quad (E)\]
|
||||||
|
|
||||||
|
\medskip
|
||||||
|
|
||||||
|
\begin{enumerate}
|
||||||
|
\item Vérifier que l'équation $(E)$ a pour solutions $\theta(t) = K e^{at} + 25$ où $K$ est un nombre réel.
|
||||||
|
|
||||||
|
Donner alors, en fonction de $a$, l'ensemble des solutions de $(E)$.
|
||||||
|
\end{enumerate}
|
||||||
|
On rappelle que la température des macarons à l'instant $t = 0$ est égale à $-15$~\degres C et que, au bout de $15$~min, elle est de $0$~\degres C.
|
||||||
|
\begin{enumerate}
|
||||||
|
\setcounter{enumi}{1}
|
||||||
|
\item En utilisant la condition à $t=0$ démontrer que $K = -40$.
|
||||||
|
\item En utilisant la condition à $t=15$ démontrer que $a \approx -0.03$.
|
||||||
|
\item En déduire l'expression de la solution de l'équation différentielle puis étudier ses variations.
|
||||||
|
\item La température idéale de dégustation des macarons étant de $22$~\degres C, Marie estime que
|
||||||
|
celle-ci sera atteinte au bout de $30$~min. A-t-elle raison ? Justifier la réponse.
|
||||||
|
|
||||||
|
Sinon, combien de temps faudra-t-il attendre ?
|
||||||
|
\end{enumerate}
|
||||||
|
\end{exercise}
|
||||||
|
|
||||||
|
|
||||||
|
|
||||||
|
\end{document}
|
||||||
|
|
||||||
|
%%% Local Variables:
|
||||||
|
%%% mode: latex
|
||||||
|
%%% TeX-master: "master"
|
||||||
|
%%% End:
|
|
@ -0,0 +1,136 @@
|
||||||
|
\documentclass[a4paper,10pt]{article}
|
||||||
|
\usepackage{myXsim}
|
||||||
|
|
||||||
|
% Title Page
|
||||||
|
\title{DS8 \hfill MERCIER Almandin}
|
||||||
|
\tribe{TST sti2d}
|
||||||
|
\date{\hfillÀ render pour le vendredi 9 avril à 10h au plus tard}
|
||||||
|
|
||||||
|
\xsimsetup{
|
||||||
|
solution/print = false
|
||||||
|
}
|
||||||
|
|
||||||
|
\begin{document}
|
||||||
|
\maketitle
|
||||||
|
|
||||||
|
\begin{exercise}[subtitle={Étude de fonction}]
|
||||||
|
On considère la fonction $f$ définie sur $\intOF{0}{+\infty}$ par $ f(x) = 2.5x^2 + - 50x + 120\ln(x)$
|
||||||
|
\begin{enumerate}
|
||||||
|
\item Démontrer que la dérivée de $f$ est $f'(x) = \frac{5x^2 + - 50x + 120}{x}$.
|
||||||
|
\item Étude du numérateur de $f'(x)$: $N(x) = 5x^2 - 50x + 120$
|
||||||
|
\begin{enumerate}
|
||||||
|
\item Démontrer que $x=6$ et $x=4$ sont deux racines de $N(x)$..
|
||||||
|
\item Proposer une forme factorisée de $N(x)$.
|
||||||
|
\item Proposer une forme factorisée de $f'(x)$.
|
||||||
|
\end{enumerate}
|
||||||
|
\item Étudier le signe de $f'$ et en déduire les variations de $f$.
|
||||||
|
\end{enumerate}
|
||||||
|
\end{exercise}
|
||||||
|
|
||||||
|
\begin{solution}
|
||||||
|
\begin{enumerate}
|
||||||
|
\item pas de correction disponible
|
||||||
|
\item
|
||||||
|
\begin{enumerate}
|
||||||
|
\item \[N(6) = 0\]
|
||||||
|
\[N(4) = 0\]
|
||||||
|
\item \[
|
||||||
|
N(x) = 5(x - 6)(x - 4)
|
||||||
|
\]
|
||||||
|
\item
|
||||||
|
\[
|
||||||
|
f'(x) = \frac{5(x - 6)(x - 4)}{x}
|
||||||
|
\]
|
||||||
|
\end{enumerate}
|
||||||
|
\item Pas de correction disponible
|
||||||
|
\end{enumerate}
|
||||||
|
\end{solution}
|
||||||
|
|
||||||
|
\begin{exercise}[subtitle={Complexes}]
|
||||||
|
\begin{enumerate}
|
||||||
|
\item Mettre le nombre complexe suivant sous forme algébrique $z_1 = \dfrac{10 + 10 i}{-2 + 3 i} $
|
||||||
|
\item Mettre le complexe suivante sous forme exponentielle $z_2 = - 7 \sqrt{2} - 7 \sqrt{2} i$
|
||||||
|
\item Mettre le complexe suivante sous forme exponentielle $z_3 = - 9 \sqrt{2} + 9 \sqrt{2} i$
|
||||||
|
\item Calculer le produit $z_4=z_2\times z_3$ donner le résultat sous forme exponentielle puis algébrique.
|
||||||
|
\item Calculer le quotient $z_5=\frac{z_2}{z_3}$ donner le résultat sous forme exponentielle puis algébrique.
|
||||||
|
\end{enumerate}
|
||||||
|
\end{exercise}
|
||||||
|
|
||||||
|
\begin{solution}
|
||||||
|
\begin{enumerate}
|
||||||
|
\item $z_1 = \frac{10}{13} - \frac{50 i}{13}$
|
||||||
|
\item $z_2 = 14 e^{- \frac{3 i \pi}{4}}$
|
||||||
|
\item $z_3 = 18 e^{\frac{3 i \pi}{4}}$
|
||||||
|
\item $z_4 = 252 e^{0} = 252 = 252.0$
|
||||||
|
\item $z_5 = \frac{7}{9} e^{- \frac{3 i \pi}{2}} = \frac{7 i}{9} = 0.778 i$
|
||||||
|
\end{enumerate}
|
||||||
|
\end{solution}
|
||||||
|
|
||||||
|
\begin{exercise}[subtitle={Sortie du congélateur}]
|
||||||
|
Marie a invité quelques amis pour le thé. Elle souhaite leur proposer ses macarons maison.
|
||||||
|
|
||||||
|
Elle les sort de son congélateur à $-16$~\degres C et les place dans une pièce à $22$~\degres C.
|
||||||
|
|
||||||
|
Au bout de 15 minutes, la température des macarons est de $3$~\degres C.
|
||||||
|
|
||||||
|
\bigskip
|
||||||
|
|
||||||
|
\textbf{Premier modèle}
|
||||||
|
|
||||||
|
\medskip
|
||||||
|
|
||||||
|
On suppose que la vitesse de décongélation est constante : chaque minute la hausse de
|
||||||
|
température des macarons est la même.
|
||||||
|
|
||||||
|
Estimer dans ce cadre la température au bout de $30$~minutes, puis au bout de $45$~minutes.
|
||||||
|
|
||||||
|
Cette modélisation est-elle pertinente?
|
||||||
|
|
||||||
|
\bigskip
|
||||||
|
|
||||||
|
\textbf{Deuxième modèle}
|
||||||
|
|
||||||
|
\medskip
|
||||||
|
|
||||||
|
On suppose maintenant que la vitesse de décongélation est proportionnelle à la différence
|
||||||
|
de température entre les macarons et l'air ambiant (il s'agit de la loi de Newton).
|
||||||
|
|
||||||
|
On désigne par $\theta$ la température des macarons à l'instant $t$, et par $\theta'$ la vitesse de décongélation.
|
||||||
|
|
||||||
|
L'unité de temps est la minute et l'unité de température le degré Celsius.
|
||||||
|
|
||||||
|
\smallskip
|
||||||
|
|
||||||
|
On négligera la diminution de température de la pièce et on admettra donc qu'il existe un
|
||||||
|
nombre réel $a$ tel que, pour $t$ positif :
|
||||||
|
|
||||||
|
\[\theta'(t) = a [\theta(t) - 22]\quad (E)\]
|
||||||
|
|
||||||
|
\medskip
|
||||||
|
|
||||||
|
\begin{enumerate}
|
||||||
|
\item Vérifier que l'équation $(E)$ a pour solutions $\theta(t) = K e^{at} + 22$ où $K$ est un nombre réel.
|
||||||
|
|
||||||
|
Donner alors, en fonction de $a$, l'ensemble des solutions de $(E)$.
|
||||||
|
\end{enumerate}
|
||||||
|
On rappelle que la température des macarons à l'instant $t = 0$ est égale à $-16$~\degres C et que, au bout de $15$~min, elle est de $3$~\degres C.
|
||||||
|
\begin{enumerate}
|
||||||
|
\setcounter{enumi}{1}
|
||||||
|
\item En utilisant la condition à $t=0$ démontrer que $K = -38$.
|
||||||
|
\item En utilisant la condition à $t=15$ démontrer que $a \approx -0.05$.
|
||||||
|
\item En déduire l'expression de la solution de l'équation différentielle puis étudier ses variations.
|
||||||
|
\item La température idéale de dégustation des macarons étant de $19$~\degres C, Marie estime que
|
||||||
|
celle-ci sera atteinte au bout de $30$~min. A-t-elle raison ? Justifier la réponse.
|
||||||
|
|
||||||
|
Sinon, combien de temps faudra-t-il attendre ?
|
||||||
|
\end{enumerate}
|
||||||
|
\end{exercise}
|
||||||
|
|
||||||
|
|
||||||
|
|
||||||
|
\end{document}
|
||||||
|
|
||||||
|
%%% Local Variables:
|
||||||
|
%%% mode: latex
|
||||||
|
%%% TeX-master: "master"
|
||||||
|
%%% End:
|
|
@ -0,0 +1,136 @@
|
||||||
|
\documentclass[a4paper,10pt]{article}
|
||||||
|
\usepackage{myXsim}
|
||||||
|
|
||||||
|
% Title Page
|
||||||
|
\title{DS8 \hfill MOUFAQ Amine}
|
||||||
|
\tribe{TST sti2d}
|
||||||
|
\date{\hfillÀ render pour le vendredi 9 avril à 10h au plus tard}
|
||||||
|
|
||||||
|
\xsimsetup{
|
||||||
|
solution/print = false
|
||||||
|
}
|
||||||
|
|
||||||
|
\begin{document}
|
||||||
|
\maketitle
|
||||||
|
|
||||||
|
\begin{exercise}[subtitle={Étude de fonction}]
|
||||||
|
On considère la fonction $f$ définie sur $\intOF{0}{+\infty}$ par $ f(x) = 5x^2 + 0x + - 810\ln(x)$
|
||||||
|
\begin{enumerate}
|
||||||
|
\item Démontrer que la dérivée de $f$ est $f'(x) = \frac{10x^2 + 0x + - 810}{x}$.
|
||||||
|
\item Étude du numérateur de $f'(x)$: $N(x) = 10x^2 - 810$
|
||||||
|
\begin{enumerate}
|
||||||
|
\item Démontrer que $x=- 9$ et $x=9$ sont deux racines de $N(x)$..
|
||||||
|
\item Proposer une forme factorisée de $N(x)$.
|
||||||
|
\item Proposer une forme factorisée de $f'(x)$.
|
||||||
|
\end{enumerate}
|
||||||
|
\item Étudier le signe de $f'$ et en déduire les variations de $f$.
|
||||||
|
\end{enumerate}
|
||||||
|
\end{exercise}
|
||||||
|
|
||||||
|
\begin{solution}
|
||||||
|
\begin{enumerate}
|
||||||
|
\item pas de correction disponible
|
||||||
|
\item
|
||||||
|
\begin{enumerate}
|
||||||
|
\item \[N(- 9) = 0\]
|
||||||
|
\[N(9) = 0\]
|
||||||
|
\item \[
|
||||||
|
N(x) = 10(x - - 9)(x - 9)
|
||||||
|
\]
|
||||||
|
\item
|
||||||
|
\[
|
||||||
|
f'(x) = \frac{10(x - - 9)(x - 9)}{x}
|
||||||
|
\]
|
||||||
|
\end{enumerate}
|
||||||
|
\item Pas de correction disponible
|
||||||
|
\end{enumerate}
|
||||||
|
\end{solution}
|
||||||
|
|
||||||
|
\begin{exercise}[subtitle={Complexes}]
|
||||||
|
\begin{enumerate}
|
||||||
|
\item Mettre le nombre complexe suivant sous forme algébrique $z_1 = \dfrac{8 + 4 i}{-3 + 5 i} $
|
||||||
|
\item Mettre le complexe suivante sous forme exponentielle $z_2 = - 2 \sqrt{2} + 2 \sqrt{2} i$
|
||||||
|
\item Mettre le complexe suivante sous forme exponentielle $z_3 = 6 + 6 \sqrt{3} i$
|
||||||
|
\item Calculer le produit $z_4=z_2\times z_3$ donner le résultat sous forme exponentielle puis algébrique.
|
||||||
|
\item Calculer le quotient $z_5=\frac{z_2}{z_3}$ donner le résultat sous forme exponentielle puis algébrique.
|
||||||
|
\end{enumerate}
|
||||||
|
\end{exercise}
|
||||||
|
|
||||||
|
\begin{solution}
|
||||||
|
\begin{enumerate}
|
||||||
|
\item $z_1 = - \frac{2}{17} - \frac{26 i}{17}$
|
||||||
|
\item $z_2 = 4 e^{\frac{3 i \pi}{4}}$
|
||||||
|
\item $z_3 = 12 e^{\frac{i \pi}{3}}$
|
||||||
|
\item $z_4 = 48 e^{\frac{13 i \pi}{12}} = - 12 \sqrt{6} - 12 \sqrt{2} + i \left(- 12 \sqrt{6} + 12 \sqrt{2}\right) = -46.4 - 12.4 i$
|
||||||
|
\item $z_5 = \frac{1}{3} e^{\frac{5 i \pi}{12}} = - \frac{\sqrt{2}}{12} + \frac{\sqrt{6}}{12} + i \left(\frac{\sqrt{2}}{12} + \frac{\sqrt{6}}{12}\right) = 0.0863 + 0.322 i$
|
||||||
|
\end{enumerate}
|
||||||
|
\end{solution}
|
||||||
|
|
||||||
|
\begin{exercise}[subtitle={Sortie du congélateur}]
|
||||||
|
Marie a invité quelques amis pour le thé. Elle souhaite leur proposer ses macarons maison.
|
||||||
|
|
||||||
|
Elle les sort de son congélateur à $-15$~\degres C et les place dans une pièce à $20$~\degres C.
|
||||||
|
|
||||||
|
Au bout de 15 minutes, la température des macarons est de $-3$~\degres C.
|
||||||
|
|
||||||
|
\bigskip
|
||||||
|
|
||||||
|
\textbf{Premier modèle}
|
||||||
|
|
||||||
|
\medskip
|
||||||
|
|
||||||
|
On suppose que la vitesse de décongélation est constante : chaque minute la hausse de
|
||||||
|
température des macarons est la même.
|
||||||
|
|
||||||
|
Estimer dans ce cadre la température au bout de $30$~minutes, puis au bout de $45$~minutes.
|
||||||
|
|
||||||
|
Cette modélisation est-elle pertinente?
|
||||||
|
|
||||||
|
\bigskip
|
||||||
|
|
||||||
|
\textbf{Deuxième modèle}
|
||||||
|
|
||||||
|
\medskip
|
||||||
|
|
||||||
|
On suppose maintenant que la vitesse de décongélation est proportionnelle à la différence
|
||||||
|
de température entre les macarons et l'air ambiant (il s'agit de la loi de Newton).
|
||||||
|
|
||||||
|
On désigne par $\theta$ la température des macarons à l'instant $t$, et par $\theta'$ la vitesse de décongélation.
|
||||||
|
|
||||||
|
L'unité de temps est la minute et l'unité de température le degré Celsius.
|
||||||
|
|
||||||
|
\smallskip
|
||||||
|
|
||||||
|
On négligera la diminution de température de la pièce et on admettra donc qu'il existe un
|
||||||
|
nombre réel $a$ tel que, pour $t$ positif :
|
||||||
|
|
||||||
|
\[\theta'(t) = a [\theta(t) - 20]\quad (E)\]
|
||||||
|
|
||||||
|
\medskip
|
||||||
|
|
||||||
|
\begin{enumerate}
|
||||||
|
\item Vérifier que l'équation $(E)$ a pour solutions $\theta(t) = K e^{at} + 20$ où $K$ est un nombre réel.
|
||||||
|
|
||||||
|
Donner alors, en fonction de $a$, l'ensemble des solutions de $(E)$.
|
||||||
|
\end{enumerate}
|
||||||
|
On rappelle que la température des macarons à l'instant $t = 0$ est égale à $-15$~\degres C et que, au bout de $15$~min, elle est de $-3$~\degres C.
|
||||||
|
\begin{enumerate}
|
||||||
|
\setcounter{enumi}{1}
|
||||||
|
\item En utilisant la condition à $t=0$ démontrer que $K = -35$.
|
||||||
|
\item En utilisant la condition à $t=15$ démontrer que $a \approx -0.03$.
|
||||||
|
\item En déduire l'expression de la solution de l'équation différentielle puis étudier ses variations.
|
||||||
|
\item La température idéale de dégustation des macarons étant de $17$~\degres C, Marie estime que
|
||||||
|
celle-ci sera atteinte au bout de $30$~min. A-t-elle raison ? Justifier la réponse.
|
||||||
|
|
||||||
|
Sinon, combien de temps faudra-t-il attendre ?
|
||||||
|
\end{enumerate}
|
||||||
|
\end{exercise}
|
||||||
|
|
||||||
|
|
||||||
|
|
||||||
|
\end{document}
|
||||||
|
|
||||||
|
%%% Local Variables:
|
||||||
|
%%% mode: latex
|
||||||
|
%%% TeX-master: "master"
|
||||||
|
%%% End:
|
|
@ -0,0 +1,136 @@
|
||||||
|
\documentclass[a4paper,10pt]{article}
|
||||||
|
\usepackage{myXsim}
|
||||||
|
|
||||||
|
% Title Page
|
||||||
|
\title{DS8 \hfill NARDINI Kakary}
|
||||||
|
\tribe{TST sti2d}
|
||||||
|
\date{\hfillÀ render pour le vendredi 9 avril à 10h au plus tard}
|
||||||
|
|
||||||
|
\xsimsetup{
|
||||||
|
solution/print = false
|
||||||
|
}
|
||||||
|
|
||||||
|
\begin{document}
|
||||||
|
\maketitle
|
||||||
|
|
||||||
|
\begin{exercise}[subtitle={Étude de fonction}]
|
||||||
|
On considère la fonction $f$ définie sur $\intOF{0}{+\infty}$ par $ f(x) = 3x^2 + - 18x + - 108\ln(x)$
|
||||||
|
\begin{enumerate}
|
||||||
|
\item Démontrer que la dérivée de $f$ est $f'(x) = \frac{6x^2 + - 18x + - 108}{x}$.
|
||||||
|
\item Étude du numérateur de $f'(x)$: $N(x) = 6x^2 - 18x - 108$
|
||||||
|
\begin{enumerate}
|
||||||
|
\item Démontrer que $x=6$ et $x=- 3$ sont deux racines de $N(x)$..
|
||||||
|
\item Proposer une forme factorisée de $N(x)$.
|
||||||
|
\item Proposer une forme factorisée de $f'(x)$.
|
||||||
|
\end{enumerate}
|
||||||
|
\item Étudier le signe de $f'$ et en déduire les variations de $f$.
|
||||||
|
\end{enumerate}
|
||||||
|
\end{exercise}
|
||||||
|
|
||||||
|
\begin{solution}
|
||||||
|
\begin{enumerate}
|
||||||
|
\item pas de correction disponible
|
||||||
|
\item
|
||||||
|
\begin{enumerate}
|
||||||
|
\item \[N(6) = 0\]
|
||||||
|
\[N(- 3) = 0\]
|
||||||
|
\item \[
|
||||||
|
N(x) = 6(x - 6)(x - - 3)
|
||||||
|
\]
|
||||||
|
\item
|
||||||
|
\[
|
||||||
|
f'(x) = \frac{6(x - 6)(x - - 3)}{x}
|
||||||
|
\]
|
||||||
|
\end{enumerate}
|
||||||
|
\item Pas de correction disponible
|
||||||
|
\end{enumerate}
|
||||||
|
\end{solution}
|
||||||
|
|
||||||
|
\begin{exercise}[subtitle={Complexes}]
|
||||||
|
\begin{enumerate}
|
||||||
|
\item Mettre le nombre complexe suivant sous forme algébrique $z_1 = \dfrac{9 + 6 i}{-3 + 6 i} $
|
||||||
|
\item Mettre le complexe suivante sous forme exponentielle $z_2 = -4 - 4 \sqrt{3} i$
|
||||||
|
\item Mettre le complexe suivante sous forme exponentielle $z_3 = 9 \sqrt{2} + 9 \sqrt{2} i$
|
||||||
|
\item Calculer le produit $z_4=z_2\times z_3$ donner le résultat sous forme exponentielle puis algébrique.
|
||||||
|
\item Calculer le quotient $z_5=\frac{z_2}{z_3}$ donner le résultat sous forme exponentielle puis algébrique.
|
||||||
|
\end{enumerate}
|
||||||
|
\end{exercise}
|
||||||
|
|
||||||
|
\begin{solution}
|
||||||
|
\begin{enumerate}
|
||||||
|
\item $z_1 = \frac{1}{5} - \frac{8 i}{5}$
|
||||||
|
\item $z_2 = 8 e^{- \frac{2 i \pi}{3}}$
|
||||||
|
\item $z_3 = 18 e^{\frac{i \pi}{4}}$
|
||||||
|
\item $z_4 = 144 e^{- \frac{5 i \pi}{12}} = - 36 \sqrt{2} + 36 \sqrt{6} + i \left(- 36 \sqrt{6} - 36 \sqrt{2}\right) = 37.3 - 139.0 i$
|
||||||
|
\item $z_5 = \frac{4}{9} e^{- \frac{11 i \pi}{12}} = - \frac{\sqrt{6}}{9} - \frac{\sqrt{2}}{9} + i \left(- \frac{\sqrt{6}}{9} + \frac{\sqrt{2}}{9}\right) = -0.429 - 0.115 i$
|
||||||
|
\end{enumerate}
|
||||||
|
\end{solution}
|
||||||
|
|
||||||
|
\begin{exercise}[subtitle={Sortie du congélateur}]
|
||||||
|
Marie a invité quelques amis pour le thé. Elle souhaite leur proposer ses macarons maison.
|
||||||
|
|
||||||
|
Elle les sort de son congélateur à $-18$~\degres C et les place dans une pièce à $17$~\degres C.
|
||||||
|
|
||||||
|
Au bout de 15 minutes, la température des macarons est de $2$~\degres C.
|
||||||
|
|
||||||
|
\bigskip
|
||||||
|
|
||||||
|
\textbf{Premier modèle}
|
||||||
|
|
||||||
|
\medskip
|
||||||
|
|
||||||
|
On suppose que la vitesse de décongélation est constante : chaque minute la hausse de
|
||||||
|
température des macarons est la même.
|
||||||
|
|
||||||
|
Estimer dans ce cadre la température au bout de $30$~minutes, puis au bout de $45$~minutes.
|
||||||
|
|
||||||
|
Cette modélisation est-elle pertinente?
|
||||||
|
|
||||||
|
\bigskip
|
||||||
|
|
||||||
|
\textbf{Deuxième modèle}
|
||||||
|
|
||||||
|
\medskip
|
||||||
|
|
||||||
|
On suppose maintenant que la vitesse de décongélation est proportionnelle à la différence
|
||||||
|
de température entre les macarons et l'air ambiant (il s'agit de la loi de Newton).
|
||||||
|
|
||||||
|
On désigne par $\theta$ la température des macarons à l'instant $t$, et par $\theta'$ la vitesse de décongélation.
|
||||||
|
|
||||||
|
L'unité de temps est la minute et l'unité de température le degré Celsius.
|
||||||
|
|
||||||
|
\smallskip
|
||||||
|
|
||||||
|
On négligera la diminution de température de la pièce et on admettra donc qu'il existe un
|
||||||
|
nombre réel $a$ tel que, pour $t$ positif :
|
||||||
|
|
||||||
|
\[\theta'(t) = a [\theta(t) - 17]\quad (E)\]
|
||||||
|
|
||||||
|
\medskip
|
||||||
|
|
||||||
|
\begin{enumerate}
|
||||||
|
\item Vérifier que l'équation $(E)$ a pour solutions $\theta(t) = K e^{at} + 17$ où $K$ est un nombre réel.
|
||||||
|
|
||||||
|
Donner alors, en fonction de $a$, l'ensemble des solutions de $(E)$.
|
||||||
|
\end{enumerate}
|
||||||
|
On rappelle que la température des macarons à l'instant $t = 0$ est égale à $-18$~\degres C et que, au bout de $15$~min, elle est de $2$~\degres C.
|
||||||
|
\begin{enumerate}
|
||||||
|
\setcounter{enumi}{1}
|
||||||
|
\item En utilisant la condition à $t=0$ démontrer que $K = -35$.
|
||||||
|
\item En utilisant la condition à $t=15$ démontrer que $a \approx -0.06$.
|
||||||
|
\item En déduire l'expression de la solution de l'équation différentielle puis étudier ses variations.
|
||||||
|
\item La température idéale de dégustation des macarons étant de $14$~\degres C, Marie estime que
|
||||||
|
celle-ci sera atteinte au bout de $30$~min. A-t-elle raison ? Justifier la réponse.
|
||||||
|
|
||||||
|
Sinon, combien de temps faudra-t-il attendre ?
|
||||||
|
\end{enumerate}
|
||||||
|
\end{exercise}
|
||||||
|
|
||||||
|
|
||||||
|
|
||||||
|
\end{document}
|
||||||
|
|
||||||
|
%%% Local Variables:
|
||||||
|
%%% mode: latex
|
||||||
|
%%% TeX-master: "master"
|
||||||
|
%%% End:
|
|
@ -0,0 +1,136 @@
|
||||||
|
\documentclass[a4paper,10pt]{article}
|
||||||
|
\usepackage{myXsim}
|
||||||
|
|
||||||
|
% Title Page
|
||||||
|
\title{DS8 \hfill ONAL Yakub}
|
||||||
|
\tribe{TST sti2d}
|
||||||
|
\date{\hfillÀ render pour le vendredi 9 avril à 10h au plus tard}
|
||||||
|
|
||||||
|
\xsimsetup{
|
||||||
|
solution/print = false
|
||||||
|
}
|
||||||
|
|
||||||
|
\begin{document}
|
||||||
|
\maketitle
|
||||||
|
|
||||||
|
\begin{exercise}[subtitle={Étude de fonction}]
|
||||||
|
On considère la fonction $f$ définie sur $\intOF{0}{+\infty}$ par $ f(x) = 5x^2 + - 40x + - 450\ln(x)$
|
||||||
|
\begin{enumerate}
|
||||||
|
\item Démontrer que la dérivée de $f$ est $f'(x) = \frac{10x^2 + - 40x + - 450}{x}$.
|
||||||
|
\item Étude du numérateur de $f'(x)$: $N(x) = 10x^2 - 40x - 450$
|
||||||
|
\begin{enumerate}
|
||||||
|
\item Démontrer que $x=9$ et $x=- 5$ sont deux racines de $N(x)$..
|
||||||
|
\item Proposer une forme factorisée de $N(x)$.
|
||||||
|
\item Proposer une forme factorisée de $f'(x)$.
|
||||||
|
\end{enumerate}
|
||||||
|
\item Étudier le signe de $f'$ et en déduire les variations de $f$.
|
||||||
|
\end{enumerate}
|
||||||
|
\end{exercise}
|
||||||
|
|
||||||
|
\begin{solution}
|
||||||
|
\begin{enumerate}
|
||||||
|
\item pas de correction disponible
|
||||||
|
\item
|
||||||
|
\begin{enumerate}
|
||||||
|
\item \[N(9) = 0\]
|
||||||
|
\[N(- 5) = 0\]
|
||||||
|
\item \[
|
||||||
|
N(x) = 10(x - 9)(x - - 5)
|
||||||
|
\]
|
||||||
|
\item
|
||||||
|
\[
|
||||||
|
f'(x) = \frac{10(x - 9)(x - - 5)}{x}
|
||||||
|
\]
|
||||||
|
\end{enumerate}
|
||||||
|
\item Pas de correction disponible
|
||||||
|
\end{enumerate}
|
||||||
|
\end{solution}
|
||||||
|
|
||||||
|
\begin{exercise}[subtitle={Complexes}]
|
||||||
|
\begin{enumerate}
|
||||||
|
\item Mettre le nombre complexe suivant sous forme algébrique $z_1 = \dfrac{5 + 2 i}{-10 + 2 i} $
|
||||||
|
\item Mettre le complexe suivante sous forme exponentielle $z_2 = 6 - 6 \sqrt{3} i$
|
||||||
|
\item Mettre le complexe suivante sous forme exponentielle $z_3 = 10 \sqrt{3} + 10 i$
|
||||||
|
\item Calculer le produit $z_4=z_2\times z_3$ donner le résultat sous forme exponentielle puis algébrique.
|
||||||
|
\item Calculer le quotient $z_5=\frac{z_2}{z_3}$ donner le résultat sous forme exponentielle puis algébrique.
|
||||||
|
\end{enumerate}
|
||||||
|
\end{exercise}
|
||||||
|
|
||||||
|
\begin{solution}
|
||||||
|
\begin{enumerate}
|
||||||
|
\item $z_1 = - \frac{23}{52} - \frac{15 i}{52}$
|
||||||
|
\item $z_2 = 12 e^{- \frac{i \pi}{3}}$
|
||||||
|
\item $z_3 = 20 e^{\frac{i \pi}{6}}$
|
||||||
|
\item $z_4 = 240 e^{- \frac{i \pi}{6}} = 120 \sqrt{3} - 120 i = 208.0 - 120.0 i$
|
||||||
|
\item $z_5 = \frac{3}{5} e^{- \frac{i \pi}{2}} = - \frac{3 i}{5} = - 0.6 i$
|
||||||
|
\end{enumerate}
|
||||||
|
\end{solution}
|
||||||
|
|
||||||
|
\begin{exercise}[subtitle={Sortie du congélateur}]
|
||||||
|
Marie a invité quelques amis pour le thé. Elle souhaite leur proposer ses macarons maison.
|
||||||
|
|
||||||
|
Elle les sort de son congélateur à $-18$~\degres C et les place dans une pièce à $19$~\degres C.
|
||||||
|
|
||||||
|
Au bout de 15 minutes, la température des macarons est de $-2$~\degres C.
|
||||||
|
|
||||||
|
\bigskip
|
||||||
|
|
||||||
|
\textbf{Premier modèle}
|
||||||
|
|
||||||
|
\medskip
|
||||||
|
|
||||||
|
On suppose que la vitesse de décongélation est constante : chaque minute la hausse de
|
||||||
|
température des macarons est la même.
|
||||||
|
|
||||||
|
Estimer dans ce cadre la température au bout de $30$~minutes, puis au bout de $45$~minutes.
|
||||||
|
|
||||||
|
Cette modélisation est-elle pertinente?
|
||||||
|
|
||||||
|
\bigskip
|
||||||
|
|
||||||
|
\textbf{Deuxième modèle}
|
||||||
|
|
||||||
|
\medskip
|
||||||
|
|
||||||
|
On suppose maintenant que la vitesse de décongélation est proportionnelle à la différence
|
||||||
|
de température entre les macarons et l'air ambiant (il s'agit de la loi de Newton).
|
||||||
|
|
||||||
|
On désigne par $\theta$ la température des macarons à l'instant $t$, et par $\theta'$ la vitesse de décongélation.
|
||||||
|
|
||||||
|
L'unité de temps est la minute et l'unité de température le degré Celsius.
|
||||||
|
|
||||||
|
\smallskip
|
||||||
|
|
||||||
|
On négligera la diminution de température de la pièce et on admettra donc qu'il existe un
|
||||||
|
nombre réel $a$ tel que, pour $t$ positif :
|
||||||
|
|
||||||
|
\[\theta'(t) = a [\theta(t) - 19]\quad (E)\]
|
||||||
|
|
||||||
|
\medskip
|
||||||
|
|
||||||
|
\begin{enumerate}
|
||||||
|
\item Vérifier que l'équation $(E)$ a pour solutions $\theta(t) = K e^{at} + 19$ où $K$ est un nombre réel.
|
||||||
|
|
||||||
|
Donner alors, en fonction de $a$, l'ensemble des solutions de $(E)$.
|
||||||
|
\end{enumerate}
|
||||||
|
On rappelle que la température des macarons à l'instant $t = 0$ est égale à $-18$~\degres C et que, au bout de $15$~min, elle est de $-2$~\degres C.
|
||||||
|
\begin{enumerate}
|
||||||
|
\setcounter{enumi}{1}
|
||||||
|
\item En utilisant la condition à $t=0$ démontrer que $K = -37$.
|
||||||
|
\item En utilisant la condition à $t=15$ démontrer que $a \approx -0.04$.
|
||||||
|
\item En déduire l'expression de la solution de l'équation différentielle puis étudier ses variations.
|
||||||
|
\item La température idéale de dégustation des macarons étant de $16$~\degres C, Marie estime que
|
||||||
|
celle-ci sera atteinte au bout de $30$~min. A-t-elle raison ? Justifier la réponse.
|
||||||
|
|
||||||
|
Sinon, combien de temps faudra-t-il attendre ?
|
||||||
|
\end{enumerate}
|
||||||
|
\end{exercise}
|
||||||
|
|
||||||
|
|
||||||
|
|
||||||
|
\end{document}
|
||||||
|
|
||||||
|
%%% Local Variables:
|
||||||
|
%%% mode: latex
|
||||||
|
%%% TeX-master: "master"
|
||||||
|
%%% End:
|
|
@ -0,0 +1,136 @@
|
||||||
|
\documentclass[a4paper,10pt]{article}
|
||||||
|
\usepackage{myXsim}
|
||||||
|
|
||||||
|
% Title Page
|
||||||
|
\title{DS8 \hfill RADOUAA Saleh}
|
||||||
|
\tribe{TST sti2d}
|
||||||
|
\date{\hfillÀ render pour le vendredi 9 avril à 10h au plus tard}
|
||||||
|
|
||||||
|
\xsimsetup{
|
||||||
|
solution/print = false
|
||||||
|
}
|
||||||
|
|
||||||
|
\begin{document}
|
||||||
|
\maketitle
|
||||||
|
|
||||||
|
\begin{exercise}[subtitle={Étude de fonction}]
|
||||||
|
On considère la fonction $f$ définie sur $\intOF{0}{+\infty}$ par $ f(x) = 3x^2 + - 54x + 108\ln(x)$
|
||||||
|
\begin{enumerate}
|
||||||
|
\item Démontrer que la dérivée de $f$ est $f'(x) = \frac{6x^2 + - 54x + 108}{x}$.
|
||||||
|
\item Étude du numérateur de $f'(x)$: $N(x) = 6x^2 - 54x + 108$
|
||||||
|
\begin{enumerate}
|
||||||
|
\item Démontrer que $x=3$ et $x=6$ sont deux racines de $N(x)$..
|
||||||
|
\item Proposer une forme factorisée de $N(x)$.
|
||||||
|
\item Proposer une forme factorisée de $f'(x)$.
|
||||||
|
\end{enumerate}
|
||||||
|
\item Étudier le signe de $f'$ et en déduire les variations de $f$.
|
||||||
|
\end{enumerate}
|
||||||
|
\end{exercise}
|
||||||
|
|
||||||
|
\begin{solution}
|
||||||
|
\begin{enumerate}
|
||||||
|
\item pas de correction disponible
|
||||||
|
\item
|
||||||
|
\begin{enumerate}
|
||||||
|
\item \[N(3) = 0\]
|
||||||
|
\[N(6) = 0\]
|
||||||
|
\item \[
|
||||||
|
N(x) = 6(x - 3)(x - 6)
|
||||||
|
\]
|
||||||
|
\item
|
||||||
|
\[
|
||||||
|
f'(x) = \frac{6(x - 3)(x - 6)}{x}
|
||||||
|
\]
|
||||||
|
\end{enumerate}
|
||||||
|
\item Pas de correction disponible
|
||||||
|
\end{enumerate}
|
||||||
|
\end{solution}
|
||||||
|
|
||||||
|
\begin{exercise}[subtitle={Complexes}]
|
||||||
|
\begin{enumerate}
|
||||||
|
\item Mettre le nombre complexe suivant sous forme algébrique $z_1 = \dfrac{7 + 9 i}{-3 + 7 i} $
|
||||||
|
\item Mettre le complexe suivante sous forme exponentielle $z_2 = -6 + 6 \sqrt{3} i$
|
||||||
|
\item Mettre le complexe suivante sous forme exponentielle $z_3 = 10 + 10 \sqrt{3} i$
|
||||||
|
\item Calculer le produit $z_4=z_2\times z_3$ donner le résultat sous forme exponentielle puis algébrique.
|
||||||
|
\item Calculer le quotient $z_5=\frac{z_2}{z_3}$ donner le résultat sous forme exponentielle puis algébrique.
|
||||||
|
\end{enumerate}
|
||||||
|
\end{exercise}
|
||||||
|
|
||||||
|
\begin{solution}
|
||||||
|
\begin{enumerate}
|
||||||
|
\item $z_1 = \frac{21}{29} - \frac{38 i}{29}$
|
||||||
|
\item $z_2 = 12 e^{\frac{2 i \pi}{3}}$
|
||||||
|
\item $z_3 = 20 e^{\frac{i \pi}{3}}$
|
||||||
|
\item $z_4 = 240 e^{i \pi} = -240 = -240.0$
|
||||||
|
\item $z_5 = \frac{3}{5} e^{\frac{i \pi}{3}} = \frac{3}{10} + \frac{3 \sqrt{3} i}{10} = 0.3 + 0.52 i$
|
||||||
|
\end{enumerate}
|
||||||
|
\end{solution}
|
||||||
|
|
||||||
|
\begin{exercise}[subtitle={Sortie du congélateur}]
|
||||||
|
Marie a invité quelques amis pour le thé. Elle souhaite leur proposer ses macarons maison.
|
||||||
|
|
||||||
|
Elle les sort de son congélateur à $-20$~\degres C et les place dans une pièce à $16$~\degres C.
|
||||||
|
|
||||||
|
Au bout de 15 minutes, la température des macarons est de $4$~\degres C.
|
||||||
|
|
||||||
|
\bigskip
|
||||||
|
|
||||||
|
\textbf{Premier modèle}
|
||||||
|
|
||||||
|
\medskip
|
||||||
|
|
||||||
|
On suppose que la vitesse de décongélation est constante : chaque minute la hausse de
|
||||||
|
température des macarons est la même.
|
||||||
|
|
||||||
|
Estimer dans ce cadre la température au bout de $30$~minutes, puis au bout de $45$~minutes.
|
||||||
|
|
||||||
|
Cette modélisation est-elle pertinente?
|
||||||
|
|
||||||
|
\bigskip
|
||||||
|
|
||||||
|
\textbf{Deuxième modèle}
|
||||||
|
|
||||||
|
\medskip
|
||||||
|
|
||||||
|
On suppose maintenant que la vitesse de décongélation est proportionnelle à la différence
|
||||||
|
de température entre les macarons et l'air ambiant (il s'agit de la loi de Newton).
|
||||||
|
|
||||||
|
On désigne par $\theta$ la température des macarons à l'instant $t$, et par $\theta'$ la vitesse de décongélation.
|
||||||
|
|
||||||
|
L'unité de temps est la minute et l'unité de température le degré Celsius.
|
||||||
|
|
||||||
|
\smallskip
|
||||||
|
|
||||||
|
On négligera la diminution de température de la pièce et on admettra donc qu'il existe un
|
||||||
|
nombre réel $a$ tel que, pour $t$ positif :
|
||||||
|
|
||||||
|
\[\theta'(t) = a [\theta(t) - 16]\quad (E)\]
|
||||||
|
|
||||||
|
\medskip
|
||||||
|
|
||||||
|
\begin{enumerate}
|
||||||
|
\item Vérifier que l'équation $(E)$ a pour solutions $\theta(t) = K e^{at} + 16$ où $K$ est un nombre réel.
|
||||||
|
|
||||||
|
Donner alors, en fonction de $a$, l'ensemble des solutions de $(E)$.
|
||||||
|
\end{enumerate}
|
||||||
|
On rappelle que la température des macarons à l'instant $t = 0$ est égale à $-20$~\degres C et que, au bout de $15$~min, elle est de $4$~\degres C.
|
||||||
|
\begin{enumerate}
|
||||||
|
\setcounter{enumi}{1}
|
||||||
|
\item En utilisant la condition à $t=0$ démontrer que $K = -36$.
|
||||||
|
\item En utilisant la condition à $t=15$ démontrer que $a \approx -0.07$.
|
||||||
|
\item En déduire l'expression de la solution de l'équation différentielle puis étudier ses variations.
|
||||||
|
\item La température idéale de dégustation des macarons étant de $13$~\degres C, Marie estime que
|
||||||
|
celle-ci sera atteinte au bout de $30$~min. A-t-elle raison ? Justifier la réponse.
|
||||||
|
|
||||||
|
Sinon, combien de temps faudra-t-il attendre ?
|
||||||
|
\end{enumerate}
|
||||||
|
\end{exercise}
|
||||||
|
|
||||||
|
|
||||||
|
|
||||||
|
\end{document}
|
||||||
|
|
||||||
|
%%% Local Variables:
|
||||||
|
%%% mode: latex
|
||||||
|
%%% TeX-master: "master"
|
||||||
|
%%% End:
|
|
@ -0,0 +1,136 @@
|
||||||
|
\documentclass[a4paper,10pt]{article}
|
||||||
|
\usepackage{myXsim}
|
||||||
|
|
||||||
|
% Title Page
|
||||||
|
\title{DS8 \hfill TAVERNIER Joanny}
|
||||||
|
\tribe{TST sti2d}
|
||||||
|
\date{\hfillÀ render pour le vendredi 9 avril à 10h au plus tard}
|
||||||
|
|
||||||
|
\xsimsetup{
|
||||||
|
solution/print = false
|
||||||
|
}
|
||||||
|
|
||||||
|
\begin{document}
|
||||||
|
\maketitle
|
||||||
|
|
||||||
|
\begin{exercise}[subtitle={Étude de fonction}]
|
||||||
|
On considère la fonction $f$ définie sur $\intOF{0}{+\infty}$ par $ f(x) = 4.5x^2 + - 144x + 567\ln(x)$
|
||||||
|
\begin{enumerate}
|
||||||
|
\item Démontrer que la dérivée de $f$ est $f'(x) = \frac{9x^2 + - 144x + 567}{x}$.
|
||||||
|
\item Étude du numérateur de $f'(x)$: $N(x) = 9x^2 - 144x + 567$
|
||||||
|
\begin{enumerate}
|
||||||
|
\item Démontrer que $x=9$ et $x=7$ sont deux racines de $N(x)$..
|
||||||
|
\item Proposer une forme factorisée de $N(x)$.
|
||||||
|
\item Proposer une forme factorisée de $f'(x)$.
|
||||||
|
\end{enumerate}
|
||||||
|
\item Étudier le signe de $f'$ et en déduire les variations de $f$.
|
||||||
|
\end{enumerate}
|
||||||
|
\end{exercise}
|
||||||
|
|
||||||
|
\begin{solution}
|
||||||
|
\begin{enumerate}
|
||||||
|
\item pas de correction disponible
|
||||||
|
\item
|
||||||
|
\begin{enumerate}
|
||||||
|
\item \[N(9) = 0\]
|
||||||
|
\[N(7) = 0\]
|
||||||
|
\item \[
|
||||||
|
N(x) = 9(x - 9)(x - 7)
|
||||||
|
\]
|
||||||
|
\item
|
||||||
|
\[
|
||||||
|
f'(x) = \frac{9(x - 9)(x - 7)}{x}
|
||||||
|
\]
|
||||||
|
\end{enumerate}
|
||||||
|
\item Pas de correction disponible
|
||||||
|
\end{enumerate}
|
||||||
|
\end{solution}
|
||||||
|
|
||||||
|
\begin{exercise}[subtitle={Complexes}]
|
||||||
|
\begin{enumerate}
|
||||||
|
\item Mettre le nombre complexe suivant sous forme algébrique $z_1 = \dfrac{10 + 6 i}{-4 + 5 i} $
|
||||||
|
\item Mettre le complexe suivante sous forme exponentielle $z_2 = - 8 \sqrt{3} + 8 i$
|
||||||
|
\item Mettre le complexe suivante sous forme exponentielle $z_3 = 5 - 5 \sqrt{3} i$
|
||||||
|
\item Calculer le produit $z_4=z_2\times z_3$ donner le résultat sous forme exponentielle puis algébrique.
|
||||||
|
\item Calculer le quotient $z_5=\frac{z_2}{z_3}$ donner le résultat sous forme exponentielle puis algébrique.
|
||||||
|
\end{enumerate}
|
||||||
|
\end{exercise}
|
||||||
|
|
||||||
|
\begin{solution}
|
||||||
|
\begin{enumerate}
|
||||||
|
\item $z_1 = - \frac{10}{41} - \frac{74 i}{41}$
|
||||||
|
\item $z_2 = 16 e^{\frac{5 i \pi}{6}}$
|
||||||
|
\item $z_3 = 10 e^{- \frac{i \pi}{3}}$
|
||||||
|
\item $z_4 = 160 e^{\frac{i \pi}{2}} = 160 i = 160.0 i$
|
||||||
|
\item $z_5 = \frac{8}{5} e^{\frac{7 i \pi}{6}} = - \frac{4 \sqrt{3}}{5} - \frac{4 i}{5} = -1.39 - 0.8 i$
|
||||||
|
\end{enumerate}
|
||||||
|
\end{solution}
|
||||||
|
|
||||||
|
\begin{exercise}[subtitle={Sortie du congélateur}]
|
||||||
|
Marie a invité quelques amis pour le thé. Elle souhaite leur proposer ses macarons maison.
|
||||||
|
|
||||||
|
Elle les sort de son congélateur à $-19$~\degres C et les place dans une pièce à $21$~\degres C.
|
||||||
|
|
||||||
|
Au bout de 15 minutes, la température des macarons est de $-4$~\degres C.
|
||||||
|
|
||||||
|
\bigskip
|
||||||
|
|
||||||
|
\textbf{Premier modèle}
|
||||||
|
|
||||||
|
\medskip
|
||||||
|
|
||||||
|
On suppose que la vitesse de décongélation est constante : chaque minute la hausse de
|
||||||
|
température des macarons est la même.
|
||||||
|
|
||||||
|
Estimer dans ce cadre la température au bout de $30$~minutes, puis au bout de $45$~minutes.
|
||||||
|
|
||||||
|
Cette modélisation est-elle pertinente?
|
||||||
|
|
||||||
|
\bigskip
|
||||||
|
|
||||||
|
\textbf{Deuxième modèle}
|
||||||
|
|
||||||
|
\medskip
|
||||||
|
|
||||||
|
On suppose maintenant que la vitesse de décongélation est proportionnelle à la différence
|
||||||
|
de température entre les macarons et l'air ambiant (il s'agit de la loi de Newton).
|
||||||
|
|
||||||
|
On désigne par $\theta$ la température des macarons à l'instant $t$, et par $\theta'$ la vitesse de décongélation.
|
||||||
|
|
||||||
|
L'unité de temps est la minute et l'unité de température le degré Celsius.
|
||||||
|
|
||||||
|
\smallskip
|
||||||
|
|
||||||
|
On négligera la diminution de température de la pièce et on admettra donc qu'il existe un
|
||||||
|
nombre réel $a$ tel que, pour $t$ positif :
|
||||||
|
|
||||||
|
\[\theta'(t) = a [\theta(t) - 21]\quad (E)\]
|
||||||
|
|
||||||
|
\medskip
|
||||||
|
|
||||||
|
\begin{enumerate}
|
||||||
|
\item Vérifier que l'équation $(E)$ a pour solutions $\theta(t) = K e^{at} + 21$ où $K$ est un nombre réel.
|
||||||
|
|
||||||
|
Donner alors, en fonction de $a$, l'ensemble des solutions de $(E)$.
|
||||||
|
\end{enumerate}
|
||||||
|
On rappelle que la température des macarons à l'instant $t = 0$ est égale à $-19$~\degres C et que, au bout de $15$~min, elle est de $-4$~\degres C.
|
||||||
|
\begin{enumerate}
|
||||||
|
\setcounter{enumi}{1}
|
||||||
|
\item En utilisant la condition à $t=0$ démontrer que $K = -40$.
|
||||||
|
\item En utilisant la condition à $t=15$ démontrer que $a \approx -0.03$.
|
||||||
|
\item En déduire l'expression de la solution de l'équation différentielle puis étudier ses variations.
|
||||||
|
\item La température idéale de dégustation des macarons étant de $18$~\degres C, Marie estime que
|
||||||
|
celle-ci sera atteinte au bout de $30$~min. A-t-elle raison ? Justifier la réponse.
|
||||||
|
|
||||||
|
Sinon, combien de temps faudra-t-il attendre ?
|
||||||
|
\end{enumerate}
|
||||||
|
\end{exercise}
|
||||||
|
|
||||||
|
|
||||||
|
|
||||||
|
\end{document}
|
||||||
|
|
||||||
|
%%% Local Variables:
|
||||||
|
%%% mode: latex
|
||||||
|
%%% TeX-master: "master"
|
||||||
|
%%% End:
|
|
@ -0,0 +1,136 @@
|
||||||
|
\documentclass[a4paper,10pt]{article}
|
||||||
|
\usepackage{myXsim}
|
||||||
|
|
||||||
|
% Title Page
|
||||||
|
\title{DS8 \hfill ZAHORE Zahiri}
|
||||||
|
\tribe{TST sti2d}
|
||||||
|
\date{\hfillÀ render pour le vendredi 9 avril à 10h au plus tard}
|
||||||
|
|
||||||
|
\xsimsetup{
|
||||||
|
solution/print = false
|
||||||
|
}
|
||||||
|
|
||||||
|
\begin{document}
|
||||||
|
\maketitle
|
||||||
|
|
||||||
|
\begin{exercise}[subtitle={Étude de fonction}]
|
||||||
|
On considère la fonction $f$ définie sur $\intOF{0}{+\infty}$ par $ f(x) = 4.5x^2 + - 54x + - 63\ln(x)$
|
||||||
|
\begin{enumerate}
|
||||||
|
\item Démontrer que la dérivée de $f$ est $f'(x) = \frac{9x^2 + - 54x + - 63}{x}$.
|
||||||
|
\item Étude du numérateur de $f'(x)$: $N(x) = 9x^2 - 54x - 63$
|
||||||
|
\begin{enumerate}
|
||||||
|
\item Démontrer que $x=7$ et $x=- 1$ sont deux racines de $N(x)$..
|
||||||
|
\item Proposer une forme factorisée de $N(x)$.
|
||||||
|
\item Proposer une forme factorisée de $f'(x)$.
|
||||||
|
\end{enumerate}
|
||||||
|
\item Étudier le signe de $f'$ et en déduire les variations de $f$.
|
||||||
|
\end{enumerate}
|
||||||
|
\end{exercise}
|
||||||
|
|
||||||
|
\begin{solution}
|
||||||
|
\begin{enumerate}
|
||||||
|
\item pas de correction disponible
|
||||||
|
\item
|
||||||
|
\begin{enumerate}
|
||||||
|
\item \[N(7) = 0\]
|
||||||
|
\[N(- 1) = 0\]
|
||||||
|
\item \[
|
||||||
|
N(x) = 9(x - 7)(x - - 1)
|
||||||
|
\]
|
||||||
|
\item
|
||||||
|
\[
|
||||||
|
f'(x) = \frac{9(x - 7)(x - - 1)}{x}
|
||||||
|
\]
|
||||||
|
\end{enumerate}
|
||||||
|
\item Pas de correction disponible
|
||||||
|
\end{enumerate}
|
||||||
|
\end{solution}
|
||||||
|
|
||||||
|
\begin{exercise}[subtitle={Complexes}]
|
||||||
|
\begin{enumerate}
|
||||||
|
\item Mettre le nombre complexe suivant sous forme algébrique $z_1 = \dfrac{7 + 4 i}{-4 + 8 i} $
|
||||||
|
\item Mettre le complexe suivante sous forme exponentielle $z_2 = 4 \sqrt{2} + 4 \sqrt{2} i$
|
||||||
|
\item Mettre le complexe suivante sous forme exponentielle $z_3 = - 4 \sqrt{2} - 4 \sqrt{2} i$
|
||||||
|
\item Calculer le produit $z_4=z_2\times z_3$ donner le résultat sous forme exponentielle puis algébrique.
|
||||||
|
\item Calculer le quotient $z_5=\frac{z_2}{z_3}$ donner le résultat sous forme exponentielle puis algébrique.
|
||||||
|
\end{enumerate}
|
||||||
|
\end{exercise}
|
||||||
|
|
||||||
|
\begin{solution}
|
||||||
|
\begin{enumerate}
|
||||||
|
\item $z_1 = \frac{1}{20} - \frac{9 i}{10}$
|
||||||
|
\item $z_2 = 8 e^{\frac{i \pi}{4}}$
|
||||||
|
\item $z_3 = 8 e^{- \frac{3 i \pi}{4}}$
|
||||||
|
\item $z_4 = 64 e^{- \frac{i \pi}{2}} = - 64 i = - 64.0 i$
|
||||||
|
\item $z_5 = 1 e^{i \pi} = -1 = -1.0$
|
||||||
|
\end{enumerate}
|
||||||
|
\end{solution}
|
||||||
|
|
||||||
|
\begin{exercise}[subtitle={Sortie du congélateur}]
|
||||||
|
Marie a invité quelques amis pour le thé. Elle souhaite leur proposer ses macarons maison.
|
||||||
|
|
||||||
|
Elle les sort de son congélateur à $-18$~\degres C et les place dans une pièce à $21$~\degres C.
|
||||||
|
|
||||||
|
Au bout de 15 minutes, la température des macarons est de $4$~\degres C.
|
||||||
|
|
||||||
|
\bigskip
|
||||||
|
|
||||||
|
\textbf{Premier modèle}
|
||||||
|
|
||||||
|
\medskip
|
||||||
|
|
||||||
|
On suppose que la vitesse de décongélation est constante : chaque minute la hausse de
|
||||||
|
température des macarons est la même.
|
||||||
|
|
||||||
|
Estimer dans ce cadre la température au bout de $30$~minutes, puis au bout de $45$~minutes.
|
||||||
|
|
||||||
|
Cette modélisation est-elle pertinente?
|
||||||
|
|
||||||
|
\bigskip
|
||||||
|
|
||||||
|
\textbf{Deuxième modèle}
|
||||||
|
|
||||||
|
\medskip
|
||||||
|
|
||||||
|
On suppose maintenant que la vitesse de décongélation est proportionnelle à la différence
|
||||||
|
de température entre les macarons et l'air ambiant (il s'agit de la loi de Newton).
|
||||||
|
|
||||||
|
On désigne par $\theta$ la température des macarons à l'instant $t$, et par $\theta'$ la vitesse de décongélation.
|
||||||
|
|
||||||
|
L'unité de temps est la minute et l'unité de température le degré Celsius.
|
||||||
|
|
||||||
|
\smallskip
|
||||||
|
|
||||||
|
On négligera la diminution de température de la pièce et on admettra donc qu'il existe un
|
||||||
|
nombre réel $a$ tel que, pour $t$ positif :
|
||||||
|
|
||||||
|
\[\theta'(t) = a [\theta(t) - 21]\quad (E)\]
|
||||||
|
|
||||||
|
\medskip
|
||||||
|
|
||||||
|
\begin{enumerate}
|
||||||
|
\item Vérifier que l'équation $(E)$ a pour solutions $\theta(t) = K e^{at} + 21$ où $K$ est un nombre réel.
|
||||||
|
|
||||||
|
Donner alors, en fonction de $a$, l'ensemble des solutions de $(E)$.
|
||||||
|
\end{enumerate}
|
||||||
|
On rappelle que la température des macarons à l'instant $t = 0$ est égale à $-18$~\degres C et que, au bout de $15$~min, elle est de $4$~\degres C.
|
||||||
|
\begin{enumerate}
|
||||||
|
\setcounter{enumi}{1}
|
||||||
|
\item En utilisant la condition à $t=0$ démontrer que $K = -39$.
|
||||||
|
\item En utilisant la condition à $t=15$ démontrer que $a \approx -0.06$.
|
||||||
|
\item En déduire l'expression de la solution de l'équation différentielle puis étudier ses variations.
|
||||||
|
\item La température idéale de dégustation des macarons étant de $18$~\degres C, Marie estime que
|
||||||
|
celle-ci sera atteinte au bout de $30$~min. A-t-elle raison ? Justifier la réponse.
|
||||||
|
|
||||||
|
Sinon, combien de temps faudra-t-il attendre ?
|
||||||
|
\end{enumerate}
|
||||||
|
\end{exercise}
|
||||||
|
|
||||||
|
|
||||||
|
|
||||||
|
\end{document}
|
||||||
|
|
||||||
|
%%% Local Variables:
|
||||||
|
%%% mode: latex
|
||||||
|
%%% TeX-master: "master"
|
||||||
|
%%% End:
|
Binary file not shown.
|
@ -0,0 +1,136 @@
|
||||||
|
\documentclass[a4paper,10pt]{article}
|
||||||
|
\usepackage{myXsim}
|
||||||
|
|
||||||
|
% Title Page
|
||||||
|
\title{DS8 \hfill BAHBAH Zakaria}
|
||||||
|
\tribe{TST sti2d}
|
||||||
|
\date{\hfillÀ render pour le vendredi 9 avril à 10h au plus tard}
|
||||||
|
|
||||||
|
\xsimsetup{
|
||||||
|
solution/print = true
|
||||||
|
}
|
||||||
|
|
||||||
|
\begin{document}
|
||||||
|
\maketitle
|
||||||
|
|
||||||
|
\begin{exercise}[subtitle={Étude de fonction}]
|
||||||
|
On considère la fonction $f$ définie sur $\intOF{0}{+\infty}$ par $ f(x) = 4x^2 + 72x + 160\ln(x)$
|
||||||
|
\begin{enumerate}
|
||||||
|
\item Démontrer que la dérivée de $f$ est $f'(x) = \frac{8x^2 + 72x + 160}{x}$.
|
||||||
|
\item Étude du numérateur de $f'(x)$: $N(x) = 8x^2 + 72x + 160$
|
||||||
|
\begin{enumerate}
|
||||||
|
\item Démontrer que $x=- 4$ et $x=- 5$ sont deux racines de $N(x)$..
|
||||||
|
\item Proposer une forme factorisée de $N(x)$.
|
||||||
|
\item Proposer une forme factorisée de $f'(x)$.
|
||||||
|
\end{enumerate}
|
||||||
|
\item Étudier le signe de $f'$ et en déduire les variations de $f$.
|
||||||
|
\end{enumerate}
|
||||||
|
\end{exercise}
|
||||||
|
|
||||||
|
\begin{solution}
|
||||||
|
\begin{enumerate}
|
||||||
|
\item pas de correction disponible
|
||||||
|
\item
|
||||||
|
\begin{enumerate}
|
||||||
|
\item \[N(- 4) = 0\]
|
||||||
|
\[N(- 5) = 0\]
|
||||||
|
\item \[
|
||||||
|
N(x) = 8(x - - 4)(x - - 5)
|
||||||
|
\]
|
||||||
|
\item
|
||||||
|
\[
|
||||||
|
f'(x) = \frac{8(x - - 4)(x - - 5)}{x}
|
||||||
|
\]
|
||||||
|
\end{enumerate}
|
||||||
|
\item Pas de correction disponible
|
||||||
|
\end{enumerate}
|
||||||
|
\end{solution}
|
||||||
|
|
||||||
|
\begin{exercise}[subtitle={Complexes}]
|
||||||
|
\begin{enumerate}
|
||||||
|
\item Mettre le nombre complexe suivant sous forme algébrique $z_1 = \dfrac{2 + 8 i}{-9 + 3 i} $
|
||||||
|
\item Mettre le complexe suivante sous forme exponentielle $z_2 = 10 \sqrt{3} - 10 i$
|
||||||
|
\item Mettre le complexe suivante sous forme exponentielle $z_3 = 7 + 7 \sqrt{3} i$
|
||||||
|
\item Calculer le produit $z_4=z_2\times z_3$ donner le résultat sous forme exponentielle puis algébrique.
|
||||||
|
\item Calculer le quotient $z_5=\frac{z_2}{z_3}$ donner le résultat sous forme exponentielle puis algébrique.
|
||||||
|
\end{enumerate}
|
||||||
|
\end{exercise}
|
||||||
|
|
||||||
|
\begin{solution}
|
||||||
|
\begin{enumerate}
|
||||||
|
\item $z_1 = \frac{1}{15} - \frac{13 i}{15}$
|
||||||
|
\item $z_2 = 20 e^{- \frac{i \pi}{6}}$
|
||||||
|
\item $z_3 = 14 e^{\frac{i \pi}{3}}$
|
||||||
|
\item $z_4 = 280 e^{\frac{i \pi}{6}} = 140 \sqrt{3} + 140 i = 243.0 + 140.0 i$
|
||||||
|
\item $z_5 = \frac{10}{7} e^{- \frac{i \pi}{2}} = - \frac{10 i}{7} = - 1.43 i$
|
||||||
|
\end{enumerate}
|
||||||
|
\end{solution}
|
||||||
|
|
||||||
|
\begin{exercise}[subtitle={Sortie du congélateur}]
|
||||||
|
Marie a invité quelques amis pour le thé. Elle souhaite leur proposer ses macarons maison.
|
||||||
|
|
||||||
|
Elle les sort de son congélateur à $-17$~\degres C et les place dans une pièce à $16$~\degres C.
|
||||||
|
|
||||||
|
Au bout de 15 minutes, la température des macarons est de $-3$~\degres C.
|
||||||
|
|
||||||
|
\bigskip
|
||||||
|
|
||||||
|
\textbf{Premier modèle}
|
||||||
|
|
||||||
|
\medskip
|
||||||
|
|
||||||
|
On suppose que la vitesse de décongélation est constante : chaque minute la hausse de
|
||||||
|
température des macarons est la même.
|
||||||
|
|
||||||
|
Estimer dans ce cadre la température au bout de $30$~minutes, puis au bout de $45$~minutes.
|
||||||
|
|
||||||
|
Cette modélisation est-elle pertinente?
|
||||||
|
|
||||||
|
\bigskip
|
||||||
|
|
||||||
|
\textbf{Deuxième modèle}
|
||||||
|
|
||||||
|
\medskip
|
||||||
|
|
||||||
|
On suppose maintenant que la vitesse de décongélation est proportionnelle à la différence
|
||||||
|
de température entre les macarons et l'air ambiant (il s'agit de la loi de Newton).
|
||||||
|
|
||||||
|
On désigne par $\theta$ la température des macarons à l'instant $t$, et par $\theta'$ la vitesse de décongélation.
|
||||||
|
|
||||||
|
L'unité de temps est la minute et l'unité de température le degré Celsius.
|
||||||
|
|
||||||
|
\smallskip
|
||||||
|
|
||||||
|
On négligera la diminution de température de la pièce et on admettra donc qu'il existe un
|
||||||
|
nombre réel $a$ tel que, pour $t$ positif :
|
||||||
|
|
||||||
|
\[\theta'(t) = a [\theta(t) - 16]\quad (E)\]
|
||||||
|
|
||||||
|
\medskip
|
||||||
|
|
||||||
|
\begin{enumerate}
|
||||||
|
\item Vérifier que l'équation $(E)$ a pour solutions $\theta(t) = K e^{at} + 16$ où $K$ est un nombre réel.
|
||||||
|
|
||||||
|
Donner alors, en fonction de $a$, l'ensemble des solutions de $(E)$.
|
||||||
|
\end{enumerate}
|
||||||
|
On rappelle que la température des macarons à l'instant $t = 0$ est égale à $-17$~\degres C et que, au bout de $15$~min, elle est de $-3$~\degres C.
|
||||||
|
\begin{enumerate}
|
||||||
|
\setcounter{enumi}{1}
|
||||||
|
\item En utilisant la condition à $t=0$ démontrer que $K = -33$.
|
||||||
|
\item En utilisant la condition à $t=15$ démontrer que $a \approx -0.04$.
|
||||||
|
\item En déduire l'expression de la solution de l'équation différentielle puis étudier ses variations.
|
||||||
|
\item La température idéale de dégustation des macarons étant de $13$~\degres C, Marie estime que
|
||||||
|
celle-ci sera atteinte au bout de $30$~min. A-t-elle raison ? Justifier la réponse.
|
||||||
|
|
||||||
|
Sinon, combien de temps faudra-t-il attendre ?
|
||||||
|
\end{enumerate}
|
||||||
|
\end{exercise}
|
||||||
|
|
||||||
|
|
||||||
|
|
||||||
|
\end{document}
|
||||||
|
|
||||||
|
%%% Local Variables:
|
||||||
|
%%% mode: latex
|
||||||
|
%%% TeX-master: "master"
|
||||||
|
%%% End:
|
|
@ -0,0 +1,136 @@
|
||||||
|
\documentclass[a4paper,10pt]{article}
|
||||||
|
\usepackage{myXsim}
|
||||||
|
|
||||||
|
% Title Page
|
||||||
|
\title{DS8 \hfill BENALI Ilyas}
|
||||||
|
\tribe{TST sti2d}
|
||||||
|
\date{\hfillÀ render pour le vendredi 9 avril à 10h au plus tard}
|
||||||
|
|
||||||
|
\xsimsetup{
|
||||||
|
solution/print = true
|
||||||
|
}
|
||||||
|
|
||||||
|
\begin{document}
|
||||||
|
\maketitle
|
||||||
|
|
||||||
|
\begin{exercise}[subtitle={Étude de fonction}]
|
||||||
|
On considère la fonction $f$ définie sur $\intOF{0}{+\infty}$ par $ f(x) = 2.5x^2 + - 95x + 450\ln(x)$
|
||||||
|
\begin{enumerate}
|
||||||
|
\item Démontrer que la dérivée de $f$ est $f'(x) = \frac{5x^2 + - 95x + 450}{x}$.
|
||||||
|
\item Étude du numérateur de $f'(x)$: $N(x) = 5x^2 - 95x + 450$
|
||||||
|
\begin{enumerate}
|
||||||
|
\item Démontrer que $x=9$ et $x=10$ sont deux racines de $N(x)$..
|
||||||
|
\item Proposer une forme factorisée de $N(x)$.
|
||||||
|
\item Proposer une forme factorisée de $f'(x)$.
|
||||||
|
\end{enumerate}
|
||||||
|
\item Étudier le signe de $f'$ et en déduire les variations de $f$.
|
||||||
|
\end{enumerate}
|
||||||
|
\end{exercise}
|
||||||
|
|
||||||
|
\begin{solution}
|
||||||
|
\begin{enumerate}
|
||||||
|
\item pas de correction disponible
|
||||||
|
\item
|
||||||
|
\begin{enumerate}
|
||||||
|
\item \[N(9) = 0\]
|
||||||
|
\[N(10) = 0\]
|
||||||
|
\item \[
|
||||||
|
N(x) = 5(x - 9)(x - 10)
|
||||||
|
\]
|
||||||
|
\item
|
||||||
|
\[
|
||||||
|
f'(x) = \frac{5(x - 9)(x - 10)}{x}
|
||||||
|
\]
|
||||||
|
\end{enumerate}
|
||||||
|
\item Pas de correction disponible
|
||||||
|
\end{enumerate}
|
||||||
|
\end{solution}
|
||||||
|
|
||||||
|
\begin{exercise}[subtitle={Complexes}]
|
||||||
|
\begin{enumerate}
|
||||||
|
\item Mettre le nombre complexe suivant sous forme algébrique $z_1 = \dfrac{10 + 5 i}{-3 + 10 i} $
|
||||||
|
\item Mettre le complexe suivante sous forme exponentielle $z_2 = - 5 \sqrt{3} + 5 i$
|
||||||
|
\item Mettre le complexe suivante sous forme exponentielle $z_3 = - 8 \sqrt{2} - 8 \sqrt{2} i$
|
||||||
|
\item Calculer le produit $z_4=z_2\times z_3$ donner le résultat sous forme exponentielle puis algébrique.
|
||||||
|
\item Calculer le quotient $z_5=\frac{z_2}{z_3}$ donner le résultat sous forme exponentielle puis algébrique.
|
||||||
|
\end{enumerate}
|
||||||
|
\end{exercise}
|
||||||
|
|
||||||
|
\begin{solution}
|
||||||
|
\begin{enumerate}
|
||||||
|
\item $z_1 = \frac{20}{109} - \frac{115 i}{109}$
|
||||||
|
\item $z_2 = 10 e^{\frac{5 i \pi}{6}}$
|
||||||
|
\item $z_3 = 16 e^{- \frac{3 i \pi}{4}}$
|
||||||
|
\item $z_4 = 160 e^{\frac{i \pi}{12}} = 40 \sqrt{2} + 40 \sqrt{6} + i \left(- 40 \sqrt{2} + 40 \sqrt{6}\right) = 155.0 + 41.4 i$
|
||||||
|
\item $z_5 = \frac{5}{8} e^{\frac{19 i \pi}{12}} = - \frac{5 \sqrt{2}}{32} + \frac{5 \sqrt{6}}{32} + i \left(- \frac{5 \sqrt{6}}{32} - \frac{5 \sqrt{2}}{32}\right) = 0.162 - 0.604 i$
|
||||||
|
\end{enumerate}
|
||||||
|
\end{solution}
|
||||||
|
|
||||||
|
\begin{exercise}[subtitle={Sortie du congélateur}]
|
||||||
|
Marie a invité quelques amis pour le thé. Elle souhaite leur proposer ses macarons maison.
|
||||||
|
|
||||||
|
Elle les sort de son congélateur à $-17$~\degres C et les place dans une pièce à $17$~\degres C.
|
||||||
|
|
||||||
|
Au bout de 15 minutes, la température des macarons est de $-1$~\degres C.
|
||||||
|
|
||||||
|
\bigskip
|
||||||
|
|
||||||
|
\textbf{Premier modèle}
|
||||||
|
|
||||||
|
\medskip
|
||||||
|
|
||||||
|
On suppose que la vitesse de décongélation est constante : chaque minute la hausse de
|
||||||
|
température des macarons est la même.
|
||||||
|
|
||||||
|
Estimer dans ce cadre la température au bout de $30$~minutes, puis au bout de $45$~minutes.
|
||||||
|
|
||||||
|
Cette modélisation est-elle pertinente?
|
||||||
|
|
||||||
|
\bigskip
|
||||||
|
|
||||||
|
\textbf{Deuxième modèle}
|
||||||
|
|
||||||
|
\medskip
|
||||||
|
|
||||||
|
On suppose maintenant que la vitesse de décongélation est proportionnelle à la différence
|
||||||
|
de température entre les macarons et l'air ambiant (il s'agit de la loi de Newton).
|
||||||
|
|
||||||
|
On désigne par $\theta$ la température des macarons à l'instant $t$, et par $\theta'$ la vitesse de décongélation.
|
||||||
|
|
||||||
|
L'unité de temps est la minute et l'unité de température le degré Celsius.
|
||||||
|
|
||||||
|
\smallskip
|
||||||
|
|
||||||
|
On négligera la diminution de température de la pièce et on admettra donc qu'il existe un
|
||||||
|
nombre réel $a$ tel que, pour $t$ positif :
|
||||||
|
|
||||||
|
\[\theta'(t) = a [\theta(t) - 17]\quad (E)\]
|
||||||
|
|
||||||
|
\medskip
|
||||||
|
|
||||||
|
\begin{enumerate}
|
||||||
|
\item Vérifier que l'équation $(E)$ a pour solutions $\theta(t) = K e^{at} + 17$ où $K$ est un nombre réel.
|
||||||
|
|
||||||
|
Donner alors, en fonction de $a$, l'ensemble des solutions de $(E)$.
|
||||||
|
\end{enumerate}
|
||||||
|
On rappelle que la température des macarons à l'instant $t = 0$ est égale à $-17$~\degres C et que, au bout de $15$~min, elle est de $-1$~\degres C.
|
||||||
|
\begin{enumerate}
|
||||||
|
\setcounter{enumi}{1}
|
||||||
|
\item En utilisant la condition à $t=0$ démontrer que $K = -34$.
|
||||||
|
\item En utilisant la condition à $t=15$ démontrer que $a \approx -0.04$.
|
||||||
|
\item En déduire l'expression de la solution de l'équation différentielle puis étudier ses variations.
|
||||||
|
\item La température idéale de dégustation des macarons étant de $14$~\degres C, Marie estime que
|
||||||
|
celle-ci sera atteinte au bout de $30$~min. A-t-elle raison ? Justifier la réponse.
|
||||||
|
|
||||||
|
Sinon, combien de temps faudra-t-il attendre ?
|
||||||
|
\end{enumerate}
|
||||||
|
\end{exercise}
|
||||||
|
|
||||||
|
|
||||||
|
|
||||||
|
\end{document}
|
||||||
|
|
||||||
|
%%% Local Variables:
|
||||||
|
%%% mode: latex
|
||||||
|
%%% TeX-master: "master"
|
||||||
|
%%% End:
|
|
@ -0,0 +1,136 @@
|
||||||
|
\documentclass[a4paper,10pt]{article}
|
||||||
|
\usepackage{myXsim}
|
||||||
|
|
||||||
|
% Title Page
|
||||||
|
\title{DS8 \hfill BERNADAT Noah}
|
||||||
|
\tribe{TST sti2d}
|
||||||
|
\date{\hfillÀ render pour le vendredi 9 avril à 10h au plus tard}
|
||||||
|
|
||||||
|
\xsimsetup{
|
||||||
|
solution/print = true
|
||||||
|
}
|
||||||
|
|
||||||
|
\begin{document}
|
||||||
|
\maketitle
|
||||||
|
|
||||||
|
\begin{exercise}[subtitle={Étude de fonction}]
|
||||||
|
On considère la fonction $f$ définie sur $\intOF{0}{+\infty}$ par $ f(x) = 3.5x^2 + - 21x + - 196\ln(x)$
|
||||||
|
\begin{enumerate}
|
||||||
|
\item Démontrer que la dérivée de $f$ est $f'(x) = \frac{7x^2 + - 21x + - 196}{x}$.
|
||||||
|
\item Étude du numérateur de $f'(x)$: $N(x) = 7x^2 - 21x - 196$
|
||||||
|
\begin{enumerate}
|
||||||
|
\item Démontrer que $x=- 4$ et $x=7$ sont deux racines de $N(x)$..
|
||||||
|
\item Proposer une forme factorisée de $N(x)$.
|
||||||
|
\item Proposer une forme factorisée de $f'(x)$.
|
||||||
|
\end{enumerate}
|
||||||
|
\item Étudier le signe de $f'$ et en déduire les variations de $f$.
|
||||||
|
\end{enumerate}
|
||||||
|
\end{exercise}
|
||||||
|
|
||||||
|
\begin{solution}
|
||||||
|
\begin{enumerate}
|
||||||
|
\item pas de correction disponible
|
||||||
|
\item
|
||||||
|
\begin{enumerate}
|
||||||
|
\item \[N(- 4) = 0\]
|
||||||
|
\[N(7) = 0\]
|
||||||
|
\item \[
|
||||||
|
N(x) = 7(x - - 4)(x - 7)
|
||||||
|
\]
|
||||||
|
\item
|
||||||
|
\[
|
||||||
|
f'(x) = \frac{7(x - - 4)(x - 7)}{x}
|
||||||
|
\]
|
||||||
|
\end{enumerate}
|
||||||
|
\item Pas de correction disponible
|
||||||
|
\end{enumerate}
|
||||||
|
\end{solution}
|
||||||
|
|
||||||
|
\begin{exercise}[subtitle={Complexes}]
|
||||||
|
\begin{enumerate}
|
||||||
|
\item Mettre le nombre complexe suivant sous forme algébrique $z_1 = \dfrac{8 + 9 i}{-9 + 5 i} $
|
||||||
|
\item Mettre le complexe suivante sous forme exponentielle $z_2 = - 8 \sqrt{3} + 8 i$
|
||||||
|
\item Mettre le complexe suivante sous forme exponentielle $z_3 = -5 - 5 \sqrt{3} i$
|
||||||
|
\item Calculer le produit $z_4=z_2\times z_3$ donner le résultat sous forme exponentielle puis algébrique.
|
||||||
|
\item Calculer le quotient $z_5=\frac{z_2}{z_3}$ donner le résultat sous forme exponentielle puis algébrique.
|
||||||
|
\end{enumerate}
|
||||||
|
\end{exercise}
|
||||||
|
|
||||||
|
\begin{solution}
|
||||||
|
\begin{enumerate}
|
||||||
|
\item $z_1 = - \frac{27}{106} - \frac{121 i}{106}$
|
||||||
|
\item $z_2 = 16 e^{\frac{5 i \pi}{6}}$
|
||||||
|
\item $z_3 = 10 e^{- \frac{2 i \pi}{3}}$
|
||||||
|
\item $z_4 = 160 e^{\frac{i \pi}{6}} = 80 \sqrt{3} + 80 i = 139.0 + 80.0 i$
|
||||||
|
\item $z_5 = \frac{8}{5} e^{\frac{3 i \pi}{2}} = - \frac{8 i}{5} = - 1.6 i$
|
||||||
|
\end{enumerate}
|
||||||
|
\end{solution}
|
||||||
|
|
||||||
|
\begin{exercise}[subtitle={Sortie du congélateur}]
|
||||||
|
Marie a invité quelques amis pour le thé. Elle souhaite leur proposer ses macarons maison.
|
||||||
|
|
||||||
|
Elle les sort de son congélateur à $-15$~\degres C et les place dans une pièce à $22$~\degres C.
|
||||||
|
|
||||||
|
Au bout de 15 minutes, la température des macarons est de $-1$~\degres C.
|
||||||
|
|
||||||
|
\bigskip
|
||||||
|
|
||||||
|
\textbf{Premier modèle}
|
||||||
|
|
||||||
|
\medskip
|
||||||
|
|
||||||
|
On suppose que la vitesse de décongélation est constante : chaque minute la hausse de
|
||||||
|
température des macarons est la même.
|
||||||
|
|
||||||
|
Estimer dans ce cadre la température au bout de $30$~minutes, puis au bout de $45$~minutes.
|
||||||
|
|
||||||
|
Cette modélisation est-elle pertinente?
|
||||||
|
|
||||||
|
\bigskip
|
||||||
|
|
||||||
|
\textbf{Deuxième modèle}
|
||||||
|
|
||||||
|
\medskip
|
||||||
|
|
||||||
|
On suppose maintenant que la vitesse de décongélation est proportionnelle à la différence
|
||||||
|
de température entre les macarons et l'air ambiant (il s'agit de la loi de Newton).
|
||||||
|
|
||||||
|
On désigne par $\theta$ la température des macarons à l'instant $t$, et par $\theta'$ la vitesse de décongélation.
|
||||||
|
|
||||||
|
L'unité de temps est la minute et l'unité de température le degré Celsius.
|
||||||
|
|
||||||
|
\smallskip
|
||||||
|
|
||||||
|
On négligera la diminution de température de la pièce et on admettra donc qu'il existe un
|
||||||
|
nombre réel $a$ tel que, pour $t$ positif :
|
||||||
|
|
||||||
|
\[\theta'(t) = a [\theta(t) - 22]\quad (E)\]
|
||||||
|
|
||||||
|
\medskip
|
||||||
|
|
||||||
|
\begin{enumerate}
|
||||||
|
\item Vérifier que l'équation $(E)$ a pour solutions $\theta(t) = K e^{at} + 22$ où $K$ est un nombre réel.
|
||||||
|
|
||||||
|
Donner alors, en fonction de $a$, l'ensemble des solutions de $(E)$.
|
||||||
|
\end{enumerate}
|
||||||
|
On rappelle que la température des macarons à l'instant $t = 0$ est égale à $-15$~\degres C et que, au bout de $15$~min, elle est de $-1$~\degres C.
|
||||||
|
\begin{enumerate}
|
||||||
|
\setcounter{enumi}{1}
|
||||||
|
\item En utilisant la condition à $t=0$ démontrer que $K = -37$.
|
||||||
|
\item En utilisant la condition à $t=15$ démontrer que $a \approx -0.03$.
|
||||||
|
\item En déduire l'expression de la solution de l'équation différentielle puis étudier ses variations.
|
||||||
|
\item La température idéale de dégustation des macarons étant de $19$~\degres C, Marie estime que
|
||||||
|
celle-ci sera atteinte au bout de $30$~min. A-t-elle raison ? Justifier la réponse.
|
||||||
|
|
||||||
|
Sinon, combien de temps faudra-t-il attendre ?
|
||||||
|
\end{enumerate}
|
||||||
|
\end{exercise}
|
||||||
|
|
||||||
|
|
||||||
|
|
||||||
|
\end{document}
|
||||||
|
|
||||||
|
%%% Local Variables:
|
||||||
|
%%% mode: latex
|
||||||
|
%%% TeX-master: "master"
|
||||||
|
%%% End:
|
|
@ -0,0 +1,136 @@
|
||||||
|
\documentclass[a4paper,10pt]{article}
|
||||||
|
\usepackage{myXsim}
|
||||||
|
|
||||||
|
% Title Page
|
||||||
|
\title{DS8 \hfill BUDIN Nathan}
|
||||||
|
\tribe{TST sti2d}
|
||||||
|
\date{\hfillÀ render pour le vendredi 9 avril à 10h au plus tard}
|
||||||
|
|
||||||
|
\xsimsetup{
|
||||||
|
solution/print = true
|
||||||
|
}
|
||||||
|
|
||||||
|
\begin{document}
|
||||||
|
\maketitle
|
||||||
|
|
||||||
|
\begin{exercise}[subtitle={Étude de fonction}]
|
||||||
|
On considère la fonction $f$ définie sur $\intOF{0}{+\infty}$ par $ f(x) = 3.5x^2 + - 84x + 140\ln(x)$
|
||||||
|
\begin{enumerate}
|
||||||
|
\item Démontrer que la dérivée de $f$ est $f'(x) = \frac{7x^2 + - 84x + 140}{x}$.
|
||||||
|
\item Étude du numérateur de $f'(x)$: $N(x) = 7x^2 - 84x + 140$
|
||||||
|
\begin{enumerate}
|
||||||
|
\item Démontrer que $x=10$ et $x=2$ sont deux racines de $N(x)$..
|
||||||
|
\item Proposer une forme factorisée de $N(x)$.
|
||||||
|
\item Proposer une forme factorisée de $f'(x)$.
|
||||||
|
\end{enumerate}
|
||||||
|
\item Étudier le signe de $f'$ et en déduire les variations de $f$.
|
||||||
|
\end{enumerate}
|
||||||
|
\end{exercise}
|
||||||
|
|
||||||
|
\begin{solution}
|
||||||
|
\begin{enumerate}
|
||||||
|
\item pas de correction disponible
|
||||||
|
\item
|
||||||
|
\begin{enumerate}
|
||||||
|
\item \[N(10) = 0\]
|
||||||
|
\[N(2) = 0\]
|
||||||
|
\item \[
|
||||||
|
N(x) = 7(x - 10)(x - 2)
|
||||||
|
\]
|
||||||
|
\item
|
||||||
|
\[
|
||||||
|
f'(x) = \frac{7(x - 10)(x - 2)}{x}
|
||||||
|
\]
|
||||||
|
\end{enumerate}
|
||||||
|
\item Pas de correction disponible
|
||||||
|
\end{enumerate}
|
||||||
|
\end{solution}
|
||||||
|
|
||||||
|
\begin{exercise}[subtitle={Complexes}]
|
||||||
|
\begin{enumerate}
|
||||||
|
\item Mettre le nombre complexe suivant sous forme algébrique $z_1 = \dfrac{8 + 8 i}{-7 + 4 i} $
|
||||||
|
\item Mettre le complexe suivante sous forme exponentielle $z_2 = 7 \sqrt{3} - 7 i$
|
||||||
|
\item Mettre le complexe suivante sous forme exponentielle $z_3 = - 2 \sqrt{3} + 2 i$
|
||||||
|
\item Calculer le produit $z_4=z_2\times z_3$ donner le résultat sous forme exponentielle puis algébrique.
|
||||||
|
\item Calculer le quotient $z_5=\frac{z_2}{z_3}$ donner le résultat sous forme exponentielle puis algébrique.
|
||||||
|
\end{enumerate}
|
||||||
|
\end{exercise}
|
||||||
|
|
||||||
|
\begin{solution}
|
||||||
|
\begin{enumerate}
|
||||||
|
\item $z_1 = - \frac{24}{65} - \frac{88 i}{65}$
|
||||||
|
\item $z_2 = 14 e^{- \frac{i \pi}{6}}$
|
||||||
|
\item $z_3 = 4 e^{\frac{5 i \pi}{6}}$
|
||||||
|
\item $z_4 = 56 e^{\frac{2 i \pi}{3}} = -28 + 28 \sqrt{3} i = -28.0 + 48.5 i$
|
||||||
|
\item $z_5 = \frac{7}{2} e^{- i \pi} = - \frac{7}{2} = -3.5$
|
||||||
|
\end{enumerate}
|
||||||
|
\end{solution}
|
||||||
|
|
||||||
|
\begin{exercise}[subtitle={Sortie du congélateur}]
|
||||||
|
Marie a invité quelques amis pour le thé. Elle souhaite leur proposer ses macarons maison.
|
||||||
|
|
||||||
|
Elle les sort de son congélateur à $-19$~\degres C et les place dans une pièce à $23$~\degres C.
|
||||||
|
|
||||||
|
Au bout de 15 minutes, la température des macarons est de $3$~\degres C.
|
||||||
|
|
||||||
|
\bigskip
|
||||||
|
|
||||||
|
\textbf{Premier modèle}
|
||||||
|
|
||||||
|
\medskip
|
||||||
|
|
||||||
|
On suppose que la vitesse de décongélation est constante : chaque minute la hausse de
|
||||||
|
température des macarons est la même.
|
||||||
|
|
||||||
|
Estimer dans ce cadre la température au bout de $30$~minutes, puis au bout de $45$~minutes.
|
||||||
|
|
||||||
|
Cette modélisation est-elle pertinente?
|
||||||
|
|
||||||
|
\bigskip
|
||||||
|
|
||||||
|
\textbf{Deuxième modèle}
|
||||||
|
|
||||||
|
\medskip
|
||||||
|
|
||||||
|
On suppose maintenant que la vitesse de décongélation est proportionnelle à la différence
|
||||||
|
de température entre les macarons et l'air ambiant (il s'agit de la loi de Newton).
|
||||||
|
|
||||||
|
On désigne par $\theta$ la température des macarons à l'instant $t$, et par $\theta'$ la vitesse de décongélation.
|
||||||
|
|
||||||
|
L'unité de temps est la minute et l'unité de température le degré Celsius.
|
||||||
|
|
||||||
|
\smallskip
|
||||||
|
|
||||||
|
On négligera la diminution de température de la pièce et on admettra donc qu'il existe un
|
||||||
|
nombre réel $a$ tel que, pour $t$ positif :
|
||||||
|
|
||||||
|
\[\theta'(t) = a [\theta(t) - 23]\quad (E)\]
|
||||||
|
|
||||||
|
\medskip
|
||||||
|
|
||||||
|
\begin{enumerate}
|
||||||
|
\item Vérifier que l'équation $(E)$ a pour solutions $\theta(t) = K e^{at} + 23$ où $K$ est un nombre réel.
|
||||||
|
|
||||||
|
Donner alors, en fonction de $a$, l'ensemble des solutions de $(E)$.
|
||||||
|
\end{enumerate}
|
||||||
|
On rappelle que la température des macarons à l'instant $t = 0$ est égale à $-19$~\degres C et que, au bout de $15$~min, elle est de $3$~\degres C.
|
||||||
|
\begin{enumerate}
|
||||||
|
\setcounter{enumi}{1}
|
||||||
|
\item En utilisant la condition à $t=0$ démontrer que $K = -42$.
|
||||||
|
\item En utilisant la condition à $t=15$ démontrer que $a \approx -0.05$.
|
||||||
|
\item En déduire l'expression de la solution de l'équation différentielle puis étudier ses variations.
|
||||||
|
\item La température idéale de dégustation des macarons étant de $20$~\degres C, Marie estime que
|
||||||
|
celle-ci sera atteinte au bout de $30$~min. A-t-elle raison ? Justifier la réponse.
|
||||||
|
|
||||||
|
Sinon, combien de temps faudra-t-il attendre ?
|
||||||
|
\end{enumerate}
|
||||||
|
\end{exercise}
|
||||||
|
|
||||||
|
|
||||||
|
|
||||||
|
\end{document}
|
||||||
|
|
||||||
|
%%% Local Variables:
|
||||||
|
%%% mode: latex
|
||||||
|
%%% TeX-master: "master"
|
||||||
|
%%% End:
|
|
@ -0,0 +1,136 @@
|
||||||
|
\documentclass[a4paper,10pt]{article}
|
||||||
|
\usepackage{myXsim}
|
||||||
|
|
||||||
|
% Title Page
|
||||||
|
\title{DS8 \hfill CHION Léa}
|
||||||
|
\tribe{TST sti2d}
|
||||||
|
\date{\hfillÀ render pour le vendredi 9 avril à 10h au plus tard}
|
||||||
|
|
||||||
|
\xsimsetup{
|
||||||
|
solution/print = true
|
||||||
|
}
|
||||||
|
|
||||||
|
\begin{document}
|
||||||
|
\maketitle
|
||||||
|
|
||||||
|
\begin{exercise}[subtitle={Étude de fonction}]
|
||||||
|
On considère la fonction $f$ définie sur $\intOF{0}{+\infty}$ par $ f(x) = 5x^2 + 130x + 300\ln(x)$
|
||||||
|
\begin{enumerate}
|
||||||
|
\item Démontrer que la dérivée de $f$ est $f'(x) = \frac{10x^2 + 130x + 300}{x}$.
|
||||||
|
\item Étude du numérateur de $f'(x)$: $N(x) = 10x^2 + 130x + 300$
|
||||||
|
\begin{enumerate}
|
||||||
|
\item Démontrer que $x=- 10$ et $x=- 3$ sont deux racines de $N(x)$..
|
||||||
|
\item Proposer une forme factorisée de $N(x)$.
|
||||||
|
\item Proposer une forme factorisée de $f'(x)$.
|
||||||
|
\end{enumerate}
|
||||||
|
\item Étudier le signe de $f'$ et en déduire les variations de $f$.
|
||||||
|
\end{enumerate}
|
||||||
|
\end{exercise}
|
||||||
|
|
||||||
|
\begin{solution}
|
||||||
|
\begin{enumerate}
|
||||||
|
\item pas de correction disponible
|
||||||
|
\item
|
||||||
|
\begin{enumerate}
|
||||||
|
\item \[N(- 10) = 0\]
|
||||||
|
\[N(- 3) = 0\]
|
||||||
|
\item \[
|
||||||
|
N(x) = 10(x - - 10)(x - - 3)
|
||||||
|
\]
|
||||||
|
\item
|
||||||
|
\[
|
||||||
|
f'(x) = \frac{10(x - - 10)(x - - 3)}{x}
|
||||||
|
\]
|
||||||
|
\end{enumerate}
|
||||||
|
\item Pas de correction disponible
|
||||||
|
\end{enumerate}
|
||||||
|
\end{solution}
|
||||||
|
|
||||||
|
\begin{exercise}[subtitle={Complexes}]
|
||||||
|
\begin{enumerate}
|
||||||
|
\item Mettre le nombre complexe suivant sous forme algébrique $z_1 = \dfrac{8 + 10 i}{-3 + 2 i} $
|
||||||
|
\item Mettre le complexe suivante sous forme exponentielle $z_2 = - \sqrt{2} - \sqrt{2} i$
|
||||||
|
\item Mettre le complexe suivante sous forme exponentielle $z_3 = - 10 \sqrt{2} + 10 \sqrt{2} i$
|
||||||
|
\item Calculer le produit $z_4=z_2\times z_3$ donner le résultat sous forme exponentielle puis algébrique.
|
||||||
|
\item Calculer le quotient $z_5=\frac{z_2}{z_3}$ donner le résultat sous forme exponentielle puis algébrique.
|
||||||
|
\end{enumerate}
|
||||||
|
\end{exercise}
|
||||||
|
|
||||||
|
\begin{solution}
|
||||||
|
\begin{enumerate}
|
||||||
|
\item $z_1 = - \frac{4}{13} - \frac{46 i}{13}$
|
||||||
|
\item $z_2 = 2 e^{- \frac{3 i \pi}{4}}$
|
||||||
|
\item $z_3 = 20 e^{\frac{3 i \pi}{4}}$
|
||||||
|
\item $z_4 = 40 e^{0} = 40 = 40.0$
|
||||||
|
\item $z_5 = \frac{1}{10} e^{- \frac{3 i \pi}{2}} = \frac{i}{10} = 0.1 i$
|
||||||
|
\end{enumerate}
|
||||||
|
\end{solution}
|
||||||
|
|
||||||
|
\begin{exercise}[subtitle={Sortie du congélateur}]
|
||||||
|
Marie a invité quelques amis pour le thé. Elle souhaite leur proposer ses macarons maison.
|
||||||
|
|
||||||
|
Elle les sort de son congélateur à $-20$~\degres C et les place dans une pièce à $24$~\degres C.
|
||||||
|
|
||||||
|
Au bout de 15 minutes, la température des macarons est de $-3$~\degres C.
|
||||||
|
|
||||||
|
\bigskip
|
||||||
|
|
||||||
|
\textbf{Premier modèle}
|
||||||
|
|
||||||
|
\medskip
|
||||||
|
|
||||||
|
On suppose que la vitesse de décongélation est constante : chaque minute la hausse de
|
||||||
|
température des macarons est la même.
|
||||||
|
|
||||||
|
Estimer dans ce cadre la température au bout de $30$~minutes, puis au bout de $45$~minutes.
|
||||||
|
|
||||||
|
Cette modélisation est-elle pertinente?
|
||||||
|
|
||||||
|
\bigskip
|
||||||
|
|
||||||
|
\textbf{Deuxième modèle}
|
||||||
|
|
||||||
|
\medskip
|
||||||
|
|
||||||
|
On suppose maintenant que la vitesse de décongélation est proportionnelle à la différence
|
||||||
|
de température entre les macarons et l'air ambiant (il s'agit de la loi de Newton).
|
||||||
|
|
||||||
|
On désigne par $\theta$ la température des macarons à l'instant $t$, et par $\theta'$ la vitesse de décongélation.
|
||||||
|
|
||||||
|
L'unité de temps est la minute et l'unité de température le degré Celsius.
|
||||||
|
|
||||||
|
\smallskip
|
||||||
|
|
||||||
|
On négligera la diminution de température de la pièce et on admettra donc qu'il existe un
|
||||||
|
nombre réel $a$ tel que, pour $t$ positif :
|
||||||
|
|
||||||
|
\[\theta'(t) = a [\theta(t) - 24]\quad (E)\]
|
||||||
|
|
||||||
|
\medskip
|
||||||
|
|
||||||
|
\begin{enumerate}
|
||||||
|
\item Vérifier que l'équation $(E)$ a pour solutions $\theta(t) = K e^{at} + 24$ où $K$ est un nombre réel.
|
||||||
|
|
||||||
|
Donner alors, en fonction de $a$, l'ensemble des solutions de $(E)$.
|
||||||
|
\end{enumerate}
|
||||||
|
On rappelle que la température des macarons à l'instant $t = 0$ est égale à $-20$~\degres C et que, au bout de $15$~min, elle est de $-3$~\degres C.
|
||||||
|
\begin{enumerate}
|
||||||
|
\setcounter{enumi}{1}
|
||||||
|
\item En utilisant la condition à $t=0$ démontrer que $K = -44$.
|
||||||
|
\item En utilisant la condition à $t=15$ démontrer que $a \approx -0.03$.
|
||||||
|
\item En déduire l'expression de la solution de l'équation différentielle puis étudier ses variations.
|
||||||
|
\item La température idéale de dégustation des macarons étant de $21$~\degres C, Marie estime que
|
||||||
|
celle-ci sera atteinte au bout de $30$~min. A-t-elle raison ? Justifier la réponse.
|
||||||
|
|
||||||
|
Sinon, combien de temps faudra-t-il attendre ?
|
||||||
|
\end{enumerate}
|
||||||
|
\end{exercise}
|
||||||
|
|
||||||
|
|
||||||
|
|
||||||
|
\end{document}
|
||||||
|
|
||||||
|
%%% Local Variables:
|
||||||
|
%%% mode: latex
|
||||||
|
%%% TeX-master: "master"
|
||||||
|
%%% End:
|
|
@ -0,0 +1,136 @@
|
||||||
|
\documentclass[a4paper,10pt]{article}
|
||||||
|
\usepackage{myXsim}
|
||||||
|
|
||||||
|
% Title Page
|
||||||
|
\title{DS8 \hfill CLAIN Avinash}
|
||||||
|
\tribe{TST sti2d}
|
||||||
|
\date{\hfillÀ render pour le vendredi 9 avril à 10h au plus tard}
|
||||||
|
|
||||||
|
\xsimsetup{
|
||||||
|
solution/print = true
|
||||||
|
}
|
||||||
|
|
||||||
|
\begin{document}
|
||||||
|
\maketitle
|
||||||
|
|
||||||
|
\begin{exercise}[subtitle={Étude de fonction}]
|
||||||
|
On considère la fonction $f$ définie sur $\intOF{0}{+\infty}$ par $ f(x) = 3.5x^2 + - 42x + - 280\ln(x)$
|
||||||
|
\begin{enumerate}
|
||||||
|
\item Démontrer que la dérivée de $f$ est $f'(x) = \frac{7x^2 + - 42x + - 280}{x}$.
|
||||||
|
\item Étude du numérateur de $f'(x)$: $N(x) = 7x^2 - 42x - 280$
|
||||||
|
\begin{enumerate}
|
||||||
|
\item Démontrer que $x=10$ et $x=- 4$ sont deux racines de $N(x)$..
|
||||||
|
\item Proposer une forme factorisée de $N(x)$.
|
||||||
|
\item Proposer une forme factorisée de $f'(x)$.
|
||||||
|
\end{enumerate}
|
||||||
|
\item Étudier le signe de $f'$ et en déduire les variations de $f$.
|
||||||
|
\end{enumerate}
|
||||||
|
\end{exercise}
|
||||||
|
|
||||||
|
\begin{solution}
|
||||||
|
\begin{enumerate}
|
||||||
|
\item pas de correction disponible
|
||||||
|
\item
|
||||||
|
\begin{enumerate}
|
||||||
|
\item \[N(10) = 0\]
|
||||||
|
\[N(- 4) = 0\]
|
||||||
|
\item \[
|
||||||
|
N(x) = 7(x - 10)(x - - 4)
|
||||||
|
\]
|
||||||
|
\item
|
||||||
|
\[
|
||||||
|
f'(x) = \frac{7(x - 10)(x - - 4)}{x}
|
||||||
|
\]
|
||||||
|
\end{enumerate}
|
||||||
|
\item Pas de correction disponible
|
||||||
|
\end{enumerate}
|
||||||
|
\end{solution}
|
||||||
|
|
||||||
|
\begin{exercise}[subtitle={Complexes}]
|
||||||
|
\begin{enumerate}
|
||||||
|
\item Mettre le nombre complexe suivant sous forme algébrique $z_1 = \dfrac{3 + 8 i}{-10 + 5 i} $
|
||||||
|
\item Mettre le complexe suivante sous forme exponentielle $z_2 = 1 + \sqrt{3} i$
|
||||||
|
\item Mettre le complexe suivante sous forme exponentielle $z_3 = -3 + 3 \sqrt{3} i$
|
||||||
|
\item Calculer le produit $z_4=z_2\times z_3$ donner le résultat sous forme exponentielle puis algébrique.
|
||||||
|
\item Calculer le quotient $z_5=\frac{z_2}{z_3}$ donner le résultat sous forme exponentielle puis algébrique.
|
||||||
|
\end{enumerate}
|
||||||
|
\end{exercise}
|
||||||
|
|
||||||
|
\begin{solution}
|
||||||
|
\begin{enumerate}
|
||||||
|
\item $z_1 = \frac{2}{25} - \frac{19 i}{25}$
|
||||||
|
\item $z_2 = 2 e^{\frac{i \pi}{3}}$
|
||||||
|
\item $z_3 = 6 e^{\frac{2 i \pi}{3}}$
|
||||||
|
\item $z_4 = 12 e^{i \pi} = -12 = -12.0$
|
||||||
|
\item $z_5 = \frac{1}{3} e^{- \frac{i \pi}{3}} = \frac{1}{6} - \frac{\sqrt{3} i}{6} = 0.167 - 0.289 i$
|
||||||
|
\end{enumerate}
|
||||||
|
\end{solution}
|
||||||
|
|
||||||
|
\begin{exercise}[subtitle={Sortie du congélateur}]
|
||||||
|
Marie a invité quelques amis pour le thé. Elle souhaite leur proposer ses macarons maison.
|
||||||
|
|
||||||
|
Elle les sort de son congélateur à $-20$~\degres C et les place dans une pièce à $17$~\degres C.
|
||||||
|
|
||||||
|
Au bout de 15 minutes, la température des macarons est de $4$~\degres C.
|
||||||
|
|
||||||
|
\bigskip
|
||||||
|
|
||||||
|
\textbf{Premier modèle}
|
||||||
|
|
||||||
|
\medskip
|
||||||
|
|
||||||
|
On suppose que la vitesse de décongélation est constante : chaque minute la hausse de
|
||||||
|
température des macarons est la même.
|
||||||
|
|
||||||
|
Estimer dans ce cadre la température au bout de $30$~minutes, puis au bout de $45$~minutes.
|
||||||
|
|
||||||
|
Cette modélisation est-elle pertinente?
|
||||||
|
|
||||||
|
\bigskip
|
||||||
|
|
||||||
|
\textbf{Deuxième modèle}
|
||||||
|
|
||||||
|
\medskip
|
||||||
|
|
||||||
|
On suppose maintenant que la vitesse de décongélation est proportionnelle à la différence
|
||||||
|
de température entre les macarons et l'air ambiant (il s'agit de la loi de Newton).
|
||||||
|
|
||||||
|
On désigne par $\theta$ la température des macarons à l'instant $t$, et par $\theta'$ la vitesse de décongélation.
|
||||||
|
|
||||||
|
L'unité de temps est la minute et l'unité de température le degré Celsius.
|
||||||
|
|
||||||
|
\smallskip
|
||||||
|
|
||||||
|
On négligera la diminution de température de la pièce et on admettra donc qu'il existe un
|
||||||
|
nombre réel $a$ tel que, pour $t$ positif :
|
||||||
|
|
||||||
|
\[\theta'(t) = a [\theta(t) - 17]\quad (E)\]
|
||||||
|
|
||||||
|
\medskip
|
||||||
|
|
||||||
|
\begin{enumerate}
|
||||||
|
\item Vérifier que l'équation $(E)$ a pour solutions $\theta(t) = K e^{at} + 17$ où $K$ est un nombre réel.
|
||||||
|
|
||||||
|
Donner alors, en fonction de $a$, l'ensemble des solutions de $(E)$.
|
||||||
|
\end{enumerate}
|
||||||
|
On rappelle que la température des macarons à l'instant $t = 0$ est égale à $-20$~\degres C et que, au bout de $15$~min, elle est de $4$~\degres C.
|
||||||
|
\begin{enumerate}
|
||||||
|
\setcounter{enumi}{1}
|
||||||
|
\item En utilisant la condition à $t=0$ démontrer que $K = -37$.
|
||||||
|
\item En utilisant la condition à $t=15$ démontrer que $a \approx -0.07$.
|
||||||
|
\item En déduire l'expression de la solution de l'équation différentielle puis étudier ses variations.
|
||||||
|
\item La température idéale de dégustation des macarons étant de $14$~\degres C, Marie estime que
|
||||||
|
celle-ci sera atteinte au bout de $30$~min. A-t-elle raison ? Justifier la réponse.
|
||||||
|
|
||||||
|
Sinon, combien de temps faudra-t-il attendre ?
|
||||||
|
\end{enumerate}
|
||||||
|
\end{exercise}
|
||||||
|
|
||||||
|
|
||||||
|
|
||||||
|
\end{document}
|
||||||
|
|
||||||
|
%%% Local Variables:
|
||||||
|
%%% mode: latex
|
||||||
|
%%% TeX-master: "master"
|
||||||
|
%%% End:
|
|
@ -0,0 +1,136 @@
|
||||||
|
\documentclass[a4paper,10pt]{article}
|
||||||
|
\usepackage{myXsim}
|
||||||
|
|
||||||
|
% Title Page
|
||||||
|
\title{DS8 \hfill COUBAT Alexis}
|
||||||
|
\tribe{TST sti2d}
|
||||||
|
\date{\hfillÀ render pour le vendredi 9 avril à 10h au plus tard}
|
||||||
|
|
||||||
|
\xsimsetup{
|
||||||
|
solution/print = true
|
||||||
|
}
|
||||||
|
|
||||||
|
\begin{document}
|
||||||
|
\maketitle
|
||||||
|
|
||||||
|
\begin{exercise}[subtitle={Étude de fonction}]
|
||||||
|
On considère la fonction $f$ définie sur $\intOF{0}{+\infty}$ par $ f(x) = 5x^2 + 70x + 60\ln(x)$
|
||||||
|
\begin{enumerate}
|
||||||
|
\item Démontrer que la dérivée de $f$ est $f'(x) = \frac{10x^2 + 70x + 60}{x}$.
|
||||||
|
\item Étude du numérateur de $f'(x)$: $N(x) = 10x^2 + 70x + 60$
|
||||||
|
\begin{enumerate}
|
||||||
|
\item Démontrer que $x=- 6$ et $x=- 1$ sont deux racines de $N(x)$..
|
||||||
|
\item Proposer une forme factorisée de $N(x)$.
|
||||||
|
\item Proposer une forme factorisée de $f'(x)$.
|
||||||
|
\end{enumerate}
|
||||||
|
\item Étudier le signe de $f'$ et en déduire les variations de $f$.
|
||||||
|
\end{enumerate}
|
||||||
|
\end{exercise}
|
||||||
|
|
||||||
|
\begin{solution}
|
||||||
|
\begin{enumerate}
|
||||||
|
\item pas de correction disponible
|
||||||
|
\item
|
||||||
|
\begin{enumerate}
|
||||||
|
\item \[N(- 6) = 0\]
|
||||||
|
\[N(- 1) = 0\]
|
||||||
|
\item \[
|
||||||
|
N(x) = 10(x - - 6)(x - - 1)
|
||||||
|
\]
|
||||||
|
\item
|
||||||
|
\[
|
||||||
|
f'(x) = \frac{10(x - - 6)(x - - 1)}{x}
|
||||||
|
\]
|
||||||
|
\end{enumerate}
|
||||||
|
\item Pas de correction disponible
|
||||||
|
\end{enumerate}
|
||||||
|
\end{solution}
|
||||||
|
|
||||||
|
\begin{exercise}[subtitle={Complexes}]
|
||||||
|
\begin{enumerate}
|
||||||
|
\item Mettre le nombre complexe suivant sous forme algébrique $z_1 = \dfrac{3 + 5 i}{-4 + 5 i} $
|
||||||
|
\item Mettre le complexe suivante sous forme exponentielle $z_2 = 5 + 5 \sqrt{3} i$
|
||||||
|
\item Mettre le complexe suivante sous forme exponentielle $z_3 = - 4 \sqrt{3} - 4 i$
|
||||||
|
\item Calculer le produit $z_4=z_2\times z_3$ donner le résultat sous forme exponentielle puis algébrique.
|
||||||
|
\item Calculer le quotient $z_5=\frac{z_2}{z_3}$ donner le résultat sous forme exponentielle puis algébrique.
|
||||||
|
\end{enumerate}
|
||||||
|
\end{exercise}
|
||||||
|
|
||||||
|
\begin{solution}
|
||||||
|
\begin{enumerate}
|
||||||
|
\item $z_1 = \frac{13}{41} - \frac{35 i}{41}$
|
||||||
|
\item $z_2 = 10 e^{\frac{i \pi}{3}}$
|
||||||
|
\item $z_3 = 8 e^{- \frac{5 i \pi}{6}}$
|
||||||
|
\item $z_4 = 80 e^{- \frac{i \pi}{2}} = - 80 i = - 80.0 i$
|
||||||
|
\item $z_5 = \frac{5}{4} e^{\frac{7 i \pi}{6}} = - \frac{5 \sqrt{3}}{8} - \frac{5 i}{8} = -1.08 - 0.625 i$
|
||||||
|
\end{enumerate}
|
||||||
|
\end{solution}
|
||||||
|
|
||||||
|
\begin{exercise}[subtitle={Sortie du congélateur}]
|
||||||
|
Marie a invité quelques amis pour le thé. Elle souhaite leur proposer ses macarons maison.
|
||||||
|
|
||||||
|
Elle les sort de son congélateur à $-20$~\degres C et les place dans une pièce à $21$~\degres C.
|
||||||
|
|
||||||
|
Au bout de 15 minutes, la température des macarons est de $4$~\degres C.
|
||||||
|
|
||||||
|
\bigskip
|
||||||
|
|
||||||
|
\textbf{Premier modèle}
|
||||||
|
|
||||||
|
\medskip
|
||||||
|
|
||||||
|
On suppose que la vitesse de décongélation est constante : chaque minute la hausse de
|
||||||
|
température des macarons est la même.
|
||||||
|
|
||||||
|
Estimer dans ce cadre la température au bout de $30$~minutes, puis au bout de $45$~minutes.
|
||||||
|
|
||||||
|
Cette modélisation est-elle pertinente?
|
||||||
|
|
||||||
|
\bigskip
|
||||||
|
|
||||||
|
\textbf{Deuxième modèle}
|
||||||
|
|
||||||
|
\medskip
|
||||||
|
|
||||||
|
On suppose maintenant que la vitesse de décongélation est proportionnelle à la différence
|
||||||
|
de température entre les macarons et l'air ambiant (il s'agit de la loi de Newton).
|
||||||
|
|
||||||
|
On désigne par $\theta$ la température des macarons à l'instant $t$, et par $\theta'$ la vitesse de décongélation.
|
||||||
|
|
||||||
|
L'unité de temps est la minute et l'unité de température le degré Celsius.
|
||||||
|
|
||||||
|
\smallskip
|
||||||
|
|
||||||
|
On négligera la diminution de température de la pièce et on admettra donc qu'il existe un
|
||||||
|
nombre réel $a$ tel que, pour $t$ positif :
|
||||||
|
|
||||||
|
\[\theta'(t) = a [\theta(t) - 21]\quad (E)\]
|
||||||
|
|
||||||
|
\medskip
|
||||||
|
|
||||||
|
\begin{enumerate}
|
||||||
|
\item Vérifier que l'équation $(E)$ a pour solutions $\theta(t) = K e^{at} + 21$ où $K$ est un nombre réel.
|
||||||
|
|
||||||
|
Donner alors, en fonction de $a$, l'ensemble des solutions de $(E)$.
|
||||||
|
\end{enumerate}
|
||||||
|
On rappelle que la température des macarons à l'instant $t = 0$ est égale à $-20$~\degres C et que, au bout de $15$~min, elle est de $4$~\degres C.
|
||||||
|
\begin{enumerate}
|
||||||
|
\setcounter{enumi}{1}
|
||||||
|
\item En utilisant la condition à $t=0$ démontrer que $K = -41$.
|
||||||
|
\item En utilisant la condition à $t=15$ démontrer que $a \approx -0.06$.
|
||||||
|
\item En déduire l'expression de la solution de l'équation différentielle puis étudier ses variations.
|
||||||
|
\item La température idéale de dégustation des macarons étant de $18$~\degres C, Marie estime que
|
||||||
|
celle-ci sera atteinte au bout de $30$~min. A-t-elle raison ? Justifier la réponse.
|
||||||
|
|
||||||
|
Sinon, combien de temps faudra-t-il attendre ?
|
||||||
|
\end{enumerate}
|
||||||
|
\end{exercise}
|
||||||
|
|
||||||
|
|
||||||
|
|
||||||
|
\end{document}
|
||||||
|
|
||||||
|
%%% Local Variables:
|
||||||
|
%%% mode: latex
|
||||||
|
%%% TeX-master: "master"
|
||||||
|
%%% End:
|
|
@ -0,0 +1,136 @@
|
||||||
|
\documentclass[a4paper,10pt]{article}
|
||||||
|
\usepackage{myXsim}
|
||||||
|
|
||||||
|
% Title Page
|
||||||
|
\title{DS8 \hfill EVRARD Jules}
|
||||||
|
\tribe{TST sti2d}
|
||||||
|
\date{\hfillÀ render pour le vendredi 9 avril à 10h au plus tard}
|
||||||
|
|
||||||
|
\xsimsetup{
|
||||||
|
solution/print = true
|
||||||
|
}
|
||||||
|
|
||||||
|
\begin{document}
|
||||||
|
\maketitle
|
||||||
|
|
||||||
|
\begin{exercise}[subtitle={Étude de fonction}]
|
||||||
|
On considère la fonction $f$ définie sur $\intOF{0}{+\infty}$ par $ f(x) = 5x^2 + 0x + - 360\ln(x)$
|
||||||
|
\begin{enumerate}
|
||||||
|
\item Démontrer que la dérivée de $f$ est $f'(x) = \frac{10x^2 + 0x + - 360}{x}$.
|
||||||
|
\item Étude du numérateur de $f'(x)$: $N(x) = 10x^2 - 360$
|
||||||
|
\begin{enumerate}
|
||||||
|
\item Démontrer que $x=6$ et $x=- 6$ sont deux racines de $N(x)$..
|
||||||
|
\item Proposer une forme factorisée de $N(x)$.
|
||||||
|
\item Proposer une forme factorisée de $f'(x)$.
|
||||||
|
\end{enumerate}
|
||||||
|
\item Étudier le signe de $f'$ et en déduire les variations de $f$.
|
||||||
|
\end{enumerate}
|
||||||
|
\end{exercise}
|
||||||
|
|
||||||
|
\begin{solution}
|
||||||
|
\begin{enumerate}
|
||||||
|
\item pas de correction disponible
|
||||||
|
\item
|
||||||
|
\begin{enumerate}
|
||||||
|
\item \[N(6) = 0\]
|
||||||
|
\[N(- 6) = 0\]
|
||||||
|
\item \[
|
||||||
|
N(x) = 10(x - 6)(x - - 6)
|
||||||
|
\]
|
||||||
|
\item
|
||||||
|
\[
|
||||||
|
f'(x) = \frac{10(x - 6)(x - - 6)}{x}
|
||||||
|
\]
|
||||||
|
\end{enumerate}
|
||||||
|
\item Pas de correction disponible
|
||||||
|
\end{enumerate}
|
||||||
|
\end{solution}
|
||||||
|
|
||||||
|
\begin{exercise}[subtitle={Complexes}]
|
||||||
|
\begin{enumerate}
|
||||||
|
\item Mettre le nombre complexe suivant sous forme algébrique $z_1 = \dfrac{10 + 8 i}{-5 + 7 i} $
|
||||||
|
\item Mettre le complexe suivante sous forme exponentielle $z_2 = 9 \sqrt{2} - 9 \sqrt{2} i$
|
||||||
|
\item Mettre le complexe suivante sous forme exponentielle $z_3 = 7 \sqrt{3} + 7 i$
|
||||||
|
\item Calculer le produit $z_4=z_2\times z_3$ donner le résultat sous forme exponentielle puis algébrique.
|
||||||
|
\item Calculer le quotient $z_5=\frac{z_2}{z_3}$ donner le résultat sous forme exponentielle puis algébrique.
|
||||||
|
\end{enumerate}
|
||||||
|
\end{exercise}
|
||||||
|
|
||||||
|
\begin{solution}
|
||||||
|
\begin{enumerate}
|
||||||
|
\item $z_1 = \frac{3}{37} - \frac{55 i}{37}$
|
||||||
|
\item $z_2 = 18 e^{- \frac{i \pi}{4}}$
|
||||||
|
\item $z_3 = 14 e^{\frac{i \pi}{6}}$
|
||||||
|
\item $z_4 = 252 e^{- \frac{i \pi}{12}} = 63 \sqrt{2} + 63 \sqrt{6} + i \left(- 63 \sqrt{6} + 63 \sqrt{2}\right) = 243.0 - 65.2 i$
|
||||||
|
\item $z_5 = \frac{9}{7} e^{- \frac{5 i \pi}{12}} = - \frac{9 \sqrt{2}}{28} + \frac{9 \sqrt{6}}{28} + i \left(- \frac{9 \sqrt{6}}{28} - \frac{9 \sqrt{2}}{28}\right) = 0.333 - 1.24 i$
|
||||||
|
\end{enumerate}
|
||||||
|
\end{solution}
|
||||||
|
|
||||||
|
\begin{exercise}[subtitle={Sortie du congélateur}]
|
||||||
|
Marie a invité quelques amis pour le thé. Elle souhaite leur proposer ses macarons maison.
|
||||||
|
|
||||||
|
Elle les sort de son congélateur à $-15$~\degres C et les place dans une pièce à $17$~\degres C.
|
||||||
|
|
||||||
|
Au bout de 15 minutes, la température des macarons est de $-1$~\degres C.
|
||||||
|
|
||||||
|
\bigskip
|
||||||
|
|
||||||
|
\textbf{Premier modèle}
|
||||||
|
|
||||||
|
\medskip
|
||||||
|
|
||||||
|
On suppose que la vitesse de décongélation est constante : chaque minute la hausse de
|
||||||
|
température des macarons est la même.
|
||||||
|
|
||||||
|
Estimer dans ce cadre la température au bout de $30$~minutes, puis au bout de $45$~minutes.
|
||||||
|
|
||||||
|
Cette modélisation est-elle pertinente?
|
||||||
|
|
||||||
|
\bigskip
|
||||||
|
|
||||||
|
\textbf{Deuxième modèle}
|
||||||
|
|
||||||
|
\medskip
|
||||||
|
|
||||||
|
On suppose maintenant que la vitesse de décongélation est proportionnelle à la différence
|
||||||
|
de température entre les macarons et l'air ambiant (il s'agit de la loi de Newton).
|
||||||
|
|
||||||
|
On désigne par $\theta$ la température des macarons à l'instant $t$, et par $\theta'$ la vitesse de décongélation.
|
||||||
|
|
||||||
|
L'unité de temps est la minute et l'unité de température le degré Celsius.
|
||||||
|
|
||||||
|
\smallskip
|
||||||
|
|
||||||
|
On négligera la diminution de température de la pièce et on admettra donc qu'il existe un
|
||||||
|
nombre réel $a$ tel que, pour $t$ positif :
|
||||||
|
|
||||||
|
\[\theta'(t) = a [\theta(t) - 17]\quad (E)\]
|
||||||
|
|
||||||
|
\medskip
|
||||||
|
|
||||||
|
\begin{enumerate}
|
||||||
|
\item Vérifier que l'équation $(E)$ a pour solutions $\theta(t) = K e^{at} + 17$ où $K$ est un nombre réel.
|
||||||
|
|
||||||
|
Donner alors, en fonction de $a$, l'ensemble des solutions de $(E)$.
|
||||||
|
\end{enumerate}
|
||||||
|
On rappelle que la température des macarons à l'instant $t = 0$ est égale à $-15$~\degres C et que, au bout de $15$~min, elle est de $-1$~\degres C.
|
||||||
|
\begin{enumerate}
|
||||||
|
\setcounter{enumi}{1}
|
||||||
|
\item En utilisant la condition à $t=0$ démontrer que $K = -32$.
|
||||||
|
\item En utilisant la condition à $t=15$ démontrer que $a \approx -0.04$.
|
||||||
|
\item En déduire l'expression de la solution de l'équation différentielle puis étudier ses variations.
|
||||||
|
\item La température idéale de dégustation des macarons étant de $14$~\degres C, Marie estime que
|
||||||
|
celle-ci sera atteinte au bout de $30$~min. A-t-elle raison ? Justifier la réponse.
|
||||||
|
|
||||||
|
Sinon, combien de temps faudra-t-il attendre ?
|
||||||
|
\end{enumerate}
|
||||||
|
\end{exercise}
|
||||||
|
|
||||||
|
|
||||||
|
|
||||||
|
\end{document}
|
||||||
|
|
||||||
|
%%% Local Variables:
|
||||||
|
%%% mode: latex
|
||||||
|
%%% TeX-master: "master"
|
||||||
|
%%% End:
|
|
@ -0,0 +1,136 @@
|
||||||
|
\documentclass[a4paper,10pt]{article}
|
||||||
|
\usepackage{myXsim}
|
||||||
|
|
||||||
|
% Title Page
|
||||||
|
\title{DS8 \hfill HADJRAS Mohcine}
|
||||||
|
\tribe{TST sti2d}
|
||||||
|
\date{\hfillÀ render pour le vendredi 9 avril à 10h au plus tard}
|
||||||
|
|
||||||
|
\xsimsetup{
|
||||||
|
solution/print = true
|
||||||
|
}
|
||||||
|
|
||||||
|
\begin{document}
|
||||||
|
\maketitle
|
||||||
|
|
||||||
|
\begin{exercise}[subtitle={Étude de fonction}]
|
||||||
|
On considère la fonction $f$ définie sur $\intOF{0}{+\infty}$ par $ f(x) = 3x^2 + 78x + 240\ln(x)$
|
||||||
|
\begin{enumerate}
|
||||||
|
\item Démontrer que la dérivée de $f$ est $f'(x) = \frac{6x^2 + 78x + 240}{x}$.
|
||||||
|
\item Étude du numérateur de $f'(x)$: $N(x) = 6x^2 + 78x + 240$
|
||||||
|
\begin{enumerate}
|
||||||
|
\item Démontrer que $x=- 5$ et $x=- 8$ sont deux racines de $N(x)$..
|
||||||
|
\item Proposer une forme factorisée de $N(x)$.
|
||||||
|
\item Proposer une forme factorisée de $f'(x)$.
|
||||||
|
\end{enumerate}
|
||||||
|
\item Étudier le signe de $f'$ et en déduire les variations de $f$.
|
||||||
|
\end{enumerate}
|
||||||
|
\end{exercise}
|
||||||
|
|
||||||
|
\begin{solution}
|
||||||
|
\begin{enumerate}
|
||||||
|
\item pas de correction disponible
|
||||||
|
\item
|
||||||
|
\begin{enumerate}
|
||||||
|
\item \[N(- 5) = 0\]
|
||||||
|
\[N(- 8) = 0\]
|
||||||
|
\item \[
|
||||||
|
N(x) = 6(x - - 5)(x - - 8)
|
||||||
|
\]
|
||||||
|
\item
|
||||||
|
\[
|
||||||
|
f'(x) = \frac{6(x - - 5)(x - - 8)}{x}
|
||||||
|
\]
|
||||||
|
\end{enumerate}
|
||||||
|
\item Pas de correction disponible
|
||||||
|
\end{enumerate}
|
||||||
|
\end{solution}
|
||||||
|
|
||||||
|
\begin{exercise}[subtitle={Complexes}]
|
||||||
|
\begin{enumerate}
|
||||||
|
\item Mettre le nombre complexe suivant sous forme algébrique $z_1 = \dfrac{10 + 5 i}{-6 + 4 i} $
|
||||||
|
\item Mettre le complexe suivante sous forme exponentielle $z_2 = 1 + \sqrt{3} i$
|
||||||
|
\item Mettre le complexe suivante sous forme exponentielle $z_3 = -7 - 7 \sqrt{3} i$
|
||||||
|
\item Calculer le produit $z_4=z_2\times z_3$ donner le résultat sous forme exponentielle puis algébrique.
|
||||||
|
\item Calculer le quotient $z_5=\frac{z_2}{z_3}$ donner le résultat sous forme exponentielle puis algébrique.
|
||||||
|
\end{enumerate}
|
||||||
|
\end{exercise}
|
||||||
|
|
||||||
|
\begin{solution}
|
||||||
|
\begin{enumerate}
|
||||||
|
\item $z_1 = - \frac{10}{13} - \frac{35 i}{26}$
|
||||||
|
\item $z_2 = 2 e^{\frac{i \pi}{3}}$
|
||||||
|
\item $z_3 = 14 e^{- \frac{2 i \pi}{3}}$
|
||||||
|
\item $z_4 = 28 e^{- \frac{i \pi}{3}} = 14 - 14 \sqrt{3} i = 14.0 - 24.3 i$
|
||||||
|
\item $z_5 = \frac{1}{7} e^{i \pi} = - \frac{1}{7} = -0.143$
|
||||||
|
\end{enumerate}
|
||||||
|
\end{solution}
|
||||||
|
|
||||||
|
\begin{exercise}[subtitle={Sortie du congélateur}]
|
||||||
|
Marie a invité quelques amis pour le thé. Elle souhaite leur proposer ses macarons maison.
|
||||||
|
|
||||||
|
Elle les sort de son congélateur à $-18$~\degres C et les place dans une pièce à $25$~\degres C.
|
||||||
|
|
||||||
|
Au bout de 15 minutes, la température des macarons est de $4$~\degres C.
|
||||||
|
|
||||||
|
\bigskip
|
||||||
|
|
||||||
|
\textbf{Premier modèle}
|
||||||
|
|
||||||
|
\medskip
|
||||||
|
|
||||||
|
On suppose que la vitesse de décongélation est constante : chaque minute la hausse de
|
||||||
|
température des macarons est la même.
|
||||||
|
|
||||||
|
Estimer dans ce cadre la température au bout de $30$~minutes, puis au bout de $45$~minutes.
|
||||||
|
|
||||||
|
Cette modélisation est-elle pertinente?
|
||||||
|
|
||||||
|
\bigskip
|
||||||
|
|
||||||
|
\textbf{Deuxième modèle}
|
||||||
|
|
||||||
|
\medskip
|
||||||
|
|
||||||
|
On suppose maintenant que la vitesse de décongélation est proportionnelle à la différence
|
||||||
|
de température entre les macarons et l'air ambiant (il s'agit de la loi de Newton).
|
||||||
|
|
||||||
|
On désigne par $\theta$ la température des macarons à l'instant $t$, et par $\theta'$ la vitesse de décongélation.
|
||||||
|
|
||||||
|
L'unité de temps est la minute et l'unité de température le degré Celsius.
|
||||||
|
|
||||||
|
\smallskip
|
||||||
|
|
||||||
|
On négligera la diminution de température de la pièce et on admettra donc qu'il existe un
|
||||||
|
nombre réel $a$ tel que, pour $t$ positif :
|
||||||
|
|
||||||
|
\[\theta'(t) = a [\theta(t) - 25]\quad (E)\]
|
||||||
|
|
||||||
|
\medskip
|
||||||
|
|
||||||
|
\begin{enumerate}
|
||||||
|
\item Vérifier que l'équation $(E)$ a pour solutions $\theta(t) = K e^{at} + 25$ où $K$ est un nombre réel.
|
||||||
|
|
||||||
|
Donner alors, en fonction de $a$, l'ensemble des solutions de $(E)$.
|
||||||
|
\end{enumerate}
|
||||||
|
On rappelle que la température des macarons à l'instant $t = 0$ est égale à $-18$~\degres C et que, au bout de $15$~min, elle est de $4$~\degres C.
|
||||||
|
\begin{enumerate}
|
||||||
|
\setcounter{enumi}{1}
|
||||||
|
\item En utilisant la condition à $t=0$ démontrer que $K = -43$.
|
||||||
|
\item En utilisant la condition à $t=15$ démontrer que $a \approx -0.05$.
|
||||||
|
\item En déduire l'expression de la solution de l'équation différentielle puis étudier ses variations.
|
||||||
|
\item La température idéale de dégustation des macarons étant de $22$~\degres C, Marie estime que
|
||||||
|
celle-ci sera atteinte au bout de $30$~min. A-t-elle raison ? Justifier la réponse.
|
||||||
|
|
||||||
|
Sinon, combien de temps faudra-t-il attendre ?
|
||||||
|
\end{enumerate}
|
||||||
|
\end{exercise}
|
||||||
|
|
||||||
|
|
||||||
|
|
||||||
|
\end{document}
|
||||||
|
|
||||||
|
%%% Local Variables:
|
||||||
|
%%% mode: latex
|
||||||
|
%%% TeX-master: "master"
|
||||||
|
%%% End:
|
|
@ -0,0 +1,136 @@
|
||||||
|
\documentclass[a4paper,10pt]{article}
|
||||||
|
\usepackage{myXsim}
|
||||||
|
|
||||||
|
% Title Page
|
||||||
|
\title{DS8 \hfill HENRIST Maxime}
|
||||||
|
\tribe{TST sti2d}
|
||||||
|
\date{\hfillÀ render pour le vendredi 9 avril à 10h au plus tard}
|
||||||
|
|
||||||
|
\xsimsetup{
|
||||||
|
solution/print = true
|
||||||
|
}
|
||||||
|
|
||||||
|
\begin{document}
|
||||||
|
\maketitle
|
||||||
|
|
||||||
|
\begin{exercise}[subtitle={Étude de fonction}]
|
||||||
|
On considère la fonction $f$ définie sur $\intOF{0}{+\infty}$ par $ f(x) = 4.5x^2 + - 27x + - 36\ln(x)$
|
||||||
|
\begin{enumerate}
|
||||||
|
\item Démontrer que la dérivée de $f$ est $f'(x) = \frac{9x^2 + - 27x + - 36}{x}$.
|
||||||
|
\item Étude du numérateur de $f'(x)$: $N(x) = 9x^2 - 27x - 36$
|
||||||
|
\begin{enumerate}
|
||||||
|
\item Démontrer que $x=4$ et $x=- 1$ sont deux racines de $N(x)$..
|
||||||
|
\item Proposer une forme factorisée de $N(x)$.
|
||||||
|
\item Proposer une forme factorisée de $f'(x)$.
|
||||||
|
\end{enumerate}
|
||||||
|
\item Étudier le signe de $f'$ et en déduire les variations de $f$.
|
||||||
|
\end{enumerate}
|
||||||
|
\end{exercise}
|
||||||
|
|
||||||
|
\begin{solution}
|
||||||
|
\begin{enumerate}
|
||||||
|
\item pas de correction disponible
|
||||||
|
\item
|
||||||
|
\begin{enumerate}
|
||||||
|
\item \[N(4) = 0\]
|
||||||
|
\[N(- 1) = 0\]
|
||||||
|
\item \[
|
||||||
|
N(x) = 9(x - 4)(x - - 1)
|
||||||
|
\]
|
||||||
|
\item
|
||||||
|
\[
|
||||||
|
f'(x) = \frac{9(x - 4)(x - - 1)}{x}
|
||||||
|
\]
|
||||||
|
\end{enumerate}
|
||||||
|
\item Pas de correction disponible
|
||||||
|
\end{enumerate}
|
||||||
|
\end{solution}
|
||||||
|
|
||||||
|
\begin{exercise}[subtitle={Complexes}]
|
||||||
|
\begin{enumerate}
|
||||||
|
\item Mettre le nombre complexe suivant sous forme algébrique $z_1 = \dfrac{6 + 5 i}{-6 + 5 i} $
|
||||||
|
\item Mettre le complexe suivante sous forme exponentielle $z_2 = 8 \sqrt{3} - 8 i$
|
||||||
|
\item Mettre le complexe suivante sous forme exponentielle $z_3 = 5 - 5 \sqrt{3} i$
|
||||||
|
\item Calculer le produit $z_4=z_2\times z_3$ donner le résultat sous forme exponentielle puis algébrique.
|
||||||
|
\item Calculer le quotient $z_5=\frac{z_2}{z_3}$ donner le résultat sous forme exponentielle puis algébrique.
|
||||||
|
\end{enumerate}
|
||||||
|
\end{exercise}
|
||||||
|
|
||||||
|
\begin{solution}
|
||||||
|
\begin{enumerate}
|
||||||
|
\item $z_1 = - \frac{11}{61} - \frac{60 i}{61}$
|
||||||
|
\item $z_2 = 16 e^{- \frac{i \pi}{6}}$
|
||||||
|
\item $z_3 = 10 e^{- \frac{i \pi}{3}}$
|
||||||
|
\item $z_4 = 160 e^{- \frac{i \pi}{2}} = - 160 i = - 160.0 i$
|
||||||
|
\item $z_5 = \frac{8}{5} e^{\frac{i \pi}{6}} = \frac{4 \sqrt{3}}{5} + \frac{4 i}{5} = 1.39 + 0.8 i$
|
||||||
|
\end{enumerate}
|
||||||
|
\end{solution}
|
||||||
|
|
||||||
|
\begin{exercise}[subtitle={Sortie du congélateur}]
|
||||||
|
Marie a invité quelques amis pour le thé. Elle souhaite leur proposer ses macarons maison.
|
||||||
|
|
||||||
|
Elle les sort de son congélateur à $-16$~\degres C et les place dans une pièce à $23$~\degres C.
|
||||||
|
|
||||||
|
Au bout de 15 minutes, la température des macarons est de $0$~\degres C.
|
||||||
|
|
||||||
|
\bigskip
|
||||||
|
|
||||||
|
\textbf{Premier modèle}
|
||||||
|
|
||||||
|
\medskip
|
||||||
|
|
||||||
|
On suppose que la vitesse de décongélation est constante : chaque minute la hausse de
|
||||||
|
température des macarons est la même.
|
||||||
|
|
||||||
|
Estimer dans ce cadre la température au bout de $30$~minutes, puis au bout de $45$~minutes.
|
||||||
|
|
||||||
|
Cette modélisation est-elle pertinente?
|
||||||
|
|
||||||
|
\bigskip
|
||||||
|
|
||||||
|
\textbf{Deuxième modèle}
|
||||||
|
|
||||||
|
\medskip
|
||||||
|
|
||||||
|
On suppose maintenant que la vitesse de décongélation est proportionnelle à la différence
|
||||||
|
de température entre les macarons et l'air ambiant (il s'agit de la loi de Newton).
|
||||||
|
|
||||||
|
On désigne par $\theta$ la température des macarons à l'instant $t$, et par $\theta'$ la vitesse de décongélation.
|
||||||
|
|
||||||
|
L'unité de temps est la minute et l'unité de température le degré Celsius.
|
||||||
|
|
||||||
|
\smallskip
|
||||||
|
|
||||||
|
On négligera la diminution de température de la pièce et on admettra donc qu'il existe un
|
||||||
|
nombre réel $a$ tel que, pour $t$ positif :
|
||||||
|
|
||||||
|
\[\theta'(t) = a [\theta(t) - 23]\quad (E)\]
|
||||||
|
|
||||||
|
\medskip
|
||||||
|
|
||||||
|
\begin{enumerate}
|
||||||
|
\item Vérifier que l'équation $(E)$ a pour solutions $\theta(t) = K e^{at} + 23$ où $K$ est un nombre réel.
|
||||||
|
|
||||||
|
Donner alors, en fonction de $a$, l'ensemble des solutions de $(E)$.
|
||||||
|
\end{enumerate}
|
||||||
|
On rappelle que la température des macarons à l'instant $t = 0$ est égale à $-16$~\degres C et que, au bout de $15$~min, elle est de $0$~\degres C.
|
||||||
|
\begin{enumerate}
|
||||||
|
\setcounter{enumi}{1}
|
||||||
|
\item En utilisant la condition à $t=0$ démontrer que $K = -39$.
|
||||||
|
\item En utilisant la condition à $t=15$ démontrer que $a \approx -0.04$.
|
||||||
|
\item En déduire l'expression de la solution de l'équation différentielle puis étudier ses variations.
|
||||||
|
\item La température idéale de dégustation des macarons étant de $20$~\degres C, Marie estime que
|
||||||
|
celle-ci sera atteinte au bout de $30$~min. A-t-elle raison ? Justifier la réponse.
|
||||||
|
|
||||||
|
Sinon, combien de temps faudra-t-il attendre ?
|
||||||
|
\end{enumerate}
|
||||||
|
\end{exercise}
|
||||||
|
|
||||||
|
|
||||||
|
|
||||||
|
\end{document}
|
||||||
|
|
||||||
|
%%% Local Variables:
|
||||||
|
%%% mode: latex
|
||||||
|
%%% TeX-master: "master"
|
||||||
|
%%% End:
|
|
@ -0,0 +1,136 @@
|
||||||
|
\documentclass[a4paper,10pt]{article}
|
||||||
|
\usepackage{myXsim}
|
||||||
|
|
||||||
|
% Title Page
|
||||||
|
\title{DS8 \hfill HUMBERT Rayan}
|
||||||
|
\tribe{TST sti2d}
|
||||||
|
\date{\hfillÀ render pour le vendredi 9 avril à 10h au plus tard}
|
||||||
|
|
||||||
|
\xsimsetup{
|
||||||
|
solution/print = true
|
||||||
|
}
|
||||||
|
|
||||||
|
\begin{document}
|
||||||
|
\maketitle
|
||||||
|
|
||||||
|
\begin{exercise}[subtitle={Étude de fonction}]
|
||||||
|
On considère la fonction $f$ définie sur $\intOF{0}{+\infty}$ par $ f(x) = 2.5x^2 + - 25x + - 180\ln(x)$
|
||||||
|
\begin{enumerate}
|
||||||
|
\item Démontrer que la dérivée de $f$ est $f'(x) = \frac{5x^2 + - 25x + - 180}{x}$.
|
||||||
|
\item Étude du numérateur de $f'(x)$: $N(x) = 5x^2 - 25x - 180$
|
||||||
|
\begin{enumerate}
|
||||||
|
\item Démontrer que $x=- 4$ et $x=9$ sont deux racines de $N(x)$..
|
||||||
|
\item Proposer une forme factorisée de $N(x)$.
|
||||||
|
\item Proposer une forme factorisée de $f'(x)$.
|
||||||
|
\end{enumerate}
|
||||||
|
\item Étudier le signe de $f'$ et en déduire les variations de $f$.
|
||||||
|
\end{enumerate}
|
||||||
|
\end{exercise}
|
||||||
|
|
||||||
|
\begin{solution}
|
||||||
|
\begin{enumerate}
|
||||||
|
\item pas de correction disponible
|
||||||
|
\item
|
||||||
|
\begin{enumerate}
|
||||||
|
\item \[N(- 4) = 0\]
|
||||||
|
\[N(9) = 0\]
|
||||||
|
\item \[
|
||||||
|
N(x) = 5(x - - 4)(x - 9)
|
||||||
|
\]
|
||||||
|
\item
|
||||||
|
\[
|
||||||
|
f'(x) = \frac{5(x - - 4)(x - 9)}{x}
|
||||||
|
\]
|
||||||
|
\end{enumerate}
|
||||||
|
\item Pas de correction disponible
|
||||||
|
\end{enumerate}
|
||||||
|
\end{solution}
|
||||||
|
|
||||||
|
\begin{exercise}[subtitle={Complexes}]
|
||||||
|
\begin{enumerate}
|
||||||
|
\item Mettre le nombre complexe suivant sous forme algébrique $z_1 = \dfrac{8 + 3 i}{-6 + 6 i} $
|
||||||
|
\item Mettre le complexe suivante sous forme exponentielle $z_2 = -4 - 4 \sqrt{3} i$
|
||||||
|
\item Mettre le complexe suivante sous forme exponentielle $z_3 = 8 \sqrt{3} + 8 i$
|
||||||
|
\item Calculer le produit $z_4=z_2\times z_3$ donner le résultat sous forme exponentielle puis algébrique.
|
||||||
|
\item Calculer le quotient $z_5=\frac{z_2}{z_3}$ donner le résultat sous forme exponentielle puis algébrique.
|
||||||
|
\end{enumerate}
|
||||||
|
\end{exercise}
|
||||||
|
|
||||||
|
\begin{solution}
|
||||||
|
\begin{enumerate}
|
||||||
|
\item $z_1 = - \frac{5}{12} - \frac{11 i}{12}$
|
||||||
|
\item $z_2 = 8 e^{- \frac{2 i \pi}{3}}$
|
||||||
|
\item $z_3 = 16 e^{\frac{i \pi}{6}}$
|
||||||
|
\item $z_4 = 128 e^{- \frac{i \pi}{2}} = - 128 i = - 128.0 i$
|
||||||
|
\item $z_5 = \frac{1}{2} e^{- \frac{5 i \pi}{6}} = - \frac{\sqrt{3}}{4} - \frac{i}{4} = -0.433 - 0.25 i$
|
||||||
|
\end{enumerate}
|
||||||
|
\end{solution}
|
||||||
|
|
||||||
|
\begin{exercise}[subtitle={Sortie du congélateur}]
|
||||||
|
Marie a invité quelques amis pour le thé. Elle souhaite leur proposer ses macarons maison.
|
||||||
|
|
||||||
|
Elle les sort de son congélateur à $-20$~\degres C et les place dans une pièce à $25$~\degres C.
|
||||||
|
|
||||||
|
Au bout de 15 minutes, la température des macarons est de $-3$~\degres C.
|
||||||
|
|
||||||
|
\bigskip
|
||||||
|
|
||||||
|
\textbf{Premier modèle}
|
||||||
|
|
||||||
|
\medskip
|
||||||
|
|
||||||
|
On suppose que la vitesse de décongélation est constante : chaque minute la hausse de
|
||||||
|
température des macarons est la même.
|
||||||
|
|
||||||
|
Estimer dans ce cadre la température au bout de $30$~minutes, puis au bout de $45$~minutes.
|
||||||
|
|
||||||
|
Cette modélisation est-elle pertinente?
|
||||||
|
|
||||||
|
\bigskip
|
||||||
|
|
||||||
|
\textbf{Deuxième modèle}
|
||||||
|
|
||||||
|
\medskip
|
||||||
|
|
||||||
|
On suppose maintenant que la vitesse de décongélation est proportionnelle à la différence
|
||||||
|
de température entre les macarons et l'air ambiant (il s'agit de la loi de Newton).
|
||||||
|
|
||||||
|
On désigne par $\theta$ la température des macarons à l'instant $t$, et par $\theta'$ la vitesse de décongélation.
|
||||||
|
|
||||||
|
L'unité de temps est la minute et l'unité de température le degré Celsius.
|
||||||
|
|
||||||
|
\smallskip
|
||||||
|
|
||||||
|
On négligera la diminution de température de la pièce et on admettra donc qu'il existe un
|
||||||
|
nombre réel $a$ tel que, pour $t$ positif :
|
||||||
|
|
||||||
|
\[\theta'(t) = a [\theta(t) - 25]\quad (E)\]
|
||||||
|
|
||||||
|
\medskip
|
||||||
|
|
||||||
|
\begin{enumerate}
|
||||||
|
\item Vérifier que l'équation $(E)$ a pour solutions $\theta(t) = K e^{at} + 25$ où $K$ est un nombre réel.
|
||||||
|
|
||||||
|
Donner alors, en fonction de $a$, l'ensemble des solutions de $(E)$.
|
||||||
|
\end{enumerate}
|
||||||
|
On rappelle que la température des macarons à l'instant $t = 0$ est égale à $-20$~\degres C et que, au bout de $15$~min, elle est de $-3$~\degres C.
|
||||||
|
\begin{enumerate}
|
||||||
|
\setcounter{enumi}{1}
|
||||||
|
\item En utilisant la condition à $t=0$ démontrer que $K = -45$.
|
||||||
|
\item En utilisant la condition à $t=15$ démontrer que $a \approx -0.03$.
|
||||||
|
\item En déduire l'expression de la solution de l'équation différentielle puis étudier ses variations.
|
||||||
|
\item La température idéale de dégustation des macarons étant de $22$~\degres C, Marie estime que
|
||||||
|
celle-ci sera atteinte au bout de $30$~min. A-t-elle raison ? Justifier la réponse.
|
||||||
|
|
||||||
|
Sinon, combien de temps faudra-t-il attendre ?
|
||||||
|
\end{enumerate}
|
||||||
|
\end{exercise}
|
||||||
|
|
||||||
|
|
||||||
|
|
||||||
|
\end{document}
|
||||||
|
|
||||||
|
%%% Local Variables:
|
||||||
|
%%% mode: latex
|
||||||
|
%%% TeX-master: "master"
|
||||||
|
%%% End:
|
|
@ -0,0 +1,136 @@
|
||||||
|
\documentclass[a4paper,10pt]{article}
|
||||||
|
\usepackage{myXsim}
|
||||||
|
|
||||||
|
% Title Page
|
||||||
|
\title{DS8 \hfill KILINC Suleyman}
|
||||||
|
\tribe{TST sti2d}
|
||||||
|
\date{\hfillÀ render pour le vendredi 9 avril à 10h au plus tard}
|
||||||
|
|
||||||
|
\xsimsetup{
|
||||||
|
solution/print = true
|
||||||
|
}
|
||||||
|
|
||||||
|
\begin{document}
|
||||||
|
\maketitle
|
||||||
|
|
||||||
|
\begin{exercise}[subtitle={Étude de fonction}]
|
||||||
|
On considère la fonction $f$ définie sur $\intOF{0}{+\infty}$ par $ f(x) = 3.5x^2 + 21x + - 490\ln(x)$
|
||||||
|
\begin{enumerate}
|
||||||
|
\item Démontrer que la dérivée de $f$ est $f'(x) = \frac{7x^2 + 21x + - 490}{x}$.
|
||||||
|
\item Étude du numérateur de $f'(x)$: $N(x) = 7x^2 + 21x - 490$
|
||||||
|
\begin{enumerate}
|
||||||
|
\item Démontrer que $x=- 10$ et $x=7$ sont deux racines de $N(x)$..
|
||||||
|
\item Proposer une forme factorisée de $N(x)$.
|
||||||
|
\item Proposer une forme factorisée de $f'(x)$.
|
||||||
|
\end{enumerate}
|
||||||
|
\item Étudier le signe de $f'$ et en déduire les variations de $f$.
|
||||||
|
\end{enumerate}
|
||||||
|
\end{exercise}
|
||||||
|
|
||||||
|
\begin{solution}
|
||||||
|
\begin{enumerate}
|
||||||
|
\item pas de correction disponible
|
||||||
|
\item
|
||||||
|
\begin{enumerate}
|
||||||
|
\item \[N(- 10) = 0\]
|
||||||
|
\[N(7) = 0\]
|
||||||
|
\item \[
|
||||||
|
N(x) = 7(x - - 10)(x - 7)
|
||||||
|
\]
|
||||||
|
\item
|
||||||
|
\[
|
||||||
|
f'(x) = \frac{7(x - - 10)(x - 7)}{x}
|
||||||
|
\]
|
||||||
|
\end{enumerate}
|
||||||
|
\item Pas de correction disponible
|
||||||
|
\end{enumerate}
|
||||||
|
\end{solution}
|
||||||
|
|
||||||
|
\begin{exercise}[subtitle={Complexes}]
|
||||||
|
\begin{enumerate}
|
||||||
|
\item Mettre le nombre complexe suivant sous forme algébrique $z_1 = \dfrac{5 + 7 i}{-6 + 4 i} $
|
||||||
|
\item Mettre le complexe suivante sous forme exponentielle $z_2 = 1 - \sqrt{3} i$
|
||||||
|
\item Mettre le complexe suivante sous forme exponentielle $z_3 = - 8 \sqrt{2} + 8 \sqrt{2} i$
|
||||||
|
\item Calculer le produit $z_4=z_2\times z_3$ donner le résultat sous forme exponentielle puis algébrique.
|
||||||
|
\item Calculer le quotient $z_5=\frac{z_2}{z_3}$ donner le résultat sous forme exponentielle puis algébrique.
|
||||||
|
\end{enumerate}
|
||||||
|
\end{exercise}
|
||||||
|
|
||||||
|
\begin{solution}
|
||||||
|
\begin{enumerate}
|
||||||
|
\item $z_1 = - \frac{1}{26} - \frac{31 i}{26}$
|
||||||
|
\item $z_2 = 2 e^{- \frac{i \pi}{3}}$
|
||||||
|
\item $z_3 = 16 e^{\frac{3 i \pi}{4}}$
|
||||||
|
\item $z_4 = 32 e^{\frac{5 i \pi}{12}} = - 8 \sqrt{2} + 8 \sqrt{6} + i \left(8 \sqrt{2} + 8 \sqrt{6}\right) = 8.28 + 30.9 i$
|
||||||
|
\item $z_5 = \frac{1}{8} e^{- \frac{13 i \pi}{12}} = - \frac{\sqrt{6}}{32} - \frac{\sqrt{2}}{32} + i \left(- \frac{\sqrt{2}}{32} + \frac{\sqrt{6}}{32}\right) = -0.121 + 0.0323 i$
|
||||||
|
\end{enumerate}
|
||||||
|
\end{solution}
|
||||||
|
|
||||||
|
\begin{exercise}[subtitle={Sortie du congélateur}]
|
||||||
|
Marie a invité quelques amis pour le thé. Elle souhaite leur proposer ses macarons maison.
|
||||||
|
|
||||||
|
Elle les sort de son congélateur à $-19$~\degres C et les place dans une pièce à $15$~\degres C.
|
||||||
|
|
||||||
|
Au bout de 15 minutes, la température des macarons est de $-2$~\degres C.
|
||||||
|
|
||||||
|
\bigskip
|
||||||
|
|
||||||
|
\textbf{Premier modèle}
|
||||||
|
|
||||||
|
\medskip
|
||||||
|
|
||||||
|
On suppose que la vitesse de décongélation est constante : chaque minute la hausse de
|
||||||
|
température des macarons est la même.
|
||||||
|
|
||||||
|
Estimer dans ce cadre la température au bout de $30$~minutes, puis au bout de $45$~minutes.
|
||||||
|
|
||||||
|
Cette modélisation est-elle pertinente?
|
||||||
|
|
||||||
|
\bigskip
|
||||||
|
|
||||||
|
\textbf{Deuxième modèle}
|
||||||
|
|
||||||
|
\medskip
|
||||||
|
|
||||||
|
On suppose maintenant que la vitesse de décongélation est proportionnelle à la différence
|
||||||
|
de température entre les macarons et l'air ambiant (il s'agit de la loi de Newton).
|
||||||
|
|
||||||
|
On désigne par $\theta$ la température des macarons à l'instant $t$, et par $\theta'$ la vitesse de décongélation.
|
||||||
|
|
||||||
|
L'unité de temps est la minute et l'unité de température le degré Celsius.
|
||||||
|
|
||||||
|
\smallskip
|
||||||
|
|
||||||
|
On négligera la diminution de température de la pièce et on admettra donc qu'il existe un
|
||||||
|
nombre réel $a$ tel que, pour $t$ positif :
|
||||||
|
|
||||||
|
\[\theta'(t) = a [\theta(t) - 15]\quad (E)\]
|
||||||
|
|
||||||
|
\medskip
|
||||||
|
|
||||||
|
\begin{enumerate}
|
||||||
|
\item Vérifier que l'équation $(E)$ a pour solutions $\theta(t) = K e^{at} + 15$ où $K$ est un nombre réel.
|
||||||
|
|
||||||
|
Donner alors, en fonction de $a$, l'ensemble des solutions de $(E)$.
|
||||||
|
\end{enumerate}
|
||||||
|
On rappelle que la température des macarons à l'instant $t = 0$ est égale à $-19$~\degres C et que, au bout de $15$~min, elle est de $-2$~\degres C.
|
||||||
|
\begin{enumerate}
|
||||||
|
\setcounter{enumi}{1}
|
||||||
|
\item En utilisant la condition à $t=0$ démontrer que $K = -34$.
|
||||||
|
\item En utilisant la condition à $t=15$ démontrer que $a \approx -0.05$.
|
||||||
|
\item En déduire l'expression de la solution de l'équation différentielle puis étudier ses variations.
|
||||||
|
\item La température idéale de dégustation des macarons étant de $12$~\degres C, Marie estime que
|
||||||
|
celle-ci sera atteinte au bout de $30$~min. A-t-elle raison ? Justifier la réponse.
|
||||||
|
|
||||||
|
Sinon, combien de temps faudra-t-il attendre ?
|
||||||
|
\end{enumerate}
|
||||||
|
\end{exercise}
|
||||||
|
|
||||||
|
|
||||||
|
|
||||||
|
\end{document}
|
||||||
|
|
||||||
|
%%% Local Variables:
|
||||||
|
%%% mode: latex
|
||||||
|
%%% TeX-master: "master"
|
||||||
|
%%% End:
|
|
@ -0,0 +1,136 @@
|
||||||
|
\documentclass[a4paper,10pt]{article}
|
||||||
|
\usepackage{myXsim}
|
||||||
|
|
||||||
|
% Title Page
|
||||||
|
\title{DS8 \hfill M'BAREK HASNAOUI Bilal}
|
||||||
|
\tribe{TST sti2d}
|
||||||
|
\date{\hfillÀ render pour le vendredi 9 avril à 10h au plus tard}
|
||||||
|
|
||||||
|
\xsimsetup{
|
||||||
|
solution/print = true
|
||||||
|
}
|
||||||
|
|
||||||
|
\begin{document}
|
||||||
|
\maketitle
|
||||||
|
|
||||||
|
\begin{exercise}[subtitle={Étude de fonction}]
|
||||||
|
On considère la fonction $f$ définie sur $\intOF{0}{+\infty}$ par $ f(x) = 5x^2 + - 40x + - 450\ln(x)$
|
||||||
|
\begin{enumerate}
|
||||||
|
\item Démontrer que la dérivée de $f$ est $f'(x) = \frac{10x^2 + - 40x + - 450}{x}$.
|
||||||
|
\item Étude du numérateur de $f'(x)$: $N(x) = 10x^2 - 40x - 450$
|
||||||
|
\begin{enumerate}
|
||||||
|
\item Démontrer que $x=9$ et $x=- 5$ sont deux racines de $N(x)$..
|
||||||
|
\item Proposer une forme factorisée de $N(x)$.
|
||||||
|
\item Proposer une forme factorisée de $f'(x)$.
|
||||||
|
\end{enumerate}
|
||||||
|
\item Étudier le signe de $f'$ et en déduire les variations de $f$.
|
||||||
|
\end{enumerate}
|
||||||
|
\end{exercise}
|
||||||
|
|
||||||
|
\begin{solution}
|
||||||
|
\begin{enumerate}
|
||||||
|
\item pas de correction disponible
|
||||||
|
\item
|
||||||
|
\begin{enumerate}
|
||||||
|
\item \[N(9) = 0\]
|
||||||
|
\[N(- 5) = 0\]
|
||||||
|
\item \[
|
||||||
|
N(x) = 10(x - 9)(x - - 5)
|
||||||
|
\]
|
||||||
|
\item
|
||||||
|
\[
|
||||||
|
f'(x) = \frac{10(x - 9)(x - - 5)}{x}
|
||||||
|
\]
|
||||||
|
\end{enumerate}
|
||||||
|
\item Pas de correction disponible
|
||||||
|
\end{enumerate}
|
||||||
|
\end{solution}
|
||||||
|
|
||||||
|
\begin{exercise}[subtitle={Complexes}]
|
||||||
|
\begin{enumerate}
|
||||||
|
\item Mettre le nombre complexe suivant sous forme algébrique $z_1 = \dfrac{2 + 4 i}{-5 + 7 i} $
|
||||||
|
\item Mettre le complexe suivante sous forme exponentielle $z_2 = 7 + 7 \sqrt{3} i$
|
||||||
|
\item Mettre le complexe suivante sous forme exponentielle $z_3 = 7 \sqrt{2} + 7 \sqrt{2} i$
|
||||||
|
\item Calculer le produit $z_4=z_2\times z_3$ donner le résultat sous forme exponentielle puis algébrique.
|
||||||
|
\item Calculer le quotient $z_5=\frac{z_2}{z_3}$ donner le résultat sous forme exponentielle puis algébrique.
|
||||||
|
\end{enumerate}
|
||||||
|
\end{exercise}
|
||||||
|
|
||||||
|
\begin{solution}
|
||||||
|
\begin{enumerate}
|
||||||
|
\item $z_1 = \frac{9}{37} - \frac{17 i}{37}$
|
||||||
|
\item $z_2 = 14 e^{\frac{i \pi}{3}}$
|
||||||
|
\item $z_3 = 14 e^{\frac{i \pi}{4}}$
|
||||||
|
\item $z_4 = 196 e^{\frac{7 i \pi}{12}} = - 49 \sqrt{6} + 49 \sqrt{2} + i \left(49 \sqrt{2} + 49 \sqrt{6}\right) = -50.7 + 189.0 i$
|
||||||
|
\item $z_5 = 1 e^{\frac{i \pi}{12}} = \frac{\sqrt{2}}{4} + \frac{\sqrt{6}}{4} + i \left(- \frac{\sqrt{2}}{4} + \frac{\sqrt{6}}{4}\right) = 0.966 + 0.259 i$
|
||||||
|
\end{enumerate}
|
||||||
|
\end{solution}
|
||||||
|
|
||||||
|
\begin{exercise}[subtitle={Sortie du congélateur}]
|
||||||
|
Marie a invité quelques amis pour le thé. Elle souhaite leur proposer ses macarons maison.
|
||||||
|
|
||||||
|
Elle les sort de son congélateur à $-15$~\degres C et les place dans une pièce à $25$~\degres C.
|
||||||
|
|
||||||
|
Au bout de 15 minutes, la température des macarons est de $0$~\degres C.
|
||||||
|
|
||||||
|
\bigskip
|
||||||
|
|
||||||
|
\textbf{Premier modèle}
|
||||||
|
|
||||||
|
\medskip
|
||||||
|
|
||||||
|
On suppose que la vitesse de décongélation est constante : chaque minute la hausse de
|
||||||
|
température des macarons est la même.
|
||||||
|
|
||||||
|
Estimer dans ce cadre la température au bout de $30$~minutes, puis au bout de $45$~minutes.
|
||||||
|
|
||||||
|
Cette modélisation est-elle pertinente?
|
||||||
|
|
||||||
|
\bigskip
|
||||||
|
|
||||||
|
\textbf{Deuxième modèle}
|
||||||
|
|
||||||
|
\medskip
|
||||||
|
|
||||||
|
On suppose maintenant que la vitesse de décongélation est proportionnelle à la différence
|
||||||
|
de température entre les macarons et l'air ambiant (il s'agit de la loi de Newton).
|
||||||
|
|
||||||
|
On désigne par $\theta$ la température des macarons à l'instant $t$, et par $\theta'$ la vitesse de décongélation.
|
||||||
|
|
||||||
|
L'unité de temps est la minute et l'unité de température le degré Celsius.
|
||||||
|
|
||||||
|
\smallskip
|
||||||
|
|
||||||
|
On négligera la diminution de température de la pièce et on admettra donc qu'il existe un
|
||||||
|
nombre réel $a$ tel que, pour $t$ positif :
|
||||||
|
|
||||||
|
\[\theta'(t) = a [\theta(t) - 25]\quad (E)\]
|
||||||
|
|
||||||
|
\medskip
|
||||||
|
|
||||||
|
\begin{enumerate}
|
||||||
|
\item Vérifier que l'équation $(E)$ a pour solutions $\theta(t) = K e^{at} + 25$ où $K$ est un nombre réel.
|
||||||
|
|
||||||
|
Donner alors, en fonction de $a$, l'ensemble des solutions de $(E)$.
|
||||||
|
\end{enumerate}
|
||||||
|
On rappelle que la température des macarons à l'instant $t = 0$ est égale à $-15$~\degres C et que, au bout de $15$~min, elle est de $0$~\degres C.
|
||||||
|
\begin{enumerate}
|
||||||
|
\setcounter{enumi}{1}
|
||||||
|
\item En utilisant la condition à $t=0$ démontrer que $K = -40$.
|
||||||
|
\item En utilisant la condition à $t=15$ démontrer que $a \approx -0.03$.
|
||||||
|
\item En déduire l'expression de la solution de l'équation différentielle puis étudier ses variations.
|
||||||
|
\item La température idéale de dégustation des macarons étant de $22$~\degres C, Marie estime que
|
||||||
|
celle-ci sera atteinte au bout de $30$~min. A-t-elle raison ? Justifier la réponse.
|
||||||
|
|
||||||
|
Sinon, combien de temps faudra-t-il attendre ?
|
||||||
|
\end{enumerate}
|
||||||
|
\end{exercise}
|
||||||
|
|
||||||
|
|
||||||
|
|
||||||
|
\end{document}
|
||||||
|
|
||||||
|
%%% Local Variables:
|
||||||
|
%%% mode: latex
|
||||||
|
%%% TeX-master: "master"
|
||||||
|
%%% End:
|
|
@ -0,0 +1,136 @@
|
||||||
|
\documentclass[a4paper,10pt]{article}
|
||||||
|
\usepackage{myXsim}
|
||||||
|
|
||||||
|
% Title Page
|
||||||
|
\title{DS8 \hfill MERCIER Almandin}
|
||||||
|
\tribe{TST sti2d}
|
||||||
|
\date{\hfillÀ render pour le vendredi 9 avril à 10h au plus tard}
|
||||||
|
|
||||||
|
\xsimsetup{
|
||||||
|
solution/print = true
|
||||||
|
}
|
||||||
|
|
||||||
|
\begin{document}
|
||||||
|
\maketitle
|
||||||
|
|
||||||
|
\begin{exercise}[subtitle={Étude de fonction}]
|
||||||
|
On considère la fonction $f$ définie sur $\intOF{0}{+\infty}$ par $ f(x) = 2.5x^2 + - 50x + 120\ln(x)$
|
||||||
|
\begin{enumerate}
|
||||||
|
\item Démontrer que la dérivée de $f$ est $f'(x) = \frac{5x^2 + - 50x + 120}{x}$.
|
||||||
|
\item Étude du numérateur de $f'(x)$: $N(x) = 5x^2 - 50x + 120$
|
||||||
|
\begin{enumerate}
|
||||||
|
\item Démontrer que $x=6$ et $x=4$ sont deux racines de $N(x)$..
|
||||||
|
\item Proposer une forme factorisée de $N(x)$.
|
||||||
|
\item Proposer une forme factorisée de $f'(x)$.
|
||||||
|
\end{enumerate}
|
||||||
|
\item Étudier le signe de $f'$ et en déduire les variations de $f$.
|
||||||
|
\end{enumerate}
|
||||||
|
\end{exercise}
|
||||||
|
|
||||||
|
\begin{solution}
|
||||||
|
\begin{enumerate}
|
||||||
|
\item pas de correction disponible
|
||||||
|
\item
|
||||||
|
\begin{enumerate}
|
||||||
|
\item \[N(6) = 0\]
|
||||||
|
\[N(4) = 0\]
|
||||||
|
\item \[
|
||||||
|
N(x) = 5(x - 6)(x - 4)
|
||||||
|
\]
|
||||||
|
\item
|
||||||
|
\[
|
||||||
|
f'(x) = \frac{5(x - 6)(x - 4)}{x}
|
||||||
|
\]
|
||||||
|
\end{enumerate}
|
||||||
|
\item Pas de correction disponible
|
||||||
|
\end{enumerate}
|
||||||
|
\end{solution}
|
||||||
|
|
||||||
|
\begin{exercise}[subtitle={Complexes}]
|
||||||
|
\begin{enumerate}
|
||||||
|
\item Mettre le nombre complexe suivant sous forme algébrique $z_1 = \dfrac{10 + 10 i}{-2 + 3 i} $
|
||||||
|
\item Mettre le complexe suivante sous forme exponentielle $z_2 = - 7 \sqrt{2} - 7 \sqrt{2} i$
|
||||||
|
\item Mettre le complexe suivante sous forme exponentielle $z_3 = - 9 \sqrt{2} + 9 \sqrt{2} i$
|
||||||
|
\item Calculer le produit $z_4=z_2\times z_3$ donner le résultat sous forme exponentielle puis algébrique.
|
||||||
|
\item Calculer le quotient $z_5=\frac{z_2}{z_3}$ donner le résultat sous forme exponentielle puis algébrique.
|
||||||
|
\end{enumerate}
|
||||||
|
\end{exercise}
|
||||||
|
|
||||||
|
\begin{solution}
|
||||||
|
\begin{enumerate}
|
||||||
|
\item $z_1 = \frac{10}{13} - \frac{50 i}{13}$
|
||||||
|
\item $z_2 = 14 e^{- \frac{3 i \pi}{4}}$
|
||||||
|
\item $z_3 = 18 e^{\frac{3 i \pi}{4}}$
|
||||||
|
\item $z_4 = 252 e^{0} = 252 = 252.0$
|
||||||
|
\item $z_5 = \frac{7}{9} e^{- \frac{3 i \pi}{2}} = \frac{7 i}{9} = 0.778 i$
|
||||||
|
\end{enumerate}
|
||||||
|
\end{solution}
|
||||||
|
|
||||||
|
\begin{exercise}[subtitle={Sortie du congélateur}]
|
||||||
|
Marie a invité quelques amis pour le thé. Elle souhaite leur proposer ses macarons maison.
|
||||||
|
|
||||||
|
Elle les sort de son congélateur à $-16$~\degres C et les place dans une pièce à $22$~\degres C.
|
||||||
|
|
||||||
|
Au bout de 15 minutes, la température des macarons est de $3$~\degres C.
|
||||||
|
|
||||||
|
\bigskip
|
||||||
|
|
||||||
|
\textbf{Premier modèle}
|
||||||
|
|
||||||
|
\medskip
|
||||||
|
|
||||||
|
On suppose que la vitesse de décongélation est constante : chaque minute la hausse de
|
||||||
|
température des macarons est la même.
|
||||||
|
|
||||||
|
Estimer dans ce cadre la température au bout de $30$~minutes, puis au bout de $45$~minutes.
|
||||||
|
|
||||||
|
Cette modélisation est-elle pertinente?
|
||||||
|
|
||||||
|
\bigskip
|
||||||
|
|
||||||
|
\textbf{Deuxième modèle}
|
||||||
|
|
||||||
|
\medskip
|
||||||
|
|
||||||
|
On suppose maintenant que la vitesse de décongélation est proportionnelle à la différence
|
||||||
|
de température entre les macarons et l'air ambiant (il s'agit de la loi de Newton).
|
||||||
|
|
||||||
|
On désigne par $\theta$ la température des macarons à l'instant $t$, et par $\theta'$ la vitesse de décongélation.
|
||||||
|
|
||||||
|
L'unité de temps est la minute et l'unité de température le degré Celsius.
|
||||||
|
|
||||||
|
\smallskip
|
||||||
|
|
||||||
|
On négligera la diminution de température de la pièce et on admettra donc qu'il existe un
|
||||||
|
nombre réel $a$ tel que, pour $t$ positif :
|
||||||
|
|
||||||
|
\[\theta'(t) = a [\theta(t) - 22]\quad (E)\]
|
||||||
|
|
||||||
|
\medskip
|
||||||
|
|
||||||
|
\begin{enumerate}
|
||||||
|
\item Vérifier que l'équation $(E)$ a pour solutions $\theta(t) = K e^{at} + 22$ où $K$ est un nombre réel.
|
||||||
|
|
||||||
|
Donner alors, en fonction de $a$, l'ensemble des solutions de $(E)$.
|
||||||
|
\end{enumerate}
|
||||||
|
On rappelle que la température des macarons à l'instant $t = 0$ est égale à $-16$~\degres C et que, au bout de $15$~min, elle est de $3$~\degres C.
|
||||||
|
\begin{enumerate}
|
||||||
|
\setcounter{enumi}{1}
|
||||||
|
\item En utilisant la condition à $t=0$ démontrer que $K = -38$.
|
||||||
|
\item En utilisant la condition à $t=15$ démontrer que $a \approx -0.05$.
|
||||||
|
\item En déduire l'expression de la solution de l'équation différentielle puis étudier ses variations.
|
||||||
|
\item La température idéale de dégustation des macarons étant de $19$~\degres C, Marie estime que
|
||||||
|
celle-ci sera atteinte au bout de $30$~min. A-t-elle raison ? Justifier la réponse.
|
||||||
|
|
||||||
|
Sinon, combien de temps faudra-t-il attendre ?
|
||||||
|
\end{enumerate}
|
||||||
|
\end{exercise}
|
||||||
|
|
||||||
|
|
||||||
|
|
||||||
|
\end{document}
|
||||||
|
|
||||||
|
%%% Local Variables:
|
||||||
|
%%% mode: latex
|
||||||
|
%%% TeX-master: "master"
|
||||||
|
%%% End:
|
|
@ -0,0 +1,136 @@
|
||||||
|
\documentclass[a4paper,10pt]{article}
|
||||||
|
\usepackage{myXsim}
|
||||||
|
|
||||||
|
% Title Page
|
||||||
|
\title{DS8 \hfill MOUFAQ Amine}
|
||||||
|
\tribe{TST sti2d}
|
||||||
|
\date{\hfillÀ render pour le vendredi 9 avril à 10h au plus tard}
|
||||||
|
|
||||||
|
\xsimsetup{
|
||||||
|
solution/print = true
|
||||||
|
}
|
||||||
|
|
||||||
|
\begin{document}
|
||||||
|
\maketitle
|
||||||
|
|
||||||
|
\begin{exercise}[subtitle={Étude de fonction}]
|
||||||
|
On considère la fonction $f$ définie sur $\intOF{0}{+\infty}$ par $ f(x) = 5x^2 + 0x + - 810\ln(x)$
|
||||||
|
\begin{enumerate}
|
||||||
|
\item Démontrer que la dérivée de $f$ est $f'(x) = \frac{10x^2 + 0x + - 810}{x}$.
|
||||||
|
\item Étude du numérateur de $f'(x)$: $N(x) = 10x^2 - 810$
|
||||||
|
\begin{enumerate}
|
||||||
|
\item Démontrer que $x=- 9$ et $x=9$ sont deux racines de $N(x)$..
|
||||||
|
\item Proposer une forme factorisée de $N(x)$.
|
||||||
|
\item Proposer une forme factorisée de $f'(x)$.
|
||||||
|
\end{enumerate}
|
||||||
|
\item Étudier le signe de $f'$ et en déduire les variations de $f$.
|
||||||
|
\end{enumerate}
|
||||||
|
\end{exercise}
|
||||||
|
|
||||||
|
\begin{solution}
|
||||||
|
\begin{enumerate}
|
||||||
|
\item pas de correction disponible
|
||||||
|
\item
|
||||||
|
\begin{enumerate}
|
||||||
|
\item \[N(- 9) = 0\]
|
||||||
|
\[N(9) = 0\]
|
||||||
|
\item \[
|
||||||
|
N(x) = 10(x - - 9)(x - 9)
|
||||||
|
\]
|
||||||
|
\item
|
||||||
|
\[
|
||||||
|
f'(x) = \frac{10(x - - 9)(x - 9)}{x}
|
||||||
|
\]
|
||||||
|
\end{enumerate}
|
||||||
|
\item Pas de correction disponible
|
||||||
|
\end{enumerate}
|
||||||
|
\end{solution}
|
||||||
|
|
||||||
|
\begin{exercise}[subtitle={Complexes}]
|
||||||
|
\begin{enumerate}
|
||||||
|
\item Mettre le nombre complexe suivant sous forme algébrique $z_1 = \dfrac{8 + 4 i}{-3 + 5 i} $
|
||||||
|
\item Mettre le complexe suivante sous forme exponentielle $z_2 = - 2 \sqrt{2} + 2 \sqrt{2} i$
|
||||||
|
\item Mettre le complexe suivante sous forme exponentielle $z_3 = 6 + 6 \sqrt{3} i$
|
||||||
|
\item Calculer le produit $z_4=z_2\times z_3$ donner le résultat sous forme exponentielle puis algébrique.
|
||||||
|
\item Calculer le quotient $z_5=\frac{z_2}{z_3}$ donner le résultat sous forme exponentielle puis algébrique.
|
||||||
|
\end{enumerate}
|
||||||
|
\end{exercise}
|
||||||
|
|
||||||
|
\begin{solution}
|
||||||
|
\begin{enumerate}
|
||||||
|
\item $z_1 = - \frac{2}{17} - \frac{26 i}{17}$
|
||||||
|
\item $z_2 = 4 e^{\frac{3 i \pi}{4}}$
|
||||||
|
\item $z_3 = 12 e^{\frac{i \pi}{3}}$
|
||||||
|
\item $z_4 = 48 e^{\frac{13 i \pi}{12}} = - 12 \sqrt{6} - 12 \sqrt{2} + i \left(- 12 \sqrt{6} + 12 \sqrt{2}\right) = -46.4 - 12.4 i$
|
||||||
|
\item $z_5 = \frac{1}{3} e^{\frac{5 i \pi}{12}} = - \frac{\sqrt{2}}{12} + \frac{\sqrt{6}}{12} + i \left(\frac{\sqrt{2}}{12} + \frac{\sqrt{6}}{12}\right) = 0.0863 + 0.322 i$
|
||||||
|
\end{enumerate}
|
||||||
|
\end{solution}
|
||||||
|
|
||||||
|
\begin{exercise}[subtitle={Sortie du congélateur}]
|
||||||
|
Marie a invité quelques amis pour le thé. Elle souhaite leur proposer ses macarons maison.
|
||||||
|
|
||||||
|
Elle les sort de son congélateur à $-15$~\degres C et les place dans une pièce à $20$~\degres C.
|
||||||
|
|
||||||
|
Au bout de 15 minutes, la température des macarons est de $-3$~\degres C.
|
||||||
|
|
||||||
|
\bigskip
|
||||||
|
|
||||||
|
\textbf{Premier modèle}
|
||||||
|
|
||||||
|
\medskip
|
||||||
|
|
||||||
|
On suppose que la vitesse de décongélation est constante : chaque minute la hausse de
|
||||||
|
température des macarons est la même.
|
||||||
|
|
||||||
|
Estimer dans ce cadre la température au bout de $30$~minutes, puis au bout de $45$~minutes.
|
||||||
|
|
||||||
|
Cette modélisation est-elle pertinente?
|
||||||
|
|
||||||
|
\bigskip
|
||||||
|
|
||||||
|
\textbf{Deuxième modèle}
|
||||||
|
|
||||||
|
\medskip
|
||||||
|
|
||||||
|
On suppose maintenant que la vitesse de décongélation est proportionnelle à la différence
|
||||||
|
de température entre les macarons et l'air ambiant (il s'agit de la loi de Newton).
|
||||||
|
|
||||||
|
On désigne par $\theta$ la température des macarons à l'instant $t$, et par $\theta'$ la vitesse de décongélation.
|
||||||
|
|
||||||
|
L'unité de temps est la minute et l'unité de température le degré Celsius.
|
||||||
|
|
||||||
|
\smallskip
|
||||||
|
|
||||||
|
On négligera la diminution de température de la pièce et on admettra donc qu'il existe un
|
||||||
|
nombre réel $a$ tel que, pour $t$ positif :
|
||||||
|
|
||||||
|
\[\theta'(t) = a [\theta(t) - 20]\quad (E)\]
|
||||||
|
|
||||||
|
\medskip
|
||||||
|
|
||||||
|
\begin{enumerate}
|
||||||
|
\item Vérifier que l'équation $(E)$ a pour solutions $\theta(t) = K e^{at} + 20$ où $K$ est un nombre réel.
|
||||||
|
|
||||||
|
Donner alors, en fonction de $a$, l'ensemble des solutions de $(E)$.
|
||||||
|
\end{enumerate}
|
||||||
|
On rappelle que la température des macarons à l'instant $t = 0$ est égale à $-15$~\degres C et que, au bout de $15$~min, elle est de $-3$~\degres C.
|
||||||
|
\begin{enumerate}
|
||||||
|
\setcounter{enumi}{1}
|
||||||
|
\item En utilisant la condition à $t=0$ démontrer que $K = -35$.
|
||||||
|
\item En utilisant la condition à $t=15$ démontrer que $a \approx -0.03$.
|
||||||
|
\item En déduire l'expression de la solution de l'équation différentielle puis étudier ses variations.
|
||||||
|
\item La température idéale de dégustation des macarons étant de $17$~\degres C, Marie estime que
|
||||||
|
celle-ci sera atteinte au bout de $30$~min. A-t-elle raison ? Justifier la réponse.
|
||||||
|
|
||||||
|
Sinon, combien de temps faudra-t-il attendre ?
|
||||||
|
\end{enumerate}
|
||||||
|
\end{exercise}
|
||||||
|
|
||||||
|
|
||||||
|
|
||||||
|
\end{document}
|
||||||
|
|
||||||
|
%%% Local Variables:
|
||||||
|
%%% mode: latex
|
||||||
|
%%% TeX-master: "master"
|
||||||
|
%%% End:
|
|
@ -0,0 +1,136 @@
|
||||||
|
\documentclass[a4paper,10pt]{article}
|
||||||
|
\usepackage{myXsim}
|
||||||
|
|
||||||
|
% Title Page
|
||||||
|
\title{DS8 \hfill NARDINI Kakary}
|
||||||
|
\tribe{TST sti2d}
|
||||||
|
\date{\hfillÀ render pour le vendredi 9 avril à 10h au plus tard}
|
||||||
|
|
||||||
|
\xsimsetup{
|
||||||
|
solution/print = true
|
||||||
|
}
|
||||||
|
|
||||||
|
\begin{document}
|
||||||
|
\maketitle
|
||||||
|
|
||||||
|
\begin{exercise}[subtitle={Étude de fonction}]
|
||||||
|
On considère la fonction $f$ définie sur $\intOF{0}{+\infty}$ par $ f(x) = 3x^2 + - 18x + - 108\ln(x)$
|
||||||
|
\begin{enumerate}
|
||||||
|
\item Démontrer que la dérivée de $f$ est $f'(x) = \frac{6x^2 + - 18x + - 108}{x}$.
|
||||||
|
\item Étude du numérateur de $f'(x)$: $N(x) = 6x^2 - 18x - 108$
|
||||||
|
\begin{enumerate}
|
||||||
|
\item Démontrer que $x=6$ et $x=- 3$ sont deux racines de $N(x)$..
|
||||||
|
\item Proposer une forme factorisée de $N(x)$.
|
||||||
|
\item Proposer une forme factorisée de $f'(x)$.
|
||||||
|
\end{enumerate}
|
||||||
|
\item Étudier le signe de $f'$ et en déduire les variations de $f$.
|
||||||
|
\end{enumerate}
|
||||||
|
\end{exercise}
|
||||||
|
|
||||||
|
\begin{solution}
|
||||||
|
\begin{enumerate}
|
||||||
|
\item pas de correction disponible
|
||||||
|
\item
|
||||||
|
\begin{enumerate}
|
||||||
|
\item \[N(6) = 0\]
|
||||||
|
\[N(- 3) = 0\]
|
||||||
|
\item \[
|
||||||
|
N(x) = 6(x - 6)(x - - 3)
|
||||||
|
\]
|
||||||
|
\item
|
||||||
|
\[
|
||||||
|
f'(x) = \frac{6(x - 6)(x - - 3)}{x}
|
||||||
|
\]
|
||||||
|
\end{enumerate}
|
||||||
|
\item Pas de correction disponible
|
||||||
|
\end{enumerate}
|
||||||
|
\end{solution}
|
||||||
|
|
||||||
|
\begin{exercise}[subtitle={Complexes}]
|
||||||
|
\begin{enumerate}
|
||||||
|
\item Mettre le nombre complexe suivant sous forme algébrique $z_1 = \dfrac{9 + 6 i}{-3 + 6 i} $
|
||||||
|
\item Mettre le complexe suivante sous forme exponentielle $z_2 = -4 - 4 \sqrt{3} i$
|
||||||
|
\item Mettre le complexe suivante sous forme exponentielle $z_3 = 9 \sqrt{2} + 9 \sqrt{2} i$
|
||||||
|
\item Calculer le produit $z_4=z_2\times z_3$ donner le résultat sous forme exponentielle puis algébrique.
|
||||||
|
\item Calculer le quotient $z_5=\frac{z_2}{z_3}$ donner le résultat sous forme exponentielle puis algébrique.
|
||||||
|
\end{enumerate}
|
||||||
|
\end{exercise}
|
||||||
|
|
||||||
|
\begin{solution}
|
||||||
|
\begin{enumerate}
|
||||||
|
\item $z_1 = \frac{1}{5} - \frac{8 i}{5}$
|
||||||
|
\item $z_2 = 8 e^{- \frac{2 i \pi}{3}}$
|
||||||
|
\item $z_3 = 18 e^{\frac{i \pi}{4}}$
|
||||||
|
\item $z_4 = 144 e^{- \frac{5 i \pi}{12}} = - 36 \sqrt{2} + 36 \sqrt{6} + i \left(- 36 \sqrt{6} - 36 \sqrt{2}\right) = 37.3 - 139.0 i$
|
||||||
|
\item $z_5 = \frac{4}{9} e^{- \frac{11 i \pi}{12}} = - \frac{\sqrt{6}}{9} - \frac{\sqrt{2}}{9} + i \left(- \frac{\sqrt{6}}{9} + \frac{\sqrt{2}}{9}\right) = -0.429 - 0.115 i$
|
||||||
|
\end{enumerate}
|
||||||
|
\end{solution}
|
||||||
|
|
||||||
|
\begin{exercise}[subtitle={Sortie du congélateur}]
|
||||||
|
Marie a invité quelques amis pour le thé. Elle souhaite leur proposer ses macarons maison.
|
||||||
|
|
||||||
|
Elle les sort de son congélateur à $-18$~\degres C et les place dans une pièce à $17$~\degres C.
|
||||||
|
|
||||||
|
Au bout de 15 minutes, la température des macarons est de $2$~\degres C.
|
||||||
|
|
||||||
|
\bigskip
|
||||||
|
|
||||||
|
\textbf{Premier modèle}
|
||||||
|
|
||||||
|
\medskip
|
||||||
|
|
||||||
|
On suppose que la vitesse de décongélation est constante : chaque minute la hausse de
|
||||||
|
température des macarons est la même.
|
||||||
|
|
||||||
|
Estimer dans ce cadre la température au bout de $30$~minutes, puis au bout de $45$~minutes.
|
||||||
|
|
||||||
|
Cette modélisation est-elle pertinente?
|
||||||
|
|
||||||
|
\bigskip
|
||||||
|
|
||||||
|
\textbf{Deuxième modèle}
|
||||||
|
|
||||||
|
\medskip
|
||||||
|
|
||||||
|
On suppose maintenant que la vitesse de décongélation est proportionnelle à la différence
|
||||||
|
de température entre les macarons et l'air ambiant (il s'agit de la loi de Newton).
|
||||||
|
|
||||||
|
On désigne par $\theta$ la température des macarons à l'instant $t$, et par $\theta'$ la vitesse de décongélation.
|
||||||
|
|
||||||
|
L'unité de temps est la minute et l'unité de température le degré Celsius.
|
||||||
|
|
||||||
|
\smallskip
|
||||||
|
|
||||||
|
On négligera la diminution de température de la pièce et on admettra donc qu'il existe un
|
||||||
|
nombre réel $a$ tel que, pour $t$ positif :
|
||||||
|
|
||||||
|
\[\theta'(t) = a [\theta(t) - 17]\quad (E)\]
|
||||||
|
|
||||||
|
\medskip
|
||||||
|
|
||||||
|
\begin{enumerate}
|
||||||
|
\item Vérifier que l'équation $(E)$ a pour solutions $\theta(t) = K e^{at} + 17$ où $K$ est un nombre réel.
|
||||||
|
|
||||||
|
Donner alors, en fonction de $a$, l'ensemble des solutions de $(E)$.
|
||||||
|
\end{enumerate}
|
||||||
|
On rappelle que la température des macarons à l'instant $t = 0$ est égale à $-18$~\degres C et que, au bout de $15$~min, elle est de $2$~\degres C.
|
||||||
|
\begin{enumerate}
|
||||||
|
\setcounter{enumi}{1}
|
||||||
|
\item En utilisant la condition à $t=0$ démontrer que $K = -35$.
|
||||||
|
\item En utilisant la condition à $t=15$ démontrer que $a \approx -0.06$.
|
||||||
|
\item En déduire l'expression de la solution de l'équation différentielle puis étudier ses variations.
|
||||||
|
\item La température idéale de dégustation des macarons étant de $14$~\degres C, Marie estime que
|
||||||
|
celle-ci sera atteinte au bout de $30$~min. A-t-elle raison ? Justifier la réponse.
|
||||||
|
|
||||||
|
Sinon, combien de temps faudra-t-il attendre ?
|
||||||
|
\end{enumerate}
|
||||||
|
\end{exercise}
|
||||||
|
|
||||||
|
|
||||||
|
|
||||||
|
\end{document}
|
||||||
|
|
||||||
|
%%% Local Variables:
|
||||||
|
%%% mode: latex
|
||||||
|
%%% TeX-master: "master"
|
||||||
|
%%% End:
|
|
@ -0,0 +1,136 @@
|
||||||
|
\documentclass[a4paper,10pt]{article}
|
||||||
|
\usepackage{myXsim}
|
||||||
|
|
||||||
|
% Title Page
|
||||||
|
\title{DS8 \hfill ONAL Yakub}
|
||||||
|
\tribe{TST sti2d}
|
||||||
|
\date{\hfillÀ render pour le vendredi 9 avril à 10h au plus tard}
|
||||||
|
|
||||||
|
\xsimsetup{
|
||||||
|
solution/print = true
|
||||||
|
}
|
||||||
|
|
||||||
|
\begin{document}
|
||||||
|
\maketitle
|
||||||
|
|
||||||
|
\begin{exercise}[subtitle={Étude de fonction}]
|
||||||
|
On considère la fonction $f$ définie sur $\intOF{0}{+\infty}$ par $ f(x) = 5x^2 + - 40x + - 450\ln(x)$
|
||||||
|
\begin{enumerate}
|
||||||
|
\item Démontrer que la dérivée de $f$ est $f'(x) = \frac{10x^2 + - 40x + - 450}{x}$.
|
||||||
|
\item Étude du numérateur de $f'(x)$: $N(x) = 10x^2 - 40x - 450$
|
||||||
|
\begin{enumerate}
|
||||||
|
\item Démontrer que $x=9$ et $x=- 5$ sont deux racines de $N(x)$..
|
||||||
|
\item Proposer une forme factorisée de $N(x)$.
|
||||||
|
\item Proposer une forme factorisée de $f'(x)$.
|
||||||
|
\end{enumerate}
|
||||||
|
\item Étudier le signe de $f'$ et en déduire les variations de $f$.
|
||||||
|
\end{enumerate}
|
||||||
|
\end{exercise}
|
||||||
|
|
||||||
|
\begin{solution}
|
||||||
|
\begin{enumerate}
|
||||||
|
\item pas de correction disponible
|
||||||
|
\item
|
||||||
|
\begin{enumerate}
|
||||||
|
\item \[N(9) = 0\]
|
||||||
|
\[N(- 5) = 0\]
|
||||||
|
\item \[
|
||||||
|
N(x) = 10(x - 9)(x - - 5)
|
||||||
|
\]
|
||||||
|
\item
|
||||||
|
\[
|
||||||
|
f'(x) = \frac{10(x - 9)(x - - 5)}{x}
|
||||||
|
\]
|
||||||
|
\end{enumerate}
|
||||||
|
\item Pas de correction disponible
|
||||||
|
\end{enumerate}
|
||||||
|
\end{solution}
|
||||||
|
|
||||||
|
\begin{exercise}[subtitle={Complexes}]
|
||||||
|
\begin{enumerate}
|
||||||
|
\item Mettre le nombre complexe suivant sous forme algébrique $z_1 = \dfrac{5 + 2 i}{-10 + 2 i} $
|
||||||
|
\item Mettre le complexe suivante sous forme exponentielle $z_2 = 6 - 6 \sqrt{3} i$
|
||||||
|
\item Mettre le complexe suivante sous forme exponentielle $z_3 = 10 \sqrt{3} + 10 i$
|
||||||
|
\item Calculer le produit $z_4=z_2\times z_3$ donner le résultat sous forme exponentielle puis algébrique.
|
||||||
|
\item Calculer le quotient $z_5=\frac{z_2}{z_3}$ donner le résultat sous forme exponentielle puis algébrique.
|
||||||
|
\end{enumerate}
|
||||||
|
\end{exercise}
|
||||||
|
|
||||||
|
\begin{solution}
|
||||||
|
\begin{enumerate}
|
||||||
|
\item $z_1 = - \frac{23}{52} - \frac{15 i}{52}$
|
||||||
|
\item $z_2 = 12 e^{- \frac{i \pi}{3}}$
|
||||||
|
\item $z_3 = 20 e^{\frac{i \pi}{6}}$
|
||||||
|
\item $z_4 = 240 e^{- \frac{i \pi}{6}} = 120 \sqrt{3} - 120 i = 208.0 - 120.0 i$
|
||||||
|
\item $z_5 = \frac{3}{5} e^{- \frac{i \pi}{2}} = - \frac{3 i}{5} = - 0.6 i$
|
||||||
|
\end{enumerate}
|
||||||
|
\end{solution}
|
||||||
|
|
||||||
|
\begin{exercise}[subtitle={Sortie du congélateur}]
|
||||||
|
Marie a invité quelques amis pour le thé. Elle souhaite leur proposer ses macarons maison.
|
||||||
|
|
||||||
|
Elle les sort de son congélateur à $-18$~\degres C et les place dans une pièce à $19$~\degres C.
|
||||||
|
|
||||||
|
Au bout de 15 minutes, la température des macarons est de $-2$~\degres C.
|
||||||
|
|
||||||
|
\bigskip
|
||||||
|
|
||||||
|
\textbf{Premier modèle}
|
||||||
|
|
||||||
|
\medskip
|
||||||
|
|
||||||
|
On suppose que la vitesse de décongélation est constante : chaque minute la hausse de
|
||||||
|
température des macarons est la même.
|
||||||
|
|
||||||
|
Estimer dans ce cadre la température au bout de $30$~minutes, puis au bout de $45$~minutes.
|
||||||
|
|
||||||
|
Cette modélisation est-elle pertinente?
|
||||||
|
|
||||||
|
\bigskip
|
||||||
|
|
||||||
|
\textbf{Deuxième modèle}
|
||||||
|
|
||||||
|
\medskip
|
||||||
|
|
||||||
|
On suppose maintenant que la vitesse de décongélation est proportionnelle à la différence
|
||||||
|
de température entre les macarons et l'air ambiant (il s'agit de la loi de Newton).
|
||||||
|
|
||||||
|
On désigne par $\theta$ la température des macarons à l'instant $t$, et par $\theta'$ la vitesse de décongélation.
|
||||||
|
|
||||||
|
L'unité de temps est la minute et l'unité de température le degré Celsius.
|
||||||
|
|
||||||
|
\smallskip
|
||||||
|
|
||||||
|
On négligera la diminution de température de la pièce et on admettra donc qu'il existe un
|
||||||
|
nombre réel $a$ tel que, pour $t$ positif :
|
||||||
|
|
||||||
|
\[\theta'(t) = a [\theta(t) - 19]\quad (E)\]
|
||||||
|
|
||||||
|
\medskip
|
||||||
|
|
||||||
|
\begin{enumerate}
|
||||||
|
\item Vérifier que l'équation $(E)$ a pour solutions $\theta(t) = K e^{at} + 19$ où $K$ est un nombre réel.
|
||||||
|
|
||||||
|
Donner alors, en fonction de $a$, l'ensemble des solutions de $(E)$.
|
||||||
|
\end{enumerate}
|
||||||
|
On rappelle que la température des macarons à l'instant $t = 0$ est égale à $-18$~\degres C et que, au bout de $15$~min, elle est de $-2$~\degres C.
|
||||||
|
\begin{enumerate}
|
||||||
|
\setcounter{enumi}{1}
|
||||||
|
\item En utilisant la condition à $t=0$ démontrer que $K = -37$.
|
||||||
|
\item En utilisant la condition à $t=15$ démontrer que $a \approx -0.04$.
|
||||||
|
\item En déduire l'expression de la solution de l'équation différentielle puis étudier ses variations.
|
||||||
|
\item La température idéale de dégustation des macarons étant de $16$~\degres C, Marie estime que
|
||||||
|
celle-ci sera atteinte au bout de $30$~min. A-t-elle raison ? Justifier la réponse.
|
||||||
|
|
||||||
|
Sinon, combien de temps faudra-t-il attendre ?
|
||||||
|
\end{enumerate}
|
||||||
|
\end{exercise}
|
||||||
|
|
||||||
|
|
||||||
|
|
||||||
|
\end{document}
|
||||||
|
|
||||||
|
%%% Local Variables:
|
||||||
|
%%% mode: latex
|
||||||
|
%%% TeX-master: "master"
|
||||||
|
%%% End:
|
|
@ -0,0 +1,136 @@
|
||||||
|
\documentclass[a4paper,10pt]{article}
|
||||||
|
\usepackage{myXsim}
|
||||||
|
|
||||||
|
% Title Page
|
||||||
|
\title{DS8 \hfill RADOUAA Saleh}
|
||||||
|
\tribe{TST sti2d}
|
||||||
|
\date{\hfillÀ render pour le vendredi 9 avril à 10h au plus tard}
|
||||||
|
|
||||||
|
\xsimsetup{
|
||||||
|
solution/print = true
|
||||||
|
}
|
||||||
|
|
||||||
|
\begin{document}
|
||||||
|
\maketitle
|
||||||
|
|
||||||
|
\begin{exercise}[subtitle={Étude de fonction}]
|
||||||
|
On considère la fonction $f$ définie sur $\intOF{0}{+\infty}$ par $ f(x) = 3x^2 + - 54x + 108\ln(x)$
|
||||||
|
\begin{enumerate}
|
||||||
|
\item Démontrer que la dérivée de $f$ est $f'(x) = \frac{6x^2 + - 54x + 108}{x}$.
|
||||||
|
\item Étude du numérateur de $f'(x)$: $N(x) = 6x^2 - 54x + 108$
|
||||||
|
\begin{enumerate}
|
||||||
|
\item Démontrer que $x=3$ et $x=6$ sont deux racines de $N(x)$..
|
||||||
|
\item Proposer une forme factorisée de $N(x)$.
|
||||||
|
\item Proposer une forme factorisée de $f'(x)$.
|
||||||
|
\end{enumerate}
|
||||||
|
\item Étudier le signe de $f'$ et en déduire les variations de $f$.
|
||||||
|
\end{enumerate}
|
||||||
|
\end{exercise}
|
||||||
|
|
||||||
|
\begin{solution}
|
||||||
|
\begin{enumerate}
|
||||||
|
\item pas de correction disponible
|
||||||
|
\item
|
||||||
|
\begin{enumerate}
|
||||||
|
\item \[N(3) = 0\]
|
||||||
|
\[N(6) = 0\]
|
||||||
|
\item \[
|
||||||
|
N(x) = 6(x - 3)(x - 6)
|
||||||
|
\]
|
||||||
|
\item
|
||||||
|
\[
|
||||||
|
f'(x) = \frac{6(x - 3)(x - 6)}{x}
|
||||||
|
\]
|
||||||
|
\end{enumerate}
|
||||||
|
\item Pas de correction disponible
|
||||||
|
\end{enumerate}
|
||||||
|
\end{solution}
|
||||||
|
|
||||||
|
\begin{exercise}[subtitle={Complexes}]
|
||||||
|
\begin{enumerate}
|
||||||
|
\item Mettre le nombre complexe suivant sous forme algébrique $z_1 = \dfrac{7 + 9 i}{-3 + 7 i} $
|
||||||
|
\item Mettre le complexe suivante sous forme exponentielle $z_2 = -6 + 6 \sqrt{3} i$
|
||||||
|
\item Mettre le complexe suivante sous forme exponentielle $z_3 = 10 + 10 \sqrt{3} i$
|
||||||
|
\item Calculer le produit $z_4=z_2\times z_3$ donner le résultat sous forme exponentielle puis algébrique.
|
||||||
|
\item Calculer le quotient $z_5=\frac{z_2}{z_3}$ donner le résultat sous forme exponentielle puis algébrique.
|
||||||
|
\end{enumerate}
|
||||||
|
\end{exercise}
|
||||||
|
|
||||||
|
\begin{solution}
|
||||||
|
\begin{enumerate}
|
||||||
|
\item $z_1 = \frac{21}{29} - \frac{38 i}{29}$
|
||||||
|
\item $z_2 = 12 e^{\frac{2 i \pi}{3}}$
|
||||||
|
\item $z_3 = 20 e^{\frac{i \pi}{3}}$
|
||||||
|
\item $z_4 = 240 e^{i \pi} = -240 = -240.0$
|
||||||
|
\item $z_5 = \frac{3}{5} e^{\frac{i \pi}{3}} = \frac{3}{10} + \frac{3 \sqrt{3} i}{10} = 0.3 + 0.52 i$
|
||||||
|
\end{enumerate}
|
||||||
|
\end{solution}
|
||||||
|
|
||||||
|
\begin{exercise}[subtitle={Sortie du congélateur}]
|
||||||
|
Marie a invité quelques amis pour le thé. Elle souhaite leur proposer ses macarons maison.
|
||||||
|
|
||||||
|
Elle les sort de son congélateur à $-20$~\degres C et les place dans une pièce à $16$~\degres C.
|
||||||
|
|
||||||
|
Au bout de 15 minutes, la température des macarons est de $4$~\degres C.
|
||||||
|
|
||||||
|
\bigskip
|
||||||
|
|
||||||
|
\textbf{Premier modèle}
|
||||||
|
|
||||||
|
\medskip
|
||||||
|
|
||||||
|
On suppose que la vitesse de décongélation est constante : chaque minute la hausse de
|
||||||
|
température des macarons est la même.
|
||||||
|
|
||||||
|
Estimer dans ce cadre la température au bout de $30$~minutes, puis au bout de $45$~minutes.
|
||||||
|
|
||||||
|
Cette modélisation est-elle pertinente?
|
||||||
|
|
||||||
|
\bigskip
|
||||||
|
|
||||||
|
\textbf{Deuxième modèle}
|
||||||
|
|
||||||
|
\medskip
|
||||||
|
|
||||||
|
On suppose maintenant que la vitesse de décongélation est proportionnelle à la différence
|
||||||
|
de température entre les macarons et l'air ambiant (il s'agit de la loi de Newton).
|
||||||
|
|
||||||
|
On désigne par $\theta$ la température des macarons à l'instant $t$, et par $\theta'$ la vitesse de décongélation.
|
||||||
|
|
||||||
|
L'unité de temps est la minute et l'unité de température le degré Celsius.
|
||||||
|
|
||||||
|
\smallskip
|
||||||
|
|
||||||
|
On négligera la diminution de température de la pièce et on admettra donc qu'il existe un
|
||||||
|
nombre réel $a$ tel que, pour $t$ positif :
|
||||||
|
|
||||||
|
\[\theta'(t) = a [\theta(t) - 16]\quad (E)\]
|
||||||
|
|
||||||
|
\medskip
|
||||||
|
|
||||||
|
\begin{enumerate}
|
||||||
|
\item Vérifier que l'équation $(E)$ a pour solutions $\theta(t) = K e^{at} + 16$ où $K$ est un nombre réel.
|
||||||
|
|
||||||
|
Donner alors, en fonction de $a$, l'ensemble des solutions de $(E)$.
|
||||||
|
\end{enumerate}
|
||||||
|
On rappelle que la température des macarons à l'instant $t = 0$ est égale à $-20$~\degres C et que, au bout de $15$~min, elle est de $4$~\degres C.
|
||||||
|
\begin{enumerate}
|
||||||
|
\setcounter{enumi}{1}
|
||||||
|
\item En utilisant la condition à $t=0$ démontrer que $K = -36$.
|
||||||
|
\item En utilisant la condition à $t=15$ démontrer que $a \approx -0.07$.
|
||||||
|
\item En déduire l'expression de la solution de l'équation différentielle puis étudier ses variations.
|
||||||
|
\item La température idéale de dégustation des macarons étant de $13$~\degres C, Marie estime que
|
||||||
|
celle-ci sera atteinte au bout de $30$~min. A-t-elle raison ? Justifier la réponse.
|
||||||
|
|
||||||
|
Sinon, combien de temps faudra-t-il attendre ?
|
||||||
|
\end{enumerate}
|
||||||
|
\end{exercise}
|
||||||
|
|
||||||
|
|
||||||
|
|
||||||
|
\end{document}
|
||||||
|
|
||||||
|
%%% Local Variables:
|
||||||
|
%%% mode: latex
|
||||||
|
%%% TeX-master: "master"
|
||||||
|
%%% End:
|
|
@ -0,0 +1,136 @@
|
||||||
|
\documentclass[a4paper,10pt]{article}
|
||||||
|
\usepackage{myXsim}
|
||||||
|
|
||||||
|
% Title Page
|
||||||
|
\title{DS8 \hfill TAVERNIER Joanny}
|
||||||
|
\tribe{TST sti2d}
|
||||||
|
\date{\hfillÀ render pour le vendredi 9 avril à 10h au plus tard}
|
||||||
|
|
||||||
|
\xsimsetup{
|
||||||
|
solution/print = true
|
||||||
|
}
|
||||||
|
|
||||||
|
\begin{document}
|
||||||
|
\maketitle
|
||||||
|
|
||||||
|
\begin{exercise}[subtitle={Étude de fonction}]
|
||||||
|
On considère la fonction $f$ définie sur $\intOF{0}{+\infty}$ par $ f(x) = 4.5x^2 + - 144x + 567\ln(x)$
|
||||||
|
\begin{enumerate}
|
||||||
|
\item Démontrer que la dérivée de $f$ est $f'(x) = \frac{9x^2 + - 144x + 567}{x}$.
|
||||||
|
\item Étude du numérateur de $f'(x)$: $N(x) = 9x^2 - 144x + 567$
|
||||||
|
\begin{enumerate}
|
||||||
|
\item Démontrer que $x=9$ et $x=7$ sont deux racines de $N(x)$..
|
||||||
|
\item Proposer une forme factorisée de $N(x)$.
|
||||||
|
\item Proposer une forme factorisée de $f'(x)$.
|
||||||
|
\end{enumerate}
|
||||||
|
\item Étudier le signe de $f'$ et en déduire les variations de $f$.
|
||||||
|
\end{enumerate}
|
||||||
|
\end{exercise}
|
||||||
|
|
||||||
|
\begin{solution}
|
||||||
|
\begin{enumerate}
|
||||||
|
\item pas de correction disponible
|
||||||
|
\item
|
||||||
|
\begin{enumerate}
|
||||||
|
\item \[N(9) = 0\]
|
||||||
|
\[N(7) = 0\]
|
||||||
|
\item \[
|
||||||
|
N(x) = 9(x - 9)(x - 7)
|
||||||
|
\]
|
||||||
|
\item
|
||||||
|
\[
|
||||||
|
f'(x) = \frac{9(x - 9)(x - 7)}{x}
|
||||||
|
\]
|
||||||
|
\end{enumerate}
|
||||||
|
\item Pas de correction disponible
|
||||||
|
\end{enumerate}
|
||||||
|
\end{solution}
|
||||||
|
|
||||||
|
\begin{exercise}[subtitle={Complexes}]
|
||||||
|
\begin{enumerate}
|
||||||
|
\item Mettre le nombre complexe suivant sous forme algébrique $z_1 = \dfrac{10 + 6 i}{-4 + 5 i} $
|
||||||
|
\item Mettre le complexe suivante sous forme exponentielle $z_2 = - 8 \sqrt{3} + 8 i$
|
||||||
|
\item Mettre le complexe suivante sous forme exponentielle $z_3 = 5 - 5 \sqrt{3} i$
|
||||||
|
\item Calculer le produit $z_4=z_2\times z_3$ donner le résultat sous forme exponentielle puis algébrique.
|
||||||
|
\item Calculer le quotient $z_5=\frac{z_2}{z_3}$ donner le résultat sous forme exponentielle puis algébrique.
|
||||||
|
\end{enumerate}
|
||||||
|
\end{exercise}
|
||||||
|
|
||||||
|
\begin{solution}
|
||||||
|
\begin{enumerate}
|
||||||
|
\item $z_1 = - \frac{10}{41} - \frac{74 i}{41}$
|
||||||
|
\item $z_2 = 16 e^{\frac{5 i \pi}{6}}$
|
||||||
|
\item $z_3 = 10 e^{- \frac{i \pi}{3}}$
|
||||||
|
\item $z_4 = 160 e^{\frac{i \pi}{2}} = 160 i = 160.0 i$
|
||||||
|
\item $z_5 = \frac{8}{5} e^{\frac{7 i \pi}{6}} = - \frac{4 \sqrt{3}}{5} - \frac{4 i}{5} = -1.39 - 0.8 i$
|
||||||
|
\end{enumerate}
|
||||||
|
\end{solution}
|
||||||
|
|
||||||
|
\begin{exercise}[subtitle={Sortie du congélateur}]
|
||||||
|
Marie a invité quelques amis pour le thé. Elle souhaite leur proposer ses macarons maison.
|
||||||
|
|
||||||
|
Elle les sort de son congélateur à $-19$~\degres C et les place dans une pièce à $21$~\degres C.
|
||||||
|
|
||||||
|
Au bout de 15 minutes, la température des macarons est de $-4$~\degres C.
|
||||||
|
|
||||||
|
\bigskip
|
||||||
|
|
||||||
|
\textbf{Premier modèle}
|
||||||
|
|
||||||
|
\medskip
|
||||||
|
|
||||||
|
On suppose que la vitesse de décongélation est constante : chaque minute la hausse de
|
||||||
|
température des macarons est la même.
|
||||||
|
|
||||||
|
Estimer dans ce cadre la température au bout de $30$~minutes, puis au bout de $45$~minutes.
|
||||||
|
|
||||||
|
Cette modélisation est-elle pertinente?
|
||||||
|
|
||||||
|
\bigskip
|
||||||
|
|
||||||
|
\textbf{Deuxième modèle}
|
||||||
|
|
||||||
|
\medskip
|
||||||
|
|
||||||
|
On suppose maintenant que la vitesse de décongélation est proportionnelle à la différence
|
||||||
|
de température entre les macarons et l'air ambiant (il s'agit de la loi de Newton).
|
||||||
|
|
||||||
|
On désigne par $\theta$ la température des macarons à l'instant $t$, et par $\theta'$ la vitesse de décongélation.
|
||||||
|
|
||||||
|
L'unité de temps est la minute et l'unité de température le degré Celsius.
|
||||||
|
|
||||||
|
\smallskip
|
||||||
|
|
||||||
|
On négligera la diminution de température de la pièce et on admettra donc qu'il existe un
|
||||||
|
nombre réel $a$ tel que, pour $t$ positif :
|
||||||
|
|
||||||
|
\[\theta'(t) = a [\theta(t) - 21]\quad (E)\]
|
||||||
|
|
||||||
|
\medskip
|
||||||
|
|
||||||
|
\begin{enumerate}
|
||||||
|
\item Vérifier que l'équation $(E)$ a pour solutions $\theta(t) = K e^{at} + 21$ où $K$ est un nombre réel.
|
||||||
|
|
||||||
|
Donner alors, en fonction de $a$, l'ensemble des solutions de $(E)$.
|
||||||
|
\end{enumerate}
|
||||||
|
On rappelle que la température des macarons à l'instant $t = 0$ est égale à $-19$~\degres C et que, au bout de $15$~min, elle est de $-4$~\degres C.
|
||||||
|
\begin{enumerate}
|
||||||
|
\setcounter{enumi}{1}
|
||||||
|
\item En utilisant la condition à $t=0$ démontrer que $K = -40$.
|
||||||
|
\item En utilisant la condition à $t=15$ démontrer que $a \approx -0.03$.
|
||||||
|
\item En déduire l'expression de la solution de l'équation différentielle puis étudier ses variations.
|
||||||
|
\item La température idéale de dégustation des macarons étant de $18$~\degres C, Marie estime que
|
||||||
|
celle-ci sera atteinte au bout de $30$~min. A-t-elle raison ? Justifier la réponse.
|
||||||
|
|
||||||
|
Sinon, combien de temps faudra-t-il attendre ?
|
||||||
|
\end{enumerate}
|
||||||
|
\end{exercise}
|
||||||
|
|
||||||
|
|
||||||
|
|
||||||
|
\end{document}
|
||||||
|
|
||||||
|
%%% Local Variables:
|
||||||
|
%%% mode: latex
|
||||||
|
%%% TeX-master: "master"
|
||||||
|
%%% End:
|
|
@ -0,0 +1,136 @@
|
||||||
|
\documentclass[a4paper,10pt]{article}
|
||||||
|
\usepackage{myXsim}
|
||||||
|
|
||||||
|
% Title Page
|
||||||
|
\title{DS8 \hfill ZAHORE Zahiri}
|
||||||
|
\tribe{TST sti2d}
|
||||||
|
\date{\hfillÀ render pour le vendredi 9 avril à 10h au plus tard}
|
||||||
|
|
||||||
|
\xsimsetup{
|
||||||
|
solution/print = true
|
||||||
|
}
|
||||||
|
|
||||||
|
\begin{document}
|
||||||
|
\maketitle
|
||||||
|
|
||||||
|
\begin{exercise}[subtitle={Étude de fonction}]
|
||||||
|
On considère la fonction $f$ définie sur $\intOF{0}{+\infty}$ par $ f(x) = 4.5x^2 + - 54x + - 63\ln(x)$
|
||||||
|
\begin{enumerate}
|
||||||
|
\item Démontrer que la dérivée de $f$ est $f'(x) = \frac{9x^2 + - 54x + - 63}{x}$.
|
||||||
|
\item Étude du numérateur de $f'(x)$: $N(x) = 9x^2 - 54x - 63$
|
||||||
|
\begin{enumerate}
|
||||||
|
\item Démontrer que $x=7$ et $x=- 1$ sont deux racines de $N(x)$..
|
||||||
|
\item Proposer une forme factorisée de $N(x)$.
|
||||||
|
\item Proposer une forme factorisée de $f'(x)$.
|
||||||
|
\end{enumerate}
|
||||||
|
\item Étudier le signe de $f'$ et en déduire les variations de $f$.
|
||||||
|
\end{enumerate}
|
||||||
|
\end{exercise}
|
||||||
|
|
||||||
|
\begin{solution}
|
||||||
|
\begin{enumerate}
|
||||||
|
\item pas de correction disponible
|
||||||
|
\item
|
||||||
|
\begin{enumerate}
|
||||||
|
\item \[N(7) = 0\]
|
||||||
|
\[N(- 1) = 0\]
|
||||||
|
\item \[
|
||||||
|
N(x) = 9(x - 7)(x - - 1)
|
||||||
|
\]
|
||||||
|
\item
|
||||||
|
\[
|
||||||
|
f'(x) = \frac{9(x - 7)(x - - 1)}{x}
|
||||||
|
\]
|
||||||
|
\end{enumerate}
|
||||||
|
\item Pas de correction disponible
|
||||||
|
\end{enumerate}
|
||||||
|
\end{solution}
|
||||||
|
|
||||||
|
\begin{exercise}[subtitle={Complexes}]
|
||||||
|
\begin{enumerate}
|
||||||
|
\item Mettre le nombre complexe suivant sous forme algébrique $z_1 = \dfrac{7 + 4 i}{-4 + 8 i} $
|
||||||
|
\item Mettre le complexe suivante sous forme exponentielle $z_2 = 4 \sqrt{2} + 4 \sqrt{2} i$
|
||||||
|
\item Mettre le complexe suivante sous forme exponentielle $z_3 = - 4 \sqrt{2} - 4 \sqrt{2} i$
|
||||||
|
\item Calculer le produit $z_4=z_2\times z_3$ donner le résultat sous forme exponentielle puis algébrique.
|
||||||
|
\item Calculer le quotient $z_5=\frac{z_2}{z_3}$ donner le résultat sous forme exponentielle puis algébrique.
|
||||||
|
\end{enumerate}
|
||||||
|
\end{exercise}
|
||||||
|
|
||||||
|
\begin{solution}
|
||||||
|
\begin{enumerate}
|
||||||
|
\item $z_1 = \frac{1}{20} - \frac{9 i}{10}$
|
||||||
|
\item $z_2 = 8 e^{\frac{i \pi}{4}}$
|
||||||
|
\item $z_3 = 8 e^{- \frac{3 i \pi}{4}}$
|
||||||
|
\item $z_4 = 64 e^{- \frac{i \pi}{2}} = - 64 i = - 64.0 i$
|
||||||
|
\item $z_5 = 1 e^{i \pi} = -1 = -1.0$
|
||||||
|
\end{enumerate}
|
||||||
|
\end{solution}
|
||||||
|
|
||||||
|
\begin{exercise}[subtitle={Sortie du congélateur}]
|
||||||
|
Marie a invité quelques amis pour le thé. Elle souhaite leur proposer ses macarons maison.
|
||||||
|
|
||||||
|
Elle les sort de son congélateur à $-18$~\degres C et les place dans une pièce à $21$~\degres C.
|
||||||
|
|
||||||
|
Au bout de 15 minutes, la température des macarons est de $4$~\degres C.
|
||||||
|
|
||||||
|
\bigskip
|
||||||
|
|
||||||
|
\textbf{Premier modèle}
|
||||||
|
|
||||||
|
\medskip
|
||||||
|
|
||||||
|
On suppose que la vitesse de décongélation est constante : chaque minute la hausse de
|
||||||
|
température des macarons est la même.
|
||||||
|
|
||||||
|
Estimer dans ce cadre la température au bout de $30$~minutes, puis au bout de $45$~minutes.
|
||||||
|
|
||||||
|
Cette modélisation est-elle pertinente?
|
||||||
|
|
||||||
|
\bigskip
|
||||||
|
|
||||||
|
\textbf{Deuxième modèle}
|
||||||
|
|
||||||
|
\medskip
|
||||||
|
|
||||||
|
On suppose maintenant que la vitesse de décongélation est proportionnelle à la différence
|
||||||
|
de température entre les macarons et l'air ambiant (il s'agit de la loi de Newton).
|
||||||
|
|
||||||
|
On désigne par $\theta$ la température des macarons à l'instant $t$, et par $\theta'$ la vitesse de décongélation.
|
||||||
|
|
||||||
|
L'unité de temps est la minute et l'unité de température le degré Celsius.
|
||||||
|
|
||||||
|
\smallskip
|
||||||
|
|
||||||
|
On négligera la diminution de température de la pièce et on admettra donc qu'il existe un
|
||||||
|
nombre réel $a$ tel que, pour $t$ positif :
|
||||||
|
|
||||||
|
\[\theta'(t) = a [\theta(t) - 21]\quad (E)\]
|
||||||
|
|
||||||
|
\medskip
|
||||||
|
|
||||||
|
\begin{enumerate}
|
||||||
|
\item Vérifier que l'équation $(E)$ a pour solutions $\theta(t) = K e^{at} + 21$ où $K$ est un nombre réel.
|
||||||
|
|
||||||
|
Donner alors, en fonction de $a$, l'ensemble des solutions de $(E)$.
|
||||||
|
\end{enumerate}
|
||||||
|
On rappelle que la température des macarons à l'instant $t = 0$ est égale à $-18$~\degres C et que, au bout de $15$~min, elle est de $4$~\degres C.
|
||||||
|
\begin{enumerate}
|
||||||
|
\setcounter{enumi}{1}
|
||||||
|
\item En utilisant la condition à $t=0$ démontrer que $K = -39$.
|
||||||
|
\item En utilisant la condition à $t=15$ démontrer que $a \approx -0.06$.
|
||||||
|
\item En déduire l'expression de la solution de l'équation différentielle puis étudier ses variations.
|
||||||
|
\item La température idéale de dégustation des macarons étant de $18$~\degres C, Marie estime que
|
||||||
|
celle-ci sera atteinte au bout de $30$~min. A-t-elle raison ? Justifier la réponse.
|
||||||
|
|
||||||
|
Sinon, combien de temps faudra-t-il attendre ?
|
||||||
|
\end{enumerate}
|
||||||
|
\end{exercise}
|
||||||
|
|
||||||
|
|
||||||
|
|
||||||
|
\end{document}
|
||||||
|
|
||||||
|
%%% Local Variables:
|
||||||
|
%%% mode: latex
|
||||||
|
%%% TeX-master: "master"
|
||||||
|
%%% End:
|
Binary file not shown.
|
@ -0,0 +1,165 @@
|
||||||
|
\documentclass[a4paper,10pt]{article}
|
||||||
|
\usepackage{myXsim}
|
||||||
|
|
||||||
|
% Title Page
|
||||||
|
\title{DS8 \hfill \Var{Nom}}
|
||||||
|
\tribe{TST sti2d}
|
||||||
|
\date{\hfillÀ render pour le vendredi 9 avril à 10h au plus tard}
|
||||||
|
|
||||||
|
\xsimsetup{
|
||||||
|
solution/print = false
|
||||||
|
}
|
||||||
|
|
||||||
|
\begin{document}
|
||||||
|
\maketitle
|
||||||
|
|
||||||
|
\begin{exercise}[subtitle={Étude de fonction}]
|
||||||
|
%- set a = Integer.random(-5, 5)
|
||||||
|
%- set A = (a/2).decimal
|
||||||
|
%- set x1 = Integer.random()
|
||||||
|
%- set x2 = Integer.random(rejected=[0, 1, x1])
|
||||||
|
%- set b = -1* a*(x1+x2)
|
||||||
|
%- set c = a*x1*x2
|
||||||
|
|
||||||
|
%- set N = Polynom.from_coefficients([c._mo, b._mo, a._mo])
|
||||||
|
On considère la fonction $f$ définie sur $\intOF{0}{+\infty}$ par $ f(x) = \Var{A}x^2 + \Var{b}x + \Var{c}\ln(x)$
|
||||||
|
\begin{enumerate}
|
||||||
|
\item Démontrer que la dérivée de $f$ est $f'(x) = \frac{\Var{a}x^2 + \Var{b}x + \Var{c}}{x}$.
|
||||||
|
\item Étude du numérateur de $f'(x)$: $N(x) = \Var{N}$
|
||||||
|
\begin{enumerate}
|
||||||
|
\item Démontrer que $x=\Var{x1}$ et $x=\Var{x2}$ sont deux racines de $N(x)$..
|
||||||
|
\item Proposer une forme factorisée de $N(x)$.
|
||||||
|
\item Proposer une forme factorisée de $f'(x)$.
|
||||||
|
\end{enumerate}
|
||||||
|
\item Étudier le signe de $f'$ et en déduire les variations de $f$.
|
||||||
|
\end{enumerate}
|
||||||
|
\end{exercise}
|
||||||
|
|
||||||
|
\begin{solution}
|
||||||
|
\begin{enumerate}
|
||||||
|
\item pas de correction disponible
|
||||||
|
\item
|
||||||
|
\begin{enumerate}
|
||||||
|
\item \[N(\Var{x1}) = \Var{N(x1)}\]
|
||||||
|
\[N(\Var{x2}) = \Var{N(x2)}\]
|
||||||
|
\item \[
|
||||||
|
N(x) = \Var{a}(x - \Var{x1})(x - \Var{x2})
|
||||||
|
\]
|
||||||
|
\item
|
||||||
|
\[
|
||||||
|
f'(x) = \frac{\Var{a}(x - \Var{x1})(x - \Var{x2})}{x}
|
||||||
|
\]
|
||||||
|
\end{enumerate}
|
||||||
|
\item Pas de correction disponible
|
||||||
|
\end{enumerate}
|
||||||
|
\end{solution}
|
||||||
|
|
||||||
|
%- set I = sympy.I
|
||||||
|
%- set latex = sympy.latex
|
||||||
|
%- set sqrt = sympy.sqrt
|
||||||
|
%- set exp = sympy.functions.exp
|
||||||
|
%- set integrate = sympy.integrate
|
||||||
|
\begin{exercise}[subtitle={Complexes}]
|
||||||
|
\begin{enumerate}
|
||||||
|
%- set z_num = randint(2, 10) + I*randint(2, 10)
|
||||||
|
%- set z_denom = -randint(2, 10) + I*randint(2, 10)
|
||||||
|
%- set z1 = z_num / z_denom
|
||||||
|
\item Mettre le nombre complexe suivant sous forme algébrique $z_1 = \dfrac{\Var{latex(z_num)}}{\Var{latex(z_denom)}} $
|
||||||
|
%- set base = choice([(1, sqrt(3)), (sqrt(2), sqrt(2)), (sqrt(3), 1)])
|
||||||
|
%- set z2 = randint(1, 10)*(choice([1, -1])*base[0] + choice([1, -1])*base[1]*I)
|
||||||
|
\item Mettre le complexe suivante sous forme exponentielle $z_2 = \Var{latex(z2)}$
|
||||||
|
%- set base = choice([(1, sqrt(3)), (sqrt(2), sqrt(2)), (sqrt(3), 1)])
|
||||||
|
%- set z3 = randint(1, 10)*(choice([1, -1])*base[0] + choice([1, -1])*base[1]*I)
|
||||||
|
\item Mettre le complexe suivante sous forme exponentielle $z_3 = \Var{latex(z3)}$
|
||||||
|
%- set z4 = z2*z3
|
||||||
|
\item Calculer le produit $z_4=z_2\times z_3$ donner le résultat sous forme exponentielle puis algébrique.
|
||||||
|
%- set z5 = z2/z3
|
||||||
|
\item Calculer le quotient $z_5=\frac{z_2}{z_3}$ donner le résultat sous forme exponentielle puis algébrique.
|
||||||
|
\end{enumerate}
|
||||||
|
\end{exercise}
|
||||||
|
|
||||||
|
\begin{solution}
|
||||||
|
\begin{enumerate}
|
||||||
|
\item $z_1 = \Var{latex(sympy.re(z1) + sympy.im(z1)*I)}$
|
||||||
|
\item $z_2 = \Var{latex(sympy.Abs(z2))} e^{\Var{latex(I*sympy.arg(z2))}}$
|
||||||
|
\item $z_3 = \Var{latex(sympy.Abs(z3))} e^{\Var{latex(I*sympy.arg(z3))}}$
|
||||||
|
\item $z_4 = \Var{latex(sympy.Abs(z4))} e^{\Var{latex(I*(sympy.arg(z2) + sympy.arg(z3)))}} = \Var{latex(sympy.re(z4) + sympy.im(z4)*I)} = \Var{latex(sympy.N(sympy.re(z4), 3)+ sympy.N(sympy.im(z4), 3)*I)}$
|
||||||
|
\item $z_5 = \Var{latex(sympy.Abs(z5))} e^{\Var{latex(I*(sympy.arg(z2) - sympy.arg(z3)))}} = \Var{latex(sympy.re(z5) + sympy.im(z5)*I)} = \Var{latex(sympy.N(sympy.re(z5), 3)+ sympy.N(sympy.im(z5), 3)*I)}$
|
||||||
|
\end{enumerate}
|
||||||
|
\end{solution}
|
||||||
|
|
||||||
|
\begin{exercise}[subtitle={Sortie du congélateur}]
|
||||||
|
%- set Tp = randint(15, 25)
|
||||||
|
%- set T0 = randint(-20, -15)
|
||||||
|
%- set T15 = randint(-4, 4)
|
||||||
|
|
||||||
|
Marie a invité quelques amis pour le thé. Elle souhaite leur proposer ses macarons maison.
|
||||||
|
|
||||||
|
Elle les sort de son congélateur à $\Var{T0}$~\degres C et les place dans une pièce à $\Var{Tp}$~\degres C.
|
||||||
|
|
||||||
|
Au bout de 15 minutes, la température des macarons est de $\Var{T15}$~\degres C.
|
||||||
|
|
||||||
|
\bigskip
|
||||||
|
|
||||||
|
\textbf{Premier modèle}
|
||||||
|
|
||||||
|
\medskip
|
||||||
|
|
||||||
|
On suppose que la vitesse de décongélation est constante : chaque minute la hausse de
|
||||||
|
température des macarons est la même.
|
||||||
|
|
||||||
|
Estimer dans ce cadre la température au bout de $30$~minutes, puis au bout de $45$~minutes.
|
||||||
|
|
||||||
|
Cette modélisation est-elle pertinente?
|
||||||
|
|
||||||
|
\bigskip
|
||||||
|
|
||||||
|
\textbf{Deuxième modèle}
|
||||||
|
|
||||||
|
\medskip
|
||||||
|
|
||||||
|
On suppose maintenant que la vitesse de décongélation est proportionnelle à la différence
|
||||||
|
de température entre les macarons et l'air ambiant (il s'agit de la loi de Newton).
|
||||||
|
|
||||||
|
On désigne par $\theta$ la température des macarons à l'instant $t$, et par $\theta'$ la vitesse de décongélation.
|
||||||
|
|
||||||
|
L'unité de temps est la minute et l'unité de température le degré Celsius.
|
||||||
|
|
||||||
|
\smallskip
|
||||||
|
|
||||||
|
On négligera la diminution de température de la pièce et on admettra donc qu'il existe un
|
||||||
|
nombre réel $a$ tel que, pour $t$ positif :
|
||||||
|
|
||||||
|
\[\theta'(t) = a [\theta(t) - \Var{Tp}]\quad (E)\]
|
||||||
|
|
||||||
|
\medskip
|
||||||
|
|
||||||
|
\begin{enumerate}
|
||||||
|
\item Vérifier que l'équation $(E)$ a pour solutions $\theta(t) = K e^{at} + \Var{Tp}$ où $K$ est un nombre réel.
|
||||||
|
|
||||||
|
Donner alors, en fonction de $a$, l'ensemble des solutions de $(E)$.
|
||||||
|
\end{enumerate}
|
||||||
|
On rappelle que la température des macarons à l'instant $t = 0$ est égale à $\Var{T0}$~\degres C et que, au bout de $15$~min, elle est de $\Var{T15}$~\degres C.
|
||||||
|
\begin{enumerate}
|
||||||
|
\setcounter{enumi}{1}
|
||||||
|
%- set k = T0 - Tp
|
||||||
|
\item En utilisant la condition à $t=0$ démontrer que $K = \Var{k}$.
|
||||||
|
%- set a = round(log((T15 - Tp)/k)/15, 2)
|
||||||
|
\item En utilisant la condition à $t=15$ démontrer que $a \approx \Var{a}$.
|
||||||
|
\item En déduire l'expression de la solution de l'équation différentielle puis étudier ses variations.
|
||||||
|
\item La température idéale de dégustation des macarons étant de $\Var{Tp-3}$~\degres C, Marie estime que
|
||||||
|
celle-ci sera atteinte au bout de $30$~min. A-t-elle raison ? Justifier la réponse.
|
||||||
|
|
||||||
|
Sinon, combien de temps faudra-t-il attendre ?
|
||||||
|
\end{enumerate}
|
||||||
|
\end{exercise}
|
||||||
|
|
||||||
|
|
||||||
|
|
||||||
|
\end{document}
|
||||||
|
|
||||||
|
%%% Local Variables:
|
||||||
|
%%% mode: latex
|
||||||
|
%%% TeX-master: "master"
|
||||||
|
%%% End:
|
||||||
|
|
Loading…
Reference in New Issue